NONRESIDENT TRAINING COURSE SEPTEMBER 1998 Navy Electricity and Electronics Training Series Module 1 — Introduction to Matter, Energy, and Direct Current NAVEDTRA 14173 DISTRIBUTION STATEMENT A: Approved for public release; distribution is unlimited. DISTRIBUTION STATEMENT A: Approved for public release; distribution is unlimited. PREFACE By enrolling in this self-study course, you have demonstrated a desire to improve yourself and the Navy. Remember, however, this self-study course is only one part of the total Navy training program. Practical experience, schools, selected reading, and your desire to succeed are also necessary to successfully round out a fully meaningful training program. COURSE OVERVIEW: To introduce the student to the subject of Matter, Energy, and Direct Current who needs such a background in accomplishing daily work and/or in preparing for further study. THE COURSE: This self-study course is organized into subject matter areas, each containing learning objectives to help you determine what you should learn along with text and illustrations to help you understand the information. The subject matter reflects day-to-day requirements and experiences of personnel in the rating or skill area. It also reflects guidance provided by Enlisted Community Managers (ECMs) and other senior personnel, technical references, instructions, etc., and either the occupational or naval standards, which are listed in the Manual of Navy Enlisted Manpower Personnel Classifications and Occupational Standards , NAVPERS 18068. THE QUESTIONS: The questions that appear in this course are designed to help you understand the material in the text. VALUE: In completing this course, you will improve your military and professional knowledge. Importantly, it can also help you study for the Navy-wide advancement in rate examination. If you are studying and discover a reference in the text to another publication for further information, look it up. 1998 Edition Prepared by ETCS(SW) Donnie Jones Published by NAVAL EDUCATION AND TRAINING PROFESSIONAL DEVELOPMENT AND TECHNOLOGY CENTER NAVSUP Logistics Tracking Number 0504-LP-026-8260 Sailor’s Creed “I am a United States Sailor. I will support and defend the Constitution of the United States of America and I will obey the orders of those appointed over me. I represent the fighting spirit of the Navy and those who have gone before me to defend freedom and democracy around the world. I proudly serve my country’s Navy combat team with honor, courage and commitment. I am committed to excellence and the fair treatment of all.” TABLE OF CONTENTS CHAPTER PAGE 1. Matter, Energy, and Electricity 1-1 2. Batteries 2-1 3. Direct Current 3-1 APPENDIX I. Glossary AI-1 II. Laws of Exponents AII-1 III. Square and Square Roots AIII-1 IV. Comparison of Units in Electric and Magnetic Circuits; and Carbon Resistor Size Comparison by Wattage Rating AIV-1 V. Useful Formulas for I.C. Circuits AV-1 INDEX INDEX- 1 NAVY ELECTRICITY AND ELECTRONICS TRAINING SERIES The Navy Electricity and Electronics Training Series (NEETS) was developed for use by personnel in many electrical- and electronic-related Navy ratings. Written by, and with the advice of, senior technicians in these ratings, this series provides beginners with fundamental electrical and electronic concepts through self-study. The presentation of this series is not oriented to any specific rating structure, but is divided into modules containing related information organized into traditional paths of instruction. The series is designed to give small amounts of information that can be easily digested before advancing further into the more complex material. For a student just becoming acquainted with electricity or electronics, it is highly recommended that the modules be studied in their suggested sequence. While there is a listing of NEETS by module title, the following brief descriptions give a quick overview of how the individual modules flow together. Module 1, Introduction to Matter , Energy, and Direct Current , introduces the course with a short history of electricity and electronics and proceeds into the characteristics of matter, energy, and direct current (dc). It also describes some of the general safety precautions and first-aid procedures that should be common knowledge for a person working in the field of electricity. Related safety hints are located throughout the rest of the series, as well. Module 2, Introduction to Alternating Current and Transformers, is an introduction to alternating current (ac) and transformers, including basic ac theory and fundamentals of electromagnetism, inductance, capacitance, impedance, and transformers. Module 3, Introduction to Circuit Protection, Control, and Measurement, encompasses circuit breakers, fuses, and current limiters used in circuit protection, as well as the theory and use of meters as electrical measuring devices. Module 4, Introduction to Electrical Conductors, Wiring Techniques, and Schematic Reading, presents conductor usage, insulation used as wire covering, splicing, termination of wiring, soldering, and reading electrical wiring diagrams. Module 5, Introduction to Generators and Motors, is an introduction to generators and motors, and covers the uses of ac and dc generators and motors in the conversion of electrical and mechanical energies. Module 6, Introduction to Electronic Emission, Tubes, and Power Supplies, ties the first five modules together in an introduction to vacuum tubes and vacuum-tube power supplies. Module 7, Introduction to Solid-State Devices and Power Supplies, is similar to module 6, but it is in reference to solid-state devices. Module 8, Introduction to Amplifiers, covers amplifiers. Module 9 , Introduction to Wave-Generation and Wave-Shaping Circuits, discusses wave generation and wave-shaping circuits. Module 10, Introduction to Wave Propagation, Transmission Lines, and Antennas, presents the characteristics of wave propagation, transmission lines, and antennas. IV Module 11, Microwave Principles, explains microwave oscillators, amplifiers, and waveguides. Module 12, Modulation Principles, discusses the principles of modulation. Module 13, Introduction to Number Systems and Logic Circuits, presents the fundamental concepts of number systems, Boolean algebra, and logic circuits, all of which pertain to digital computers. Module 14, Introduction to Microelectronics, covers microelectronics technology and miniature and microminiature circuit repair. Module 15, Principles of Synchros, Servos, and Gyros, provides the basic principles, operations, functions, and applications of synchro, servo, and gyro mechanisms. Module 16, Introduction to Test Equipment, is an introduction to some of the more commonly used test equipments and their applications. Module 17, Radio-Frequency Communications Principles, presents the fundamentals of a radio- frequency communications system. Module 18, Radar Principles, covers the fundamentals of a radar system. Module 19, The Technician's Handbook, is a handy reference of commonly used general information, such as electrical and electronic formulas, color coding, and naval supply system data. Module 20, Master Glossary, is the glossary of terms for the series. Module 21, Test Methods and Practices, describes basic test methods and practices. Module 22, Introduction to Digital Computers, is an introduction to digital computers. Module 23, Magnetic Recording, is an introduction to the use and maintenance of magnetic recorders and the concepts of recording on magnetic tape and disks. Module 24, Introduction to Fiber Optics, is an introduction to fiber optics. Embedded questions are inserted throughout each module, except for modules 19 and 20, which are reference books. If you have any difficulty in answering any of the questions, restudy the applicable section. Although an attempt has been made to use simple language, various technical words and phrases have necessarily been included. Specific terms are defined in Module 20, Master Glossary. Considerable emphasis has been placed on illustrations to provide a maximum amount of information. In some instances, a knowledge of basic algebra may be required. Assignments are provided for each module, with the exceptions of Module 19, The Technician's Handbook ; and Module 20, Master Glossary. Course descriptions and ordering information are in NAVEDTRA 12061, Catalog of Nonresident Training Courses. v Throughout the text of this course and while using technical manuals associated with the equipment you will be working on, you will find the below notations at the end of some paragraphs. The notations are used to emphasize that safety hazards exist and care must be taken or observed. WARNING AN OPERATING PROCEDURE, PRACTICE, OR CONDITION, ETC., WHICH MAY RESULT IN INJURY OR DEATH IF NOT CAREFULLY OBSERVED OR FOLLOWED. CAUTION AN OPERATING PROCEDURE, PRACTICE, OR CONDITION, ETC., WHICH MAY RESULT IN DAMAGE TO EQUIPMENT IF NOT CAREFULLY OBSERVED OR FOLLOWED. NOTE An operating procedure, practice, or condition, etc., which is essential to emphasize. vi INSTRUCTIONS FOR TAKING THE COURSE ASSIGNMENTS The text pages that you are to study are listed at the beginning of each assignment. Study these pages carefully before attempting to answer the questions. Pay close attention to tables and illustrations and read the learning objectives. The learning objectives state what you should be able to do after studying the material. Answering the questions correctly helps you accomplish the objectives. SELECTING YOUR ANSWERS Read each question carefully, then select the BEST answer. You may refer freely to the text. The answers must be the result of your own work and decisions. You are prohibited from referring to or copying the answers of others and from giving answers to anyone else taking the course. SUBMITTING YOUR ASSIGNMENTS To have your assignments graded, you must be enrolled in the course with the Nonresident Training Course Administration Branch at the Naval Education and Training Professional Development and Technology Center (NETPDTC). Following enrollment, there are two ways of having your assignments graded: (1) use the Internet to submit your assignments as you complete them, or (2) send all the assignments at one time by mail to NETPDTC. Grading on the Internet: Advantages to Internet grading are: • you may submit your answers as soon as you complete an assignment, and • you get your results faster; usually by the next working day (approximately 24 hours). In addition to receiving grade results for each assignment, you will receive course completion confirmation once you have completed all the assignments. To submit your assignment answers via the Internet, go to: http://courses.cnet.navy.mil Grading by Mail: When you submit answer sheets by mail, send all of your assignments at one time. Do NOT submit individual answer sheets for grading. Mail all of your assignments in an envelope, which you either provide yourself or obtain from your nearest Educational Services Officer (ESO). Submit answer sheets to: COMMANDING OFFICER NETPDTC N331 6490 SAUFLEY FIELD ROAD PENSACOLA FL 32559-5000 Answer Sheets: All courses include one “scannable” answer sheet for each assignment. These answer sheets are preprinted with your SSN, name, assignment number, and course number. Explanations for completing the answer sheets are on the answer sheet. Do not use answer sheet reproductions: Use only the original answer sheets that we provide — reproductions will not work with our scanning equipment and cannot be processed. Follow the instructions for marking your answers on the answer sheet. Be sure that blocks 1, 2, and 3 are filled in correctly. This information is necessary for your course to be properly processed and for you to receive credit for your work. COMPLETION TIME Courses must be completed within 12 months from the date of enrollment. This includes time required to resubmit failed assignments. Vll PASS/FAIL ASSIGNMENT PROCEDURES If your overall course score is 3.2 or higher, you will pass the course and will not be required to resubmit assignments. Once your assignments have been graded you will receive course completion confirmation. If you receive less than a 3.2 on any assignment and your overall course score is below 3.2, you will be given the opportunity to resubmit failed assignments. You may resubmit failed assignments only once. Internet students will receive notification when they have failed an assignment— they may then resubmit failed assignments on the web site. Internet students may view and print results for failed assignments from the web site. Students who submit by mail will receive a failing result letter and a new answer sheet for resubmission of each failed assignment. COMPLETION CONFIRMATION After successfully completing this course, you will receive a letter of completion. ERRATA Errata are used to correct minor errors or delete obsolete information in a course. Errata may also be used to provide instructions to the student. If a course has an errata, it will be included as the first page(s) after the front cover. Errata for all courses can be accessed and viewed/downloaded at: http://www.advancement.cnet.navy.mil STUDENT FEEDBACK QUESTIONS For subject matter questions: E-mail: n3 1 5 .products @ cnet.navy.mil Phone: Comm: (850) 452-1001, ext. 1728 DSN: 922-1001, ext. 1728 FAX: (850)452-1370 (Do not fax answer sheets.) Address: COMMANDING OFFICER NETPDTC N315 6490 SAUFLEY FIELD ROAD PENSACOLA FL 32509-5237 For enrollment, shipping, grading, or completion letter questions E-mail: fleetservices@cnet.navy.mil Phone: Toll Free: 877-264-8583 Comm: (850)452-1511/1181/1859 DSN: 922-1511/1181/1859 FAX: (850)452-1370 (Do not fax answer sheets.) Address: COMMANDING OFFICER NETPDTC N331 6490 SAUFLEY FIELD ROAD PENSACOLA FL 32559-5000 NAVAL RESERVE RETIREMENT CREDIT If you are a member of the Naval Reserve, you will receive retirement points if you are authorized to receive them under current directives governing retirement of Naval Reserve personnel. For Naval Reserve retirement, this course is evaluated at 6 points. (Refer to Administrative Procedures for Naval Reservists on Inactive Duty , BUPERSINST 1001.39, for more information about retirement points.) We value your suggestions, questions, and criticisms on our courses. If you would like to communicate with us regarding this course, we encourage you, if possible, to use e-mail. If you write or fax, please use a copy of the Student Comment form that follows this page. vm Student Comments MEETS Module 1 Course Title: Introduction to Matter, Energy, and Direct Current NAVEDTRA: 14173 Date: We need some information about you : Rate/Rank and Name: SSN: Command/Unit Street Address: City: State/FPO: Zip Your comments, suggestions, etc .: Privacy Act Statement: Under authority of Title 5, USC 301, information regarding your military status is requested in processing your comments and in preparing a reply. This information will not be divulged without written authorization to anyone other than those within POD for official use in determining performance. NETPDTC 1550/41 (Rev 4-00) IX CHAPTER 1 MATTER, ENERGY, AND ELECTRICITY LEARNING OBJECTIVES Learning objectives are stated at the beginning of each chapter. These learning objectives serve as a preview of the information you are expected to learn in the chapter. The comprehensive check questions are based on the objectives. By successfully completing the NRTC, you indicate that you have met the objectives and have learned the information. The learning objectives are listed below. Upon completing this chapter, you will be able to: 1. State the meanings of and the relationship between matter, element, nucleus, compound, molecule, mixture, atom, electron, proton, neutron, energy, valence, valence shell, and ion. 2. State the meanings of and the relationship between kinetic energy, potential energy, photons, electron orbits, energy levels, and shells and subshells. 3. State, in terms of valence, the differences between a conductor, an insulator, and a semiconductor, and list some materials which make the best conductors and insulators. 4. State the definition of static electricity and explain how static electricity is generated. 5. State the meanings of retentivity, reluctance, permeability, ferromagnetism, natural magnet, and artificial magnet as used to describe magnetic materials. 6. State the Weber and domain theories of magnetism and list six characteristics of magnetic lines of force (magnetic flux), including their relation to magnetic induction, shielding, shape, and storage. 7. State, using the water analogy, how a difference of potential (a voltage or an electromotive force) can exist. Convert volts to microvolts, to millivolts, and to kilovolts. 8. List six methods for producing a voltage (emf) and state the operating principles of and the uses for each method. 9. State the meanings of electron current, random drift, directed drift, and ampere, and indicate the direction that an electric current flows. 10. State the relationship of current to voltage and convert amperes to milliamperes and microamperes. 11. State the definitions of and the terms and symbols for resistance and conductance, and how the temperature, contents, length and cross-sectional area of a conductor affect its resistance and conductance values. 12. List the physical and operating characteristics of and the symbols, ratings, and uses for various types of resistors; use the color code to identify resistor values. 1-1 INTRODUCTION The origin of the modern technical and electronic Navy stretches back to the beginning of naval history, when the first navies were no more than small fleets of wooden ships, using wind-filled sails and manned oars. The need for technicians then was restricted to a navigator and semiskilled seamen who could handle the sails. As time passed, larger ships that carried more sail were built. These ships, encouraging exploration and commerce, helped to establish world trade routes. Soon strong navies were needed to guard these sea lanes. Countries established their own navies to protect their citizens, commercial ships, and shipping lanes against pirates and warring nations. With the addition of mounted armament, gunners joined the ship’s company of skilled or semiskilled technicians. The advent of the steam engine signaled the rise of an energy source more practical than either wind and sails or manpower. With this technological advancement, the need for competent operators and technicians increased. However, the big call for operators and technicians in the U.S. Navy came in the early part of the 20th century, when power sources, means of communication, modes of detection, and armaments moved with amazing rapidity toward involved technical development. Electric motors and generators by then had become the most widely used sources of power. Telephone systems were well established on board ship, and radio was being used more and more to relay messages from ship to ship and from ship to shore. Listening devices were employed to detect submarines. Complex optical systems were used to aim large naval rifles. Mines and torpedoes became highly developed, effective weapons, and airplanes joined the Navy team. During the years after World War I, the Navy became more electricity and electronic minded. It was recognized that a better system of communications was needed aboard each ship, and between the ships, planes, submarines, and shore installations; and that weaponry advances were needed to keep pace with worldwide developments in that field. This growing technology carried with it the awareness that an equally skilled force of technicians was needed for maintenance and service duties. World War II proved that all of the expense of providing equipment for the fleet and of training personnel to handle that equipment paid great dividends. The U. S. Navy had the modern equipment and highly trained personnel needed to defeat the powerful fleets of the enemy. Today there is scarcely anyone on board a Navy ship who does not use electrical or electronic equipment. This equipment is needed in systems of electric lighting and power, intercommunications, radio, radar, sonar, loran, remote metering, weapon aiming, and certain types of mines and torpedoes. The Navy needs trained operators and technicians in this challenging field of electronics and electricity. It is to achieve this end that this module, and others like it, are published. MATTER, ENERGY, AND ELECTRICITY If there are roots to western science, they no doubt lie under the rubble that was once ancient Greece. With the exception of the Greeks, ancient people had little interest in the structure of materials. They accepted a solid as being just that a continuous, uninterrupted substance. One Greek school of thought believed that if a piece of matter, such as copper, were subdivided, it could be done indefinitely and still only that material would be found. Others reasoned that there must be a limit to the number of subdivisions that could be made and have the material still retain its original characteristics. They held fast to the idea that there must be a basic particle upon which all substances are built. Recent experiments have revealed that there are, indeed, several basic particles, or building blocks within all substances. 1-2 The following paragraphs explain how substances are classified as elements and compounds, and are made up of molecules and atoms. This, then, will be a learning experience about protons, electrons, valence, energy levels, and the physics of electricity. MATTER Matter is defined as anything that occupies space and has weight; that is, the weight and dimensions of matter can be measured. Examples of matter are air, water, automobiles, clothing, and even our own bodies. Thus, we can say that matter may be found in any one of three states: SOLID, LIQUID, and GASEOUS. ELEMENTS AND COMPOUNDS An ELEMENT is a substance which cannot be reduced to a simpler substance by chemical means. Examples of elements with which you are in everyday contact are iron, gold, silver, copper, and oxygen. There are now over 100 known elements. All the different substances we know about are composed of one or more of these elements. When two or more elements are chemically combined, the resulting substance is called a COMPOUND. A compound is a chemical combination of elements which can be separated by chemical but not by physical means. Examples of common compounds are water which consists of hydrogen and oxygen, and table salt, which consists of sodium and chlorine. A MIXTURE, on the other hand, is a combination of elements and compounds, not chemically combined, that can be separated by physical means. Examples of mixtures are air, which is made up of nitrogen, oxygen, carbon dioxide, and small amounts of several rare gases, and sea water, which consists chiefly of salt and water. Ql. What is matter , and in what three states is it found? Q2. What is an element? Q3. What is a compound? Q4. What is the difference between a compound and a mixture? MOLECULES A MOLECULE is a chemical combination of two or more atoms, (atoms are described in the next paragraph). In a compound the molecule is the smallest particle that has all the characteristics of the compound. Consider water, for example. Water is matter, since it occupies space and has weight. Depending on the temperature, it may exist as a liquid (water), a solid (ice), or a gas (steam). Regardless of the temperature, it will still have the same composition. If we start with a quantity of water, divide this and pour out one half, and continue this process a sufficient number of times, we will eventually end up with a quantity of water which cannot be further divided without ceasing to be water. This quantity is called a molecule of water. If this molecule of water divided, instead of two parts of water, there will be one part of oxygen and two parts of hydrogen (H 2 0). ATOMS Molecules are made up of smaller particles called ATOMS. An atom is the smallest particle of an element that retains the characteristics of that element. The atoms of one element, however, differ from 1-3 the atoms of all other elements. Since there are over 100 known elements, there must be over 100 different atoms, or a different atom for each element. Just as thousands of words can be made by combining the proper letters of the alphabet, so thousands of different materials can be made by chemically combining the proper atoms. Any particle that is a chemical combination of two or more atoms is called a molecule. The oxygen molecule consists of two atoms of oxygen, and the hydrogen molecule consists of two atoms of hydrogen. Sugar, on the other hand, is a compound composed of atoms of carbon, hydrogen, and oxygen. These atoms are combined into sugar molecules. Since the sugar molecules can be broken down by chemical means into smaller and simpler units, we cannot have sugar atoms. The atoms of each element are made up of electrons , protons , and, in most cases, neutrons , which are collectively called subatomic particles. Furthermore, the electrons, protons, and neutrons of one element are identical to those of any other element. The reason that there are different kinds of elements is that the number and the arrangement of electrons and protons within the atom are different for the different elements The electron is considered to be a small negative charge of electricity. The proton has a positive charge of electricity equal and opposite to the charge of the electron. Scientists have measured the mass and size of the electron and proton, and they know how much charge each possesses. The electron and proton each have the same quantity of charge, although the mass of the proton is approximately 1837 times that of the electron. In some atoms there exists a neutral particle called a neutron. The neutron has a mass approximately equal to that of a proton, but it has no electrical charge . According to a popular theory, the electrons, protons, and neutrons of the atoms are thought to be arranged in a manner similar to a miniature solar system. The protons and neutrons form a heavy nucleus with a positive charge, around which the very light electrons revolve. Figure 1-1 shows one hydrogen and one helium atom. Each has a relatively simple structure. The hydrogen atom has only one proton in the nucleus with one electron rotating about it. The helium atom is a little more complex. It has a nucleus made up of two protons and two neutrons, with two electrons rotating about the nucleus. Elements are classified numerically according to the complexity of their atoms. The atomic number of an atom is determined by the number of protons in its nucleus. PROTONS ELECTRONS NUCLEUS HYDROGEN NEUTRON HELIUM • PROTONS H NEUTRONS Figure 1-1. — Structures of simple atoms. In a neutral state, an atom contains an equal number of protons and electrons. Therefore, an atom of hydrogen — which contains one proton and one electron — has an atomic number of 1 ; and helium, with 1-4 two protons and two electrons, has an atomic number of 2. The complexity of atomic structure increases with the number of protons and electrons. Q5. What is a molecule ? Q6. What are the three types of subatomic particles, and what are their charges? Energy Levels Since an electron in an atom has both mass and motion, it contains two types of energy. By virtue of its motion the electron contains KINETIC ENERGY. Due to its position it also contains POTENTIAL ENERGY. The total energy contained by an electron (kinetic plus potential) is the factor which determines the radius of the electron orbit. In order for an electron to remain in this orbit, it must neither GAIN nor LOSE energy. It is well known that light is a form of energy, but the physical form in which this energy exists is not known. One accepted theory proposes the existence of light as tiny packets of energy called PHOTONS. Photons can contain various quantities of energy. The amount depends upon the color of the light involved. Should a photon of sufficient energy collide with an orbital electron, the electron will absorb the photon’s energy, as shown in figure 1-2. The electron, which now has a greater than normal amount of energy, will jump to a new orbit farther from the nucleus. The first new orbit to which the electron can jump has a radius four times as large as the radius of the original orbit. Had the electron received a greater amount of energy, the next possible orbit to which it could jump would have a radius nine times the original. Thus, each orbit may be considered to represent one of a large number of energy levels that the electron may attain. It must be emphasized that the electron cannot jump to just any orbit. The electron will remain in its lowest orbit until a sufficient amount of energy is available, at which time the electron will accept the energy and jump to one of a series of permissible orbits. An electron cannot exist in the space between energy levels. This indicates that the electron will not accept a photon of energy unless it contains enough energy to elevate itself to one of the higher energy levels. Heat energy and collisions with other particles can also cause the electron to jump orbits. Figure 1-2. — Excitation by a photon. 1-5 Once the electron has been elevated to an energy level higher than the lowest possible energy level, the atom is said to be in an excited state. The electron will not remain in this excited condition for more than a fraction of a second before it will radiate the excess energy and return to a lower energy orbit. To illustrate this principle, assume that a normal electron has just received a photon of energy sufficient to raise it from the first to the third energy level. In a short period of time the electron may jump back to the first level emitting a new photon identical to the one it received. A second alternative would be for the electron to return to the lower level in two jumps; from the third to the second, and then from the second to the first. In this case the electron would emit two photons, one for each jump. Each of these photons would have less energy than the original photon which excited the electron. This principle is used in the fluorescent light where ultraviolet light photons, which are not visible to the human eye, bombard a phosphor coating on the inside of a glass tube. The phosphor electrons, in returning to their normal orbits, emit photons of light that are visible. By using the proper chemicals for the phosphor coating, any color of light may be obtained, including white. This same principle is also used in lighting up the screen of a television picture tube. The basic principles just developed apply equally well to the atoms of more complex elements. In atoms containing two or more electrons, the electrons interact with each other and the exact path of any one electron is very difficult to predict. However, each electron lies in a specific energy band and the orbits will be considered as an average of the electron’s position. <27. What is energy of motion called? Q8. How is invisible light changed to visible light in a fluorescent light? Shells and Subshells The difference between the atoms, insofar as their chemical activity and stability are concerned, is dependent upon the number and position of the electrons included within the atom. How are these electrons positioned within the atom? In general, the electrons reside in groups of orbits called shells. These shells are elliptically shaped and are assumed to be located at fixed intervals. Thus, the shells are arranged in steps that correspond to fixed energy levels. The shells, and the number of electrons required to fill them, may be predicted by the employment of Pauli’s exclusion principle. Simply stated, this 9 principle specifies that each shell will contain a maximum of 2n electrons, where n corresponds to the shell number starting with the one closest to the nucleus. By this principle, the second shell, for example, 2 would contain 2(2) or 8 electrons when full. In addition to being numbered, the shells are also given letter designations, as pictured in figure 1-3. Starting with the shell closest to the nucleus and progressing outward, the shells are labeled K, L, M, N, O, P, and Q, respectively. The shells are considered to be full, or complete, when they contain the following quantities of electrons: two in the K shell, eight in the L shell, 18 in the M shell, and so on, in accordance with the exclusion principle. Each of these shells is a major shell and can be divided into subshells, of which there are four, labeled s, p, d, and f. Like the major shells, the subshells are also limited as to the number of electrons which they can contain. Thus, the "s" subshell is complete when it contains two electrons, the "p" subshell when it contains 10, and the "f" subshell when it contains 14 electrons. 1-6 Figure 1-3. — Shell designation. Inasmuch as the K shell can contain no more than two electrons, it must have only one subshell, the s subshell. The M shell is composed of three subshells: s, p, and d. If the electrons in the s, p, and d subshells are added, their total is found to be 18, the exact number required to fill the M shell. Notice the electron configuration for copper illustrated in figure 1-4. The copper atom contains 29 electrons, which completely fill the first three shells and subshells, leaving one electron in the "s" subshell of the N shell. Figure 1-4. — Copper atom. Valence The number of electrons in the outermost shell determines the valence of an atom. For this reason, the outer shell of an atom is called the VALENCE SHELL; and the electrons contained in this shell are called VALENCE ELECTRONS. The valence of an atom determines its ability to gain or lose an electron, which in turn determines the chemical and electrical properties of the atom. An atom that is 1-7 lacking only one or two electrons from its outer shell will easily gain electrons to complete its shell, but a large amount of energy is required to free any of its electrons. An atom having a relatively small number of electrons in its outer shell in comparison to the number of electrons required to fill the shell will easily lose these valence electrons. The valence shell always refers to the outermost shell. Q9. What determines the valence of an atom? Ionization When the atom loses electrons or gains electrons in this process of electron exchange, it is said to be IONIZED. For ionization to take place, there must be a transfer of energy which results in a change in the internal energy of the atom. An atom having more than its normal amount of electrons acquires a negative charge, and is called a NEGATIVE ION. The atom that gives up some of its normal electrons is left with less negative charges than positive charges and is called a POSITIVE ION. Thus, ionization is the process by which an atom loses or gains electrons. 010. What is an ion? CONDUCTORS, SEMICONDUCTORS, AND INSULATORS In this study of electricity and electronics, the association of matter and electricity is important. Since every electronic device is constructed of parts made from ordinary matter, the effects of electricity on matter must be well understood. As a means of accomplishing this, all elements of which matter is made may be placed into one of three categories: CONDUCTORS, SEMICONDUCTORS, and INSULATORS, depending on their ability to conduct an electric current. CONDUCTORS are elements which conduct electricity very readily, INSULATORS have an extremely high resistance to the flow of electricity. All matter between these two extremes may be called SEMICONDUCTORS. The electron theory states that all matter is composed of atoms and the atoms are composed of smaller particles called protons, electrons, and neutrons. The electrons orbit the nucleus which contains the protons and neutrons. It is the valence electrons that we are most concerned with in electricity. These are the electrons which are easiest to break loose from their parent atom. Normally, conductors have three or less valence electrons; insulators have five or more valence electrons; and semiconductors usually have four valence electrons. The electrical conductivity of matter is dependent upon the atomic structure of the material from which the conductor is made. In any solid material, such as copper, the atoms which make up the molecular structure are bound firmly together. At room temperature, copper will contain a considerable amount of heat energy. Since heat energy is one method of removing electrons from their orbits, copper will contain many free electrons that can move from atom to atom. When not under the influence of an external force, these electrons move in a haphazard manner within the conductor. This movement is equal in all directions so that electrons are not lost or gained by any part of the conductor. When controlled by an external force, the electrons move generally in the same direction. The effect of this movement is felt almost instantly from one end of the conductor to the other. This electron movement is called an ELECTRIC CURRENT. Some metals are better conductors of electricity than others. Silver, copper, gold, and aluminum are materials with many free electrons and make good conductors. Silver is the best conductor, followed by copper, gold, and aluminum. Copper is used more often than silver because of cost. Aluminum is used where weight is a major consideration, such as in high-tension power lines, with long spans between supports. Gold is used where oxidation or corrosion is a consideration and a good conductivity is 1-8 required. The ability of a conductor to handle current also depends upon its physical dimensions. Conductors are usually found in the form of wire, but may be in the form of bars, tubes, or sheets. Nonconductors have few free electrons. These materials are called INSULATORS. Some examples of these materials are rubber, plastic, enamel, glass, dry wood, and mica. Just as there is no perfect conductor, neither is there a perfect insulator. Some materials are neither good conductors nor good insulators, since their electrical characteristics fall between those of conductors and insulators. These in-between materials are classified as SEMICONDUCTORS. Germanium and silicon are two common semiconductors used in solid-state devices. OIL What determines whether a substance is a conductor or an insulator? ELECTROSTATICS Electrostatics (electricity at rest) is a subject with which most persons entering the field of electricity and electronics are somewhat familiar. For example, the way a person’s hair stands on end after a vigorous rubbing is an effect of electrostatics. While pursuing the study of electrostatics, you will gain a better understanding of this common occurrence. Of even greater significance, the study of electrostatics will provide you with the opportunity to gain important background knowledge and to develop concepts which are essential to the understanding of electricity and electronics. Interest in the subject of static electricity can be traced back to the Greeks. Thales of Miletus, a Greek philosopher and mathematician, discovered that when an amber rod is rubbed with fur, the rod has the amazing characteristic of attracting some very light objects such as bits of paper and shavings of wood. About 1600, William Gilbert, an English scientist, made a study of other substances which had been found to possess qualities of attraction similar to amber. Among these were glass, when rubbed with silk, and ebonite, when rubbed with fur. Gilbert classified all the substances which possessed properties similar to those of amber as electrics, a word of Greek origin meaning amber. Because of Gilbert’s work with electrics, a substance such as amber or glass when given a vigorous rubbing was recognized as being ELECTRIFIED, or CHARGED with electricity. In the year 1733, Charles Dufay, a French scientist, made an important discovery about electrification. He found that when a glass was rubbed with fur, both the glass rod and the fur became electrified. This realization came when he systematically placed the glass rod and the fur near other electrified substances and found that certain substances which were attracted to the glass rod were repelled by the fur, and vice versa. From experiments such as this, he concluded that there must be two exactly opposite kinds of electricity. Benjamin Franklin, American statesman, inventor, and philosopher, is credited with first using the terms POSITIVE and NEGATIVE to describe the two opposite kinds of electricity. The charge produced on a glass rod when it is rubbed with silk, Franklin labeled positive . He attached the term negative to the charge produced on the silk. Those bodies which were not electrified or charged, he called NEUTRAL. STATIC ELECTRICITY In a natural , or neutral state, each atom in a body of matter will have the proper number of electrons in orbit around it. Consequently, the whole body of matter composed of the neutral atoms will also be 1-9 electrically neutral. In this state, it is said to have a "zero charge." Electrons will neither leave nor enter the neutrally charged body should it come in contact with other neutral bodies. If, however, any number of electrons are removed from the atoms of a body of matter, there will remain more protons than electrons and the whole body of matter will become ELECTRICALLY POSITIVE. Should the positively charged body come in contact with another body having a normal charge, or having a NEGATIVE (too many electrons) charge, an electric current will flow between them. Electrons will leave the more negative body and enter the positive body. This electron flow will continue until both bodies have equal charges. When two bodies of matter have unequal charges and are near one another, an electric force is exerted between them because of their unequal charges. However, since they are not in contact, their charges cannot equalize. The existence of such an electric force, where current cannot flow, is referred to as static electricity. ("Static" in this instance means "not moving.") It is also referred to as an electrostatic force. One of the easiest ways to create a static charge is by friction. When two pieces of matter are rubbed together, electrons can be "wiped off" one material onto the other. If the materials used are good conductors , it is quite difficult to obtain a detectable charge on either, since equalizing currents can flow easily between the conducting materials. These currents equalize the charges almost as fast as they are created. A static charge is more easily created between nonconducting materials. When a hard rubber rod is rubbed with fur, the rod will accumulate electrons given up by the fur, as shown in figure 1-5. Since both materials are poor conductors, very little equalizing current can flow, and an electrostatic charge builds up. When the charge becomes great enough, current will flow regardless of the poor conductivity of the materials. These currents will cause visible sparks and produce a crackling sound. +CHARGES AND ELECTRONS ARE PRESENT IN EQUAL QUANTITIES IN THE ROD AND FUR HARD RUBBER ROD ELECTRONS TRANSFERRED FROM THE FUR TO THE ROD Figure 1-5. — Producing static electricity by friction. Q12. How is a negative charge created in a neutral body? Q13. How are static charges created? Nature of Charges When in a natural, or neutral state, an atom has an equal number of electrons and protons. Because of this balance, the net negative charge of the electrons in orbit is exactly balanced by the net positive charge of the protons in the nucleus, making the atom electrically neutral. 1-10 An atom becomes a positive ion whenever it loses an electron, and has an overall positive charge. Conversely, whenever an atom acquires an extra electron, it becomes a negative ion and has a negative charge. Due to normal molecular activity, there are always ions present in any material. If the number of positive ions and negative ions is equal, the material is electrically neutral. When the number of positive ions exceeds the number of negative ions, the material is positively charged. The material is negatively charged whenever the negative ions outnumber the positive ions. Since ions are actually atoms without their normal number of electrons, it is the excess or the lack of electrons in a substance that determines its charge. In most solids, the transfer of charges is by movement of electrons rather than ions. The transfer of charges by ions will become more significant when we consider electrical activity in liquids and gases. At this time, we will discuss electrical behavior in terms of electron movement. Q14. What is the electrical charge of an atom which contains 8 protons and 11 electrons? Charged Bodies One of the fundamental laws of electricity is that LIKE CHARGES REPEL EACH OTHER and UNLIKE CHARGES ATTRACT EACH OTHER. A positive charge and negative charge, being unlike, tend to move toward each other. In the atom, the negative electrons are drawn toward the positive protons in the nucleus. This attractive force is balanced by the electron’s centrifugal force caused by its rotation about the nucleus. As a result, the electrons remain in orbit and are not drawn into the nucleus. Electrons repel each other because of their like negative charges, and protons repel each other because of their like positive charges. The law of charged bodies may be demonstrated by a simple experiment. Two pith (paper pulp) balls are suspended near one another by threads, as shown in figure 1-6. (A) LIKE CHARGES REPEL + Figure 1-6. — Reaction between charged bodies. 1-11 If a hard rubber rod is rubbed with fur to give it a negative charge and is then held against the right- hand ball in part (A), the rod will give off a negative charge to the ball. The right-hand ball will have a negative charge with respect to the left-hand ball. When released, the two balls will be drawn together, as shown in figure 1-6(A). They will touch and remain in contact until the left-hand ball gains a portion of the negative charge of the right-hand ball, at which time they will swing apart as shown in figure 1-6(C). If a positive or a negative charge is placed on both balls (fig. 1-6(B)), the balls will repel each other. Coulomb’s Law of Charges The relationship between attracting or repelling charged bodies was first discovered and written about by a French scientist named Charles A. Coulomb. Coulomb’s Law states that CHARGED BODIES ATTRACT OR REPEL EACH OTHER WITH A FORCE THAT IS DIRECTLY PROPORTIONAL TO THE PRODUCT OF THEIR INDIVIDUAL CHARGES, AND IS INVERSELY PROPORTIONAL TO THE SQUARE OF THE DISTANCE BETWEEN THEM. The amount of attracting or repelling force which acts between two electrically charged bodies in free space depends on two things — (1) their charges and (2) the distance between them. Electric Fields The space between and around charged bodies in which their influence is felt is called an ELECTRIC FIELD OF FORCE. It can exist in air, glass, paper, or a vacuum. ELECTROSTATIC FIELDS and DIELECTRIC FIELDS are other names used to refer to this region of force. Fields of force spread out in the space surrounding their point of origin and, in general, DIMINISH IN PROPORTION TO THE SQUARE OF THE DISTANCE FROM THEIR SOURCE. The field about a charged body is generally represented by lines which are referred to as ELECTROSTATIC LINES OF FORCE. These lines are imaginary and are used merely to represent the direction and strength of the field. To avoid confusion, the lines of force exerted by a positive charge are always shown leaving the charge, and for a negative charge they are shown entering . Figure 1-7 illustrates the use of lines to represent the field about charged bodies. 1-12 (B) Figure 1-7. — Electrostatic lines of force. Figure 1-7(A) represents the repulsion of like-charged bodies and their associated fields. Part (B) represents the attraction of unlike-charged bodies and their associated fields. Q15. What is the relationship between charged bodies? Q16. What is an electrostatic field? Q17. In what direction are electrostatic lines of force drawn? MAGNETISM In order to properly understand the principles of electricity, it is necessary to study magnetism and the effects of magnetism on electrical equipment. Magnetism and electricity are so closely related that the study of either subject would be incomplete without at least a basic knowledge of the other. Much of today’s modern electrical and electronic equipment could not function without magnetism. Modern computers, tape recorders, and video reproduction equipment use magnetized tape. High-fidelity speakers use magnets to convert amplifier outputs into audible sound. Electrical motors use magnets to convert electrical energy into mechanical motion; generators use magnets to convert mechanical motion into electrical energy. Q18. What are some examples of electrical equipment which use magnetism? MAGNETIC MATERIALS Magnetism is generally defined as that property of a material which enables it to attract pieces of iron. A material possessing this property is known as a MAGNET. The word originated with the ancient Greeks, who found stones possessing this characteristic. Materials that are attracted by a magnet, such as iron, steel, nickel, and cobalt, have the ability to become magnetized. These are called magnetic materials. 1-13 Materials, such as paper, wood, glass, or tin, which are not attracted by magnets, are considered nonmagnetic. Nonmagnetic materials are not able to become magnetized. Q19. What are magnetic materials? Ferromagnetic Materials The most important group of materials connected with electricity and electronics are the ferromagnetic materials. Ferromagnetic materials are those which are relatively easy to magnetize, such as iron, steel, cobalt, and the alloys Alnico and Permalloy. (An alloy is made from combining two or more elements, one of which must be a metal). These new alloys can be very strongly magnetized, and are capable of obtaining a magnetic strength great enough to lift 500 times their own weight. Natural Magnets Magnetic stones such as those found by the ancient Greeks are considered to be NATURAL MAGNETS. These stones had the ability to attract small pieces of iron in a manner similar to the magnets which are common today. However, the magnetic properties attributed to the stones were products of nature and not the result of the efforts of man. The Greeks called these substances magnetite. The Chinese are said to have been aware of some of the effects of magnetism as early as 2600 B.C. They observed that stones similar to magnetite, when freely suspended, had a tendency to assume a nearly north and south direction. Because of the directional quality of these stones, they were later referred to as lodestones or leading stones. Natural magnets, which presently can be found in the United States, Norway, and Sweden, no longer have any practical use, for it is now possible to easily produce more powerful magnets. Q20. What characteristics do all ferromagnetic materials have in common? Artificial Magnets Magnets produced from magnetic materials are called ARTIFICIAL MAGNETS. They can be made in a variety of shapes and sizes and are used extensively in electrical apparatus. Artificial magnets are generally made from special iron or steel alloys which are usually magnetized electrically. The material to be magnetized is inserted into a coil of insulated wire and a heavy flow of electrons is passed through the wire. Magnets can also be produced by stroking a magnetic material with magnetite or with another artificial magnet. The forces causing magnetization are represented by magnetic lines of force, very similar in nature to electrostatic lines of force. Artificial magnets are usually classified as PERMANENT or TEMPORARY, depending on their ability to retain their magnetic properties after the magnetizing force has been removed. Magnets made from substances, such as hardened steel and certain alloys which retain a great deal of their magnetism, are called PERMANENT MAGNETS. These materials are relatively difficult to magnetize because of the opposition offered to the magnetic lines of force as the lines of force try to distribute themselves throughout the material. The opposition that a material offers to the magnetic lines of force is called RELUCTANCE. All permanent magnets are produced from materials having a high reluctance. A material with a low reluctance, such as soft iron or annealed silicon steel, is relatively easy to magnetize but will retain only a small part of its magnetism once the magnetizing force is removed. Materials of this type that easily lose most of their magnetic strength are called TEMPORARY MAGNETS. The amount of magnetism which remains in a temporary magnet is referred to as its 1-14 RESIDUAL MAGNETISM. The ability of a material to retain an amount of residual magnetism is called the RETENTIVITY of the material. The difference between a permanent and a temporary magnet has been indicated in terms of RELUCTANCE, a permanent magnet having a high reluctance and a temporary magnet having a low reluctance. Magnets are also described in terms of the PERMEABILITY of their materials, or the ease with which magnetic lines of force distribute themselves throughout the material. A permanent magnet, which is produced from a material with a high reluctance, has a low permeability. A temporary magnet, produced from a material with a low reluctance, would have a high permeability. Q21. What type of magnetic material should be used to make a temporary magnet? Q22. What is retentivity? MAGNETIC POLES The magnetic force surrounding a magnet is not uniform. There exists a great concentration of force at each end of the magnet and a very weak force at the center. Proof of this fact can be obtained by dipping a magnet into iron filings (fig. 1-8). It is found that many filings will cling to the ends of the magnet while very few adhere to the center. The two ends, which are the regions of concentrated lines of force, are called the POLES of the magnet. Magnets have two magnetic poles and both poles have equal magnetic strength. \ Figure 1-8. — Iron filings cling to the poles of a magnet. Law of Magnetic Poles If a bar magnet is suspended freely on a string, as shown in figure 1-9, it will align itself in a north and south direction. When this experiment is repeated, it is found that the same pole of the magnet will always swing toward the north magnetic pole of the earth. Therefore, it is called the north-seeking pole or simply the NORTH POLE. The other pole of the magnet is the south-seeking pole or the SOUTH POLE. 1-15 y Figure 1-9. — A bar magnet acts as a compass. A practical use of the directional characteristic of the magnet is the compass, a device in which a freely rotating magnetized needle indicator points toward the North Pole. The realization that the poles of a suspended magnet always move to a definite position gives an indication that the opposite poles of a magnet have opposite magnetic polarity. The law previously stated regarding the attraction and repulsion of charged bodies may also be applied to magnetism if the pole is considered as a charge. The north pole of a magnet will always be attracted to the south pole of another magnet and will show a repulsion to a north pole. The law for magnetic poles is: Like poles repel, unlike poles attract. Q23. How does the law of magnetic poles relate to the law of electric charges? The Earth’s Magnetic Poles The fact that a compass needle always aligns itself in a particular direction, regardless of its location on earth, indicates that the earth is a huge natural magnet. The distribution of the magnetic force about the earth is the same as that which might be produced by a giant bar magnet running through the center of the earth (fig. 1-10). The magnetic axis of the earth is located about 15° from its geographical axis thereby locating the magnetic poles some distance from the geographical poles. The ability of the north pole of the compass needle to point toward the north geographical pole is due to the presence of the magnetic pole nearby. This magnetic pole is named the magnetic North Pole. However, in actuality, it must have the polarity of a south magnetic pole since it attracts the north pole of a compass needle. The reason for this conflict in terminology can be traced to the early users of the compass. Knowing little about magnetic effects, they called the end of the compass needle that pointed towards the north geographical pole, the north pole of a compass. With our present knowledge of magnetism, we know the north pole of a compass needle (a small bar magnet) can be attracted only by an unlike magnetic pole, that is, a pole of south magnetic polarity. 1-16 MAGNETIC NORTH (SOUTHERN POLARITY) GEOGRAPHIC NORTH GEOGRAPHIC SOUTH MAGNETIC SOUTH (NORTHERN POLARHY) Figure 1-10. — The earth is a magnet. Q24. A compass is located at the geographical North Pole. In which direction would its needle point? THEORIES OF MAGNETISM Weber’s Theory A popular theory of magnetism considers the molecular alignment of the material. This is known as Weber’s theory. This theory assumes that all magnetic substances are composed of tiny molecular magnets. Any unmagnetized material has the magnetic forces of its molecular magnets neutralized by adjacent molecular magnets, thereby eliminating any magnetic effect. A magnetized material will have most of its molecular magnets lined up so that the north pole of each molecule points in one direction, and the south pole faces the opposite direction. A material with its molecules thus aligned will then have one effective north pole, and one effective south pole. An illustration of Weber’s Theory is shown in figure 1- 11, where a steel bar is magnetized by stroking. When a steel bar is stroked several times in the same direction by a magnet, the magnetic force from the north pole of the magnet causes the molecules to align themselves. 1-17 MAGNET MOTION BAR BEING MAGNETIZED EE EE EE EE EE EE EE EE EE EE EE LXJffiULJ » tMii l tmu th. lm^j BAR MAGNETIZED Figure 1-11. — Weber’s molecular theory of magnetism. Q25. Using Weber’s molecular theory of magnetism, describe the polarity of the magnetic poles produced by stroking a magnetic material from right to left with the south pole of a magnet. Domain Theory A more modern theory of magnetism is based on the electron spin principle. From the study of atomic structure it is known that all matter is composed of vast quantities of atoms, each atom containing one or more orbital electrons. The electrons are considered to orbit in various shells and subshells depending upon their distance from the nucleus. The structure of the atom has previously been compared to the solar system, wherein the electrons orbiting the nucleus correspond to the planets orbiting the sun. Along with its orbital motion about the sun, each planet also revolves on its axis. It is believed that the electron also revolves on its axis as it orbits the nucleus of an atom. It has been experimentally proven that an electron has a magnetic field about it along with an electric field. The effectiveness of the magnetic field of an atom is determined by the number of electrons spinning in each direction. If an atom has equal numbers of electrons spinning in opposite directions, the magnetic fields surrounding the electrons cancel one another, and the atom is unmagnetized. However, if more electrons spin in one direction than another, the atom is magnetized. An atom with an atomic number of 26, such as iron, has 26 protons in the nucleus and 26 revolving electrons orbiting its nucleus. If 13 electrons are spinning in a clockwise direction and 13 electrons are spinning in a counterclockwise direction, the opposing magnetic fields will be neutralized. When more than 13 electrons spin in either direction, the atom is magnetized. An example of a magnetized atom of iron is shown in figure 1-12. 1-18 NUMBER OF NUMBER OF ELECTRONS ELECTRONS SPINNING SPINNING Figure 1-12. — Iron atom. Q26. What is the difference between the domain theory and Weber’s theory of magnetism? MAGNETIC FIELDS The space surrounding a magnet where magnetic forces act is known as the magnetic field. A pattern of this directional force can be obtained by performing an experiment with iron filings. A piece of glass is placed over a bar magnet and the iron filings are then sprinkled on the surface of the glass. The magnetizing force of the magnet will be felt through the glass and each iron filing becomes a temporary magnet. If the glass is now tapped gently, the iron particles will align themselves with the magnetic field surrounding the magnet just as the compass needle did previously. The filings form a definite pattern, which is a visible representation of the forces comprising the magnetic field. Examination of the arrangements of iron filings in figure 1-13 will indicate that the magnetic field is very strong at the poles and weakens as the distance from the poles increases. It is also apparent that the magnetic field extends from one pole to the other, constituting a loop about the magnet. 1-19 IRON FILINGS GLASS PLATE OVER MAGNET Figure 1-13. — Pattern formed by iron filings. Q27. Refer to figure 1-13. For what purpose would you sprinkle iron filings on the glass plate? Q28. Refer to figure 1-13. What pattern would be formed if sawdust was sprinkled on the glass instead of iron filings? Lines of Force To further describe and work with magnet phenomena, lines are used to represent the force existing in the area surrounding a magnet (refer to fig. 1-14). These lines, called MAGNETIC LINES OF FORCE, do not actually exist but are imaginary lines used to illustrate and describe the pattern of the magnetic field. The magnetic lines of force are assumed to emanate from the north pole of a magnet, pass through surrounding space, and enter the south pole. The lines of force then travel inside the magnet from the south pole to the north pole, thus completing a closed loop. Figure 1-14. — Bar magnet showing lines of force. 1-20 When two magnetic poles are brought close together, the mutual attraction or repulsion of the poles produces a more complicated pattern than that of a single magnet. These magnetic lines of force can be plotted by placing a compass at various points throughout the magnetic field, or they can be roughly illustrated by the use of iron filings as before. A diagram of magnetic poles placed close together is shown in figure 1-15. UNLIKE POLES ATTRACT LIKE POLES REPEL Figure 1-15. — Magnetic poles in close proximity. Although magnetic lines of force are imaginary, a simplified version of many magnetic phenomena can be explained by assuming the magnetic lines to have certain real properties. The lines of force can be compared to rubber bands which stretch outward when a force is exerted upon them and contract when the force is removed. The characteristics of magnetic lines of force can be described as follows: 1. Magnetic lines of force are continuous and will always form closed loops. 2. Magnetic lines of force will never cross one another. 3. Parallel magnetic lines of force traveling in the same direction repel one another. Parallel magnetic lines of force traveling in opposite directions tend to unite with each other and form into single lines traveling in a direction determined by the magnetic poles creating the lines of force. 4. Magnetic lines of force tend to shorten themselves. Therefore, the magnetic lines of force existing between two unlike poles cause the poles to be pulled together. 5. Magnetic lines of force pass through all materials, both magnetic and nonmagnetic. 6. Magnetic lines of force always enter or leave a magnetic material at right angles to the surface. Q29. What is a magnetic line of force ? Q30. In what way do magnetic lines of force differ from electrostatic lines of force? 1-21 MAGNETIC EFFECTS MAGNETIC FLUX. The total number of magnetic lines of force leaving or entering the pole of a magnet is called MAGNETIC FLUX. The number of flux lines per unit area is known as FLUX DENSITY. FIELD INTENSITY. The intensity of a magnetic field is directly related to the magnetic force exerted by the field. ATTRACTION/REPULSION. The intensity of attraction or repulsion between magnetic poles may be described by a law almost identical to Coulomb’s Law of Charged Bodies. The force between two poles is directly proportional to the product of the pole strengths and inversely proportional to the square of the distance between the poles. Magnetic Induction It has been previously stated that all substances that are attracted by a magnet are capable of becoming magnetized. The fact that a material is attracted by a magnet indicates the material must itself be a magnet at the time of attraction. With the knowledge of magnetic fields and magnetic lines of force developed up to this point, it is simple to understand the manner in which a material becomes magnetized when brought near a magnet. As an iron nail is brought close to a bar magnet (fig. 1-16), some flux lines emanating from the north pole of the magnet pass through the iron nail in completing their magnetic path. Since magnetic lines of force travel inside a magnet from the south pole to the north pole, the nail will be magnetized in such a polarity that its south pole will be adjacent to the north pole of the bar magnet. There is now an attraction between the two magnets. UNLIKE POLES ATTRACT Figure 1-16. — Magnetized nail. If another nail is brought in contact with the end of the first nail, it would be magnetized by induction. This process could be repeated until the strength of the magnetic flux weakens as distance from 1-22 the bar magnet increases. However, as soon as the first iron nail is pulled away from the bar magnet, all the nails will fall. The reason being that each nail becomes a temporary magnet, and as soon as the magnetizing force is removed, their domains once again assume a random distribution. Magnetic induction will always produce a pole polarity on the material being magnetized opposite that of the adjacent pole of the magnetizing force. It is sometimes possible to bring a weak north pole of a magnet near a strong magnet north pole and note attraction between the poles. The weak magnet, when placed within the magnetic field of the strong magnet, has its magnetic polarity reversed by the field of the stronger magnet. Therefore, it is attracted to the opposite pole. For this reason, you must keep a very weak magnet, such as a compass needle, away from a strong magnet. Magnetism can be induced in a magnetic material by several means. The magnetic material may be placed in the magnetic field, brought into contact with a magnet, or stroked by a magnet. Stroking and contact both indicate actual contact with the material but are considered in magnetic studies as magnetizing by INDUCTION. Magnetic Shielding There is no known INSULATOR for magnetic flux. If a nonmagnetic material is placed in a magnetic field, there is no appreciable change in flux — that is, the flux penetrates the nonmagnetic material. For example, a glass plate placed between the poles of a horseshoe magnet will have no appreciable effect on the field although glass itself is a good insulator in an electric circuit. If a magnetic material (for example, soft iron) is placed in a magnetic field, the flux may be redirected to take advantage of the greater permeability of the magnetic material, as shown in figure 1-17. Permeability, as discussed earlier, is the quality of a substance which determines the ease with which it can be magnetized. Figure 1-17. — Effects of a magnetic substance in a magnetic field. The sensitive mechanisms of electric instruments and meters can be influenced by stray magnetic fields which will cause errors in their readings. Because instrument mechanisms cannot be insulated against magnetic flux, it is necessary to employ some means of directing the flux around the instrument. This is accomplished by placing a soft-iron case, called a MAGNETIC SCREEN or SHIELD, about the instrument. Because the flux is established more readily through the iron (even though the path is longer) than through the air inside the case, the instrument is effectively shielded, as shown by the watch and soft- iron shield in figure 1-18. 1-23 Figure 1-18. — Magnetic shield. MAGNETIC SHAPES Because of the many uses of magnets, they are found in various shapes and sizes. However, magnets usually come under one of three general classifications: bar magnets, horseshoe magnets, or ring magnets. The bar magnet is most often used in schools and laboratories for studying the properties and effects of magnetism. In the preceding material, the bar magnet proved very helpful in demonstrating magnetic effects. Another type of magnet is the ring magnet, which is used for computer memory cores. A common application for a temporary ring magnet would be the shielding of electrical instruments. The shape of the magnet most frequently used in electrical and electronic equipment is called the horseshoe magnet. A horseshoe magnet is similar to a bar magnet but is bent in the shape of a horseshoe. The horseshoe magnet provides much more magnetic strength than a bar magnet of the same size and material because of the closeness of the magnetic poles. The magnetic strength from one pole to the other is greatly increased due to the concentration of the magnetic field in a smaller area. Electrical measuring devices quite frequently use horseshoe-type magnets. CARE OF MAGNETS A piece of steel that has been magnetized can lose much of its magnetism by improper handling. If it is jarred or heated, there will be a disalignment of its domains resulting in the loss of some of its effective magnetism. Had this piece of steel formed the horseshoe magnet of a meter, the meter would no longer be 1-24 operable or would give inaccurate readings. Therefore, care must be exercised when handling instruments containing magnets. Severe jarring or subjecting the instrument to high temperatures will damage the device. A magnet may also become weakened from loss of flux. Thus when storing magnets, one should always try to avoid excess leakage of magnetic flux. A horseshoe magnet should always be stored with a keeper, a soft iron bar used to join the magnetic poles. By using the keeper while the magnet is being stored, the magnetic flux will continuously circulate through the magnet and not leak off into space. When bar magnets are stored, the same principle must be remembered. Therefore, bar magnets should always be stored in pairs with a north pole and a south pole placed together. This provides a complete path for the magnetic flux without any flux leakage. Q31. How should a delicate instrument be protected from a magnetic field? Q32. How should bar magnets be stored? ELECTRICAL ENERGY In the field of physical science, work must be defined as the PRODUCT OF FORCE AND DISPLACEMENT. That is, the force applied to move an object and the distance the object is moved are the factors of work performed. It is important to notice that no work is accomplished unless the force applied causes a change in the position of a stationary object, or a change in the velocity of a moving object. A worker may tire by pushing against a heavy wooden crate, but unless the crate moves, no work will be accomplished. ENERGY In our study of energy and work, we must define energy as THE ABILITY TO DO WORK. In order to perform any kind of work, energy must be expended (converted from one form to another). Energy supplies the required force, or power, whenever any work is accomplished. One form of energy is that which is contained by an object in motion. When a hammer is set in motion in the direction of a nail, it possesses energy of motion. As the hammer strikes the nail, the energy of motion is converted into work as the nail is driven into the wood. The distance the nail is driven into the wood depends on the velocity of the hammer at the time it strikes the nail. Energy contained by an object due to its motion is called KINETIC ENERGY. Assume that the hammer is suspended by a string in a position one meter above a nail. As a result of gravitational attraction, the hammer will experience a force pulling it downward. If the string is suddenly cut, the force of gravity will pull the hammer downward against the nail, driving it into the wood. While the hammer is suspended above the nail it has ability to do work because of its elevated position in the earth’s gravitational field. Since energy is the ability to do work, the hammer contains energy. Energy contained by an object due to its position is called POTENTIAL ENERGY. The amount of potential energy available is equal to the product of the force required to elevate the hammer and the height to which it is elevated. Another example of potential energy is that contained in a tightly coiled spring. The amount of energy released when the spring unwinds depends on the amount of force required to wind the spring initially. 1-25 Q33. What is the definition of energy? Q34. What type of energy does a rolling stone have? Q35. What kind of energy does the stone have if it is at rest at the top of a hill? Electrical Charges From the previous study of electrostatics, you learned that a field of force exists in the space surrounding any electrical charge. The strength of the field is directly dependent on the force of the charge. The charge of one electron might be used as a unit of electrical charge, since charges are created by displacement of electrons; but the charge of one electron is so small that it is impractical to use. The practical unit adopted for measuring charges is the COULOMB, named after the scientist Charles Coulomb. One coulomb is equal to the charge of 6,280,000,000,000,000,000 (six quintillion two hundred 18 and eighty quadrillion) or (6.28 x 10 ) electrons. When a charge of one coulomb exists between two bodies, one unit of electrical potential energy exists, which is called the difference of potential between the two bodies. This is referred to as ELECTROMOTIVE FORCE, or VOLTAGE, and the unit of measure is the VOLT. Electrical charges are created by the displacement of electrons, so that there exists an excess of electrons at one point, and a deficiency at another point. Consequently, a charge must always have either a negative or positive polarity. A body with an excess of electrons is considered to be negative, whereas a body with a deficiency of electrons is positive. A difference of potential can exist between two points, or bodies, only if they have different charges. In other words, there is no difference in potential between two bodies if both have a deficiency of electrons to the same degree. If, however, one body is deficient of 6 coulombs (representing 6 volts), and the other is deficient by 12 coulombs (representing 12 volts), there is a difference of potential of 6 volts. The body with the greater deficiency is positive with respect to the other. In most electrical circuits only the difference of potential between two points is of importance and the absolute potentials of the points are of little concern. Very often it is convenient to use one standard reference for all of the various potentials throughout a piece of equipment. For this reason, the potentials at various points in a circuit are generally measured with respect to the metal chassis on which all parts of the circuit are mounted. The chassis is considered to be at zero potential and all other potentials are either positive or negative with respect to the chassis. When used as the reference point, the chassis is said to be at GROUND POTENTIAL. Occasionally, rather large values of voltage may be encountered, in which case the volt becomes too small a unit for convenience. In a situation of this nature, the kilovolt (kV), meaning 1,000 volts, is frequently used. As an example, 20,000 volts would be written as 20 kV. In other cases, the volt may be too large a unit, as when dealing with very small voltages. For this purpose the millivolt (mV), meaning one-thousandth of a volt, and the microvolt (pV), meaning one-millionth of a volt, are used. For example, 0.001 volt would be written as 1 mV, and 0.000025 volt would be written as 25 pV. See Appendix II for exponential symbology. When a difference in potential exists between two charged bodies that are connected by a conductor, electrons will flow along the conductor. This flow is from the negatively charged body to the positively charged body, until the two charges are equalized and the potential difference no longer exists. 1-26 An analogy of this action is shown in the two water tanks connected by a pipe and valve in figure 1-19. At first the valve is closed and all the water is in tank A. Thus, the water pressure across the valve is at maximum. When the valve is opened, the water flows through the pipe from A to B until the water level becomes the same in both tanks. The water then stops flowing in the pipe, because there is no longer a difference in water pressure between the two tanks. TANK A TANK B T 7 ^ Figure 1-19. — Water analogy of electric differences of potential. Electron movement through an electric circuit is directly proportional to the difference in potential or electromotive force (emf), across the circuit, just as the flow of water through the pipe in figure 1-19 is directly proportional to the difference in water level in the two tanks. A fundamental law of electricity is that the ELECTRON FLOW IS DIRECTLY PROPORTIONAL TO THE APPLIED VOLTAGE. If the voltage is increased, the flow is increased. If the voltage is decreased, the flow is decreased. Q36. What term describes voltage or emf? Q37. Convert 2.1 kV to volts. Q38. Express the following in more simple terms, (a) 250,000 volts, (b) 25,000,000 microvolts, (c) 0.001 millivolt. HOW VOLTAGE IS PRODUCED It has been demonstrated that a charge can be produced by rubbing a rubber rod with fur. Because of the friction involved, the rod acquires electrons from the fur, making it negative; the fur becomes positive due to the loss of electrons. These quantities of charge constitute a difference of potential between the rod and the fur. The electrons which make up this difference of potential are capable of doing work if a discharge is allowed to occur. To be a practical source of voltage, the potential difference must not be allowed to dissipate, but must be maintained continuously. As one electron leaves the concentration of negative charge, another must be immediately provided to take its place or the charge will eventually diminish to the point where no further work can be accomplished. A VOLTAGE SOURCE, therefore, is a device which is capable of supplying and maintaining voltage while some type of electrical apparatus is connected to its terminals. The internal action of the source is such that electrons are continuously removed from one terminal, keeping it positive, and simultaneously supplied to the second terminal which maintains a negative charge. 1-27 Presently, there are six known methods for producing a voltage or electromotive force (emf). Some of these methods are more widely used than others, and some are used mostly for specific applications. Following is a list of the six known methods of producing a voltage. 1. FRICTION — Voltage produced by rubbing certain materials together. 2. PRESSURE (piezoelectricity) — Voltage produced by squeezing crystals of certain substances. 3. HEAT (thermoelectricity) — Voltage produced by heating the joint (junction) where two unlike metals are joined. 4. LIGHT (photoelectricity) — Voltage produced by light striking photosensitive (light sensitive) substances. 5. CHEMICAL ACTION — Voltage produced by chemical reaction in a battery cell. 6. MAGNETISM — Voltage produced in a conductor when the conductor moves through a magnetic field, or a magnetic field moves through the conductor in such a manner as to cut the magnetic lines of force of the field. Voltage Produced by Friction The first method discovered for creating a voltage was that of generation by friction. The development of charges by rubbing a rod with fur is a prime example of the way in which a voltage is generated by friction. Because of the nature of the materials with which this voltage is generated, it cannot be conveniently used or maintained. For this reason, very little practical use has been found for voltages generated by this method. In the search for methods to produce a voltage of a larger amplitude and of a more practical nature, machines were developed in which charges were transferred from one terminal to another by means of rotating glass discs or moving belts. The most notable of these machines is the Van de Graaff generator. It is used today to produce potentials in the order of millions of volts for nuclear research. As these machines have little value outside the field of research, their theory of operation will not be described here. Q39. A device which supplies a voltage is commonly referred to by what name? Voltage Produced by Pressure One specialized method of generating an emf utilizes the characteristics of certain ionic crystals such as quartz, Rochelle salts, and tourmaline. These crystals have the remarkable ability to generate a voltage whenever stresses are applied to their surfaces. Thus, if a crystal of quartz is squeezed, charges of opposite polarity will appear on two opposite surfaces of the crystal. If the force is reversed and the crystal is stretched, charges will again appear, but will be of the opposite polarity from those produced by squeezing. If a crystal of this type is given a vibratory motion, it will produce a voltage of reversing polarity between two of its sides. Quartz or similar crystals can thus be used to convert mechanical energy into electrical energy. This phenomenon, called the PIEZOELECTRIC EFFECT, is shown in figure 1-20. Some of the common devices that make use of piezoelectric crystals are microphones, phonograph cartridges, and oscillators used in radio transmitters, radio receivers, and sonar equipment. This method of generating an emf is not suitable for applications having large voltage or power requirements, but is widely used in sound and communications systems where small signal voltages can be effectively used. 1-28 MOLECULES OF NON-CRYSTALLIZED MATTER (A) MOLECULES OF CRYSTALLIZED MATTER (B) Figure 1-20. — (A) Noncry stallized structure; (B) crystallized structure; (C) compression of a crystal; (D) decompression of a crystal. Crystals of this type also possess another interesting property, the "converse piezoelectric effect." That is, they have the ability to convert electrical energy into mechanical energy. A voltage impressed across the proper surfaces of the crystal will cause it to expand or contract its surfaces in response to the voltage applied. Voltage Produced by Heat When a length of metal, such as copper, is heated at one end, electrons tend to move away from the hot end toward the cooler end. This is true of most metals. However, in some metals, such as iron, the opposite takes place and electrons tend to move TOWARD the hot end. These characteristics are illustrated in figure 1-21. The negative charges (electrons) are moving through the copper away from the heat and through the iron toward the heat. They cross from the iron to the copper through the current meter to the iron at the cold junction. This device is generally referred to as a THERMOCOUPLE. Figure 1-21. — Voltage produced by heat. 1-29 Thermocouples have somewhat greater power capacities than crystals, but their capacity is still very small if compared to some other sources. The thermoelectric voltage in a thermocouple depends mainly on the difference in temperature between the hot and cold junctions. Consequently, they are widely used to measure temperature, and as heat-sensing devices in automatic temperature control equipment. Thermocouples generally can be subjected to much greater temperatures than ordinary thermometers, such as the mercury or alcohol types. Voltage Produced by Light When light strikes the surface of a substance, it may dislodge electrons from their orbits around the surface atoms of the substance. This occurs because light has energy, the same as any moving force. Some substances, mostly metallic ones, are far more sensitive to light than others. That is, more electrons will be dislodged and emitted from the surface of a highly sensitive metal, with a given amount of light, than will be emitted from a less sensitive substance. Upon losing electrons, the photosensitive (light-sensitive) metal becomes positively charged, and an electric force is created. Voltage produced in this manner is referred to as a PHOTOELECTRIC VOLTAGE. The photosensitive materials most commonly used to produce a photoelectric voltage are various compounds of silver oxide or copper oxide. A complete device which operates on the photoelectric principle is referred to as a "photoelectric cell." There are many different sizes and types of photoelectric cells in use, and each serves the special purpose for which it is designed. Nearly all, however, have some of the basic features of the photoelectric cells shown in figure 1-22. 1-30 PHOTOSENSITIVE SILVER OXIDE SURFACE © © 0 | M £.k ANODE ELECTRONS EMTTED TOWARD ANODE LIGHT SOURCE 4 E LE CTRON FLOW (A) LIGHT SOURCE SEMITRANSPARENT LAYER PASSES LIGHT AND COLLECTS PHOTOELECTRONS PHOTOSENSITIVE COPPER OXIDE ELECTRON FLOW T) PURE COPPER BASE LAYER (B) Figure 1-22. — Voltage produced by light. The cell (fig. 1-22 view A) has a curved light-sensitive surface focused on the central anode. When light from the direction shown strikes the sensitive surface, it emits electrons toward the anode. The more intense the light, the greater the number of electrons emitted. When a wire is connected between the filament and the back, or dark side of the cell, the accumulated electrons will flow to the dark side. These electrons will eventually pass through the metal of the reflector and replace the electrons leaving the light- sensitive surface. Thus, light energy is converted to a flow of electrons, and a usable current is developed. The cell (fig. 1-22 view B) is constructed in layers. A base plate of pure copper is coated with light- sensitive copper oxide. An extremely thin semitransparent layer of metal is placed over the copper oxide. This additional layer serves two purposes: 1 . It permits the penetration of light to the copper oxide. 2. It collects the electrons emitted by the copper oxide. An externally connected wire completes the electron path, the same as in the reflector-type cell. The photocell’s voltage is used as needed by connecting the external wires to some other device, which amplifies (enlarges) it to a usable level. The power capacity of a photocell is very small. However, it reacts to light-intensity variations in an extremely short time. This characteristic makes the photocell very useful in detecting or accurately 1-31 controlling a great number of operations. For instance, the photoelectric cell, or some form of the photoelectric principle, is used in television cameras, automatic manufacturing process controls, door openers, burglar alarms, and so forth. Voltage Produced by Chemical Action Voltage may be produced chemically when certain substances are exposed to chemical action. If two dissimilar substances (usually metals or metallic materials) are immersed in a solution that produces a greater chemical action on one substance than on the other, a difference of potential will exist between the two. If a conductor is then connected between them, electrons will flow through the conductor to equalize the charge. This arrangement is called a primary cell. The two metallic pieces are called electrodes and the solution is called the electrolyte. The voltaic cell illustrated in figure 1-23 is a simple example of a primary cell. The difference of potential results from the fact that material from one or both of the electrodes goes into solution in the electrolyte, and in the process, ions form in the vicinity of the electrodes. Due to the electric field associated with the charged ions, the electrodes acquire charges. ZINC ELECTRODE + COPPER ELECTRODE GLASS JAR Figure 1-23. — Voltaic cell. The amount of difference in potential between the electrodes depends principally on the metals used. The type of electrolyte and the size of the cell have little or no effect on the potential difference produced. There are two types of primary cells, the wet cell and the dry cell. In a wet cell the electrolyte is a liquid. A cell with a liquid electrolyte must remain in an upright position and is not readily transportable. An automotive battery is an example of this type of cell. The dry cell, much more commonly used than the wet cell, is not actually dry, but contains an electrolyte mixed with other materials to form a paste. Flashlights and portable radios are commonly powered by dry cells. Batteries are formed when several cells are connected together to increase electrical output. 1-32 Voltage Produced by Magnetism Magnets or magnetic devices are used for thousands of different jobs. One of the most useful and widely employed applications of magnets is in the production of vast quantities of electric power from mechanical sources. The mechanical power may be provided by a number of different sources, such as gasoline or diesel engines, and water or steam turbines. However, the final conversion of these source energies to electricity is done by generators employing the principle of electromagnetic induction. These generators, of many types and sizes, are discussed in other modules in this series. The important subject to be discussed here is the fundamental operating principle of ALL such electromagnetic-induction generators. To begin with, there are three fundamental conditions which must exist before a voltage can be produced by magnetism. 1. There must be a CONDUCTOR in which the voltage will be produced. 2. There must be a MAGNETIC FIELD in the conductor’s vicinity. 3. There must be relative motion between the field and conductor. The conductor must be moved so as to cut across the magnetic lines of force, or the field must be moved so that the lines of force are cut by the conductor. In accordance with these conditions, when a conductor or conductors MOVE ACROSS a magnetic field so as to cut the lines of force, electrons WITHIN THE CONDUCTOR are propelled in one direction or another. Thus, an electric force, or voltage, is created. In figure 1-24, note the presence of the three conditions needed for creating an induced voltage. MAGNET MAGNETIC FIELD COPPER WIRE CONDUCTOR DIRECTION OF MOTION NO INDUCED E.M.F. NO MOTION INDUCED E.M.F. REVERSED MOTION REVERSED Figure 1-24. — Voltage produced by magnetism. 1 . A magnetic field exists between the poles of the C-shaped magnet. 2. There is a conductor (copper wire). 3. There is a relative motion. The wire is moved back and forth ACROSS the magnetic field. 1-33 In figure 1-24 view A, the conductor is moving TOWARD the front of the page and the electrons move from left to right. The movement of the electrons occurs because of the magnetically induced emf acting on the electrons in the copper. The right-hand end becomes negative, and the left-hand end positive. The conductor is stopped at view B, motion is eliminated (one of the three required conditions), and there is no longer an induced emf. Consequently, there is no longer any difference in potential between the two ends of the wire. The conductor at view C is moving away from the front of the page. An induced emf is again created. However, note carefully that the REVERSAL OF MOTION has caused a REVERSAL OF DIRECTION in the induced emf. If a path for electron flow is provided between the ends of the conductor, electrons will leave the negative end and flow to the positive end. This condition is shown in part view D. Electron flow will continue as long as the emf exists. In studying figure 1-24, it should be noted that the induced emf could also have been created by holding the conductor stationary and moving the magnetic field back and forth. The more complex aspects of power generation by use of mechanical motion and magnetism are discussed later in this series, under the heading "Generators and Motors." Q40. Name the six methods of producing a voltage. Q41. The piezoelectric effect is an example of a voltage being produced by what method? Q42. A thermocouple is a device that produces voltage by what method? Q43. A battery uses what method to produce a voltage? Q44. A generator uses what method to produce a voltage? ELECTRIC CURRENT It has been proven that electrons (negative charges) move through a conductor in response to an electric field. ELECTRON CURRENT FLOW will be used throughout this explanation. Electron current is defined as the directed flow of electrons . The direction of electron movement is from a region of negative potential to a region of positive potential. Therefore electric current can be said to flow from negative to positive . The direction of current flow in a material is determined by the polarity of the applied voltage. NOTE: In some electrical/electronic communities, the direction of current flow is recognized as being from positive to negative. Q45. According to electron theory , an electric current flows from what potential to what potential? Random Drift All materials are composed of atoms, each of which is capable of being ionized. If some form of energy, such as heat, is applied to a material, some electrons acquire sufficient energy to move to a higher energy level. As a result, some electrons are freed from their parent atom’s which then becomes ions. Other forms of energy, particularly light or an electric field, will cause ionization to occur. The number of free electrons resulting from ionization is dependent upon the quantity of energy applied to a material, as well as the atomic structure of the material. At room temperature some materials, classified as conductors, have an abundance of free electrons. Under a similar condition, materials classified as insulators have relatively few free electrons. In a study of electric current, conductors are of major concern. Conductors are made up of atoms that contain loosely bound electrons in their outer orbits. Due to the effects of increased energy, these outermost electrons frequently break away from their atoms and freely drift throughout the material. The 1-34 free electrons, also called mobile electrons, take a path that is not predictable and drift about the material in a haphazard manner. Consequently such a movement is termed RANDOM DRIFT. It is important to emphasize that the random drift of electrons occurs in all materials. The degree of random drift is greater in a conductor than in an insulator. Directed Drift Associated with every charged body there is an electrostatic field. Bodies that are charged alike repel one another and bodies with unlike charges attract each other. An electron will be affected by an electrostatic field in exactly the same manner as any negatively charged body. It is repelled by a negative charge and attracted by a positive charge. If a conductor has a difference in potential impressed across it, as shown in figure 1-25, a direction is imparted to the random drift. This causes the mobile electrons to be repelled away from the negative terminal and attracted toward the positive terminal. This constitutes a general migration of electrons from one end of the conductor to the other. The directed migration of mobile electrons due to the potential difference is called DIRECTED DRIFT. Figure 1-25. — Directed drift. The directed movement of the electrons occurs at a relatively low VELOCITY (rate of motion in a particular direction). The effect of this directed movement, however, is felt almost instantaneously, as explained by the use of figure 1-26. As a difference in potential is impressed across the conductor, the positive terminal of the battery attracts electrons from point A. Point A now has a deficiency of electrons. As a result, electrons are attracted from point B to point A. Point B has now developed an electron deficiency, therefore, it will attract electrons. This same effect occurs throughout the conductor and repeats itself from points D to C. At the same instant the positive battery terminal attracted electrons from point A, the negative terminal repelled electrons toward point D. These electrons are attracted to point D as it gives up electrons to point C. This process is continuous for as long as a difference of potential exists across the conductor. Though an individual electron moves quite slowly through the conductor, the effect of a directed drift occurs almost instantaneously. As an electron moves into the conductor at point D, an electron is leaving at point A. This action takes place at approximately the speed a light (186,000 miles Per Second). 1-35 Figure 1-26. — Effect of directed drift. Q46. The effects of directed drift take place at what rate of speed? Magnitude of Current Flow Electric current has been defined as the directed movement of electrons. Directed drift, therefore, is current and the terms can be used interchangeably. The expression directed drift is particularly helpful in differentiating between the random and directed motion of electrons. However, CURRENT FLOW is the terminology most commonly used in indicating a directed movement of electrons. The magnitude of current flow is directly related to the amount of energy that passes through a conductor as a result of the drift action. An increase in the number of energy carriers (the mobile electrons) or an increase in the energy of the existing mobile electrons would provide an increase in current flow. When an electric potential is impressed across a conductor, there is an increase in the velocity of the mobile electrons causing an increase in the energy of the carriers. There is also the generation of an increased number of electrons providing added carriers of energy. The additional number of free electrons is relatively small, hence the magnitude of current flow is primarily dependent on the velocity of the existing mobile electrons. The magnitude of current flow is affected by the difference of potential in the following manner. Initially, mobile electrons are given additional energy because of the repelling and attracting electrostatic field. If the potential difference is increased, the electric field will be stronger, the amount of energy imparted to a mobile electron will be greater, and the current will be increased. If the potential difference is decreased, the strength of the field is reduced, the energy supplied to the electron is diminished, and the current is decreased. Q47. What is the relationship of current to voltage in a circuit? Measurement of Current The magnitude of current is measured in AMPERES. A current of one ampere is said to flow when one coulomb of charge passes a point in one second. Remember, one coulomb is equal to the charge of 6.28 x 10 18 electrons. 1-36 Frequently, the ampere is much too large a unit for measuring current. Therefore, the MILLIAMPERE (mA), one-thousandth of an ampere, or the MICROAMPERE (pA), one-millionth of an ampere, is used. The device used to measure current is called an AMMETER and will be discussed in detail in a later module. Q48. Convert 350 mA to amperes. ELECTRICAL RESISTANCE It is known that the directed movement of electrons constitutes a current flow. It is also known that the electrons do not move freely through a conductor’s crystalline structure. Some materials offer little opposition to current flow, while others greatly oppose current flow. This opposition to current flow is known as RESISTANCE (R), and the unit of measure is the OHM. The standard of measure for one ohm is the resistance provided at zero degrees Celsius by a column of mercury having a cross-sectional area of one square millimeter and a length of 106.3 centimeters. A conductor has one ohm of resistance when an applied potential of one volt produces a current of one ampere. The symbol used to represent the ohm is the Greek letter omega (£2). Resistance, although an electrical property, is determined by the physical structure of a material. The resistance of a material is governed by many of the same factors that control current flow. Therefore, in a subsequent discussion, the factors that affect current flow will be used to assist in the explanation of the factors affecting resistance. Q49. What is the symbol for ohm? Factors That Affect Resistance The magnitude of resistance is determined in part by the "number of free electrons" available within the material. Since a decrease in the number of free electrons will decrease the current flow, it can be said that the opposition to current flow (resistance) is greater in a material with fewer free electrons. Thus, the resistance of a material is determined by the number of free electrons available in a material. A knowledge of the conditions that limit current flow and, therefore, affect resistance can now be used to consider how the type of material, physical dimensions, and temperature will affect the resistance of a conductor. TYPE OF MATERIAL. — Depending upon their atomic structure, different materials will have different quantities of free electrons. Therefore, the various conductors used in electrical applications have different values of resistance. Consider a simple metallic substance. Most metals are crystalline in structure and consist of atoms that are tightly bound in the lattice network. The atoms of such elements are so close together that the electrons in the outer shell of the atom are associated with one atom as much as with its neighbor. (See fig. 1-27 view A). As a result, the force of attachment of an outer electron with an individual atom is practically zero. Depending on the metal, at least one electron, sometimes two, and in a few cases, three electrons per atom exist in this state. In such a case, a relatively small amount of additional electron energy would free the outer electrons from the attraction of the nucleus. At normal room temperature materials of this type have many free electrons and are good conductors. Good conductors will have a low resistance. 1-37 Figure 1-27. — Atomic spacing in conductors. If the atoms of a material are farther apart, as illustrated in figure 1-27 view B, the electrons in the outer shells will not be equally attached to several atoms as they orbit the nucleus. They will be attracted by the nucleus of the parent atom only. Therefore, a greater amount of energy is required to free any of these electrons. Materials of this type are poor conductors and therefore have a high resistance . Silver, gold, and aluminum are good conductors. Therefore, materials composed of their atoms would have a low resistance. The element copper is the conductor most widely used throughout electrical applications. Silver has a lower resistance than copper but its cost limits usage to circuits where a high conductivity is demanded. Aluminum, which is considerably lighter than copper, is used as a conductor when weight is a major factor. Q50. When would silver be used as a conductor in preference to copper? EFFECT OF CROSS-SECTIONAL AREA. — Cross-sectional area greatly affects the magnitude of resistance. If the cross-sectional area of a conductor is increased, a greater quantity of electrons are available for movement through the conductor. Therefore, a larger current will flow for a given amount of applied voltage. An increase in current indicates that when the cross-sectional area of a conductor is increased, the resistance must have decreased. If the cross-sectional area of a conductor is decreased, the number of available electrons decreases and, for a given applied voltage, the current through the conductor decreases. A decrease in current flow indicates that when the cross-sectional area of a conductor is decreased, the resistance must have increased. Thus, the RESISTANCE OF A CONDUCTOR IS INVERSELY PROPORTIONAL TO ITS CROSS-SECTIONAL AREA. The diameter of conductors used in electronics is often only a fraction of an inch, therefore, the diameter is expressed in mils (thousandths of an inch). It is also standard practice to assign the unit circular mil to the cross-sectional area of the conductor. The circular mil is found by squaring the diameter when the diameter is expressed in mils. Thus, if the diameter is 35 mils (0.035 inch), the circular 1-38 9 mil area is equal to (35) or 1225 circular mils. A comparison between a square mil and a circular mil is illustrated in figure 1-28. CIRCULAR MIL SQUARE MIL NOTE: SHADING REPRESENTS DIFFERENCE IN AREA BETWEEN CIRCULAR AND SQUARE MILS. Figure 1-28. — Square and circular mils. EFFECT OF CONDUCTOR LENGTH.— The length of a conductor is also a factor which determines the resistance of a conductor. If the length of a conductor is increased, the amount of energy given up increases. As free electrons move from atom to atom some energy is given off as heat. The longer a conductor is, the more energy is lost to heat. The additional energy loss subtracts from the energy being transferred through the conductor, resulting in a decrease in current flow for a given applied voltage. A decrease in current flow indicates an increase in resistance, since voltage was held constant. Therefore, if the length of a conductor is increased, the resistance increases. THE RESISTANCE OF A CONDUCTOR IS DIRECTLY PROPORTIONAL TO ITS LENGTH. Q51. Which wire has the least resistance? Wire A-copper, 1000 circular mils, 6 inches long. Wire B-copper, 2000 circular mils, 11 inches long. EFFECT OF TEMPERATURE. — Temperature changes affect the resistance of materials in different ways. In some materials an increase in temperature causes an increase in resistance, whereas in others, an increase in temperature causes a decrease in resistance. The amount of change of resistance per unit change in temperature is known as the TEMPERATURE COEFFICIENT. If for an increase in temperature the resistance of a material increases, it is said to have a POSITIVE TEMPERATURE COEFFICIENT. A material whose resistance decreases with an increase in temperature has a NEGATIVE TEMPERATURE COEFFICIENT. Most conductors used in electronic applications have a positive temperature coefficient. However, carbon, a frequently used material, is a substance having a negative temperature coefficient. Several materials, such as the alloys constantan and manganin, are considered to have a ZERO TEMPERATURE COEFFICIENT because their resistance remains relatively constant for changes in temperature. Q52. Which temperature coefficient indicates a material whose resistance increases as temperature increases ? 1-39 Q5 3. What term describes a material whose resistance remains relatively constant with changes in temperature? CONDUCTANCE Electricity is a study that is frequently explained in terms of opposites. The term that is the opposite of resistance is CONDUCTANCE. Conductance is the ability of a material to pass electrons. The factors that affect the magnitude of resistance are exactly the same for conductance, but they affect conductance in the opposite manner. Therefore, conductance is directly proportional to area, and inversely proportional to the length of the material. The temperature of the material is definitely a factor, but assuming a constant temperature, the conductance of a material can be calculated. The unit of conductance is the MHO (G), which is ohm spelled backwards. Recently the term mho has been redesignated SIEMENS (S). Whereas the symbol used to represent resistance (R) is the Greek letter omega (£2), the symbol used to represent conductance (G) is (S). The relationship that exists between resistance (R) and conductance (G) or (S) is a reciprocal one. A reciprocal of a number is one divided by that number. In terms of resistance and conductance: Q54. What is the unit of conductance and what other term is sometimes used? Q55. What is the relationship between conductance and resistance ? ELECTRICAL RESISTORS Resistance is a property of every electrical component. At times, its effects will be undesirable. However, resistance is used in many varied ways. RESISTORS are components manufactured to possess specific values of resistance. They are manufactured in many types and sizes. When drawn using its schematic representation, a resistor is shown as a series of jagged lines, as illustrated in figure 1-29. 1-40 TYPICAL RESISTOR TYPE SYMBOL FIXED CARBON FIXED WIREWOUND (TAPPED) ADJUSTABLE WIREWOUND -y&- POTENTIOMETER RHEOSTAT Figure 1-29. — Types of resistors. Q56. What is schematic symbol for a resistor? Composition of Resistors One of the most common types of resistors is the molded composition, usually referred to as the carbon resistor. These resistors are manufactured in a variety of sizes and shapes. The chemical composition of the resistor determines its ohmic value and is accurately controlled by the manufacturer in the development process. They are made in ohmic values that range from one ohm to millions of ohms. The physical size of the resistor is related to its wattage rating, which is the ability of resistor to dissipate heat caused by the resistance. Carbon resistors, as you might suspect, have as their principal ingredient the element carbon. In the manufacturer of carbon resistors, fillers or binders are added to the carbon to obtain various resistor values. Examples of these fillers are clay, bakelite, rubber, and talc. These fillers are doping agents and cause the overall conduction characteristics to change. Carbon resistors are the most common resistors found because they are easy to manufacturer, inexpensive, and have a tolerance that is adequate for most electrical and electronic applications. Their prime disadvantage is that they have a tendency to change value as they age. One other disadvantage of carbon resistors is their limited power handling capacity. The disadvantage of carbon resistors can be overcome by the use of WIREWOUND resistors (fig. 1-29 (B) and (C)). Wirewound resistors have very accurate values and possess a higher current handling capability than carbon resistors. The material that is frequently used to manufacture wirewound resistors 1-41 is German silver which is composed of copper, nickel, and zinc. The qualities and quantities of these elements present in the wire determine the resistivity of the wire. (The resistivity of the wire is the measure or ability of the wire to resist current. Usually the percent of nickel in the wire determines the resistivity.) One disadvantage of the wirewound resistor is that it takes a large amount of wire to manufacture a resistor of high ohmic value, thereby increasing the cost. A variation of the wirewound resistor provides an exposed surface to the resistance wire on one side. An adjustable tap is attached to this side. Such resistors, sometimes with two or more adjustable taps, are used as voltage dividers in power supplies and other applications where a specific voltage is desired to be "tapped" off. Q57. What does the wattage rating of a resistor indicate? Q58. What are the two disadvantages of carbon-type resistors? Q59. What type resistor should be used to overcome the disadvantages of the carbon resistor? Fixed and Variable Resistors There are two kinds of resistors, FIXED and VARIABLE. The fixed resistor will have one value and will never change (other than through temperature, age, etc.). The resistors shown in A and B of figure 1- 29 are classed as fixed resistors. The tapped resistor illustrated in B has several fixed taps and makes more than one resistance value available. The sliding contact resistor shown in C has an adjustable collar that can be moved to tap off any resistance within the ohmic value range of the resistor. There are two types of variable resistors, one called a POTENTIOMETER and the other a RHEOSTAT (see views D and E of fig. 1-29.) An example of the potentiometer is the volume control on your radio, and an example of the rheostat is the dimmer control for the dash lights in an automobile. There is a slight difference between them. Rheostats usually have two connections, one fixed and the other moveable. Any variable resistor can properly be called a rheostat. The potentiometer always has three connections, two fixed and one moveable. Generally, the rheostat has a limited range of values and a high current-handling capability. The potentiometer has a wide range of values, but it usually has a limited current-handling capability. Potentiometers are always connected as voltage dividers. (Voltage dividers are discussed in Chapter 3.) Q60. Describe the differences between the rheostat connections and those of the potentiometer. Q61. Which type of variable resistor should you select for controlling a large amount of current? Wattage Rating When a current is passed through a resistor, heat is developed within the resistor. The resistor must be capable of dissipating this heat into the surrounding air; otherwise, the temperature of the resistor rises causing a change in resistance, or possibly causing the resistor to burn out. The ability of the resistor to dissipate heat depends upon the design of the resistor itself. This ability to dissipate heat depends on the amount of surface area which is exposed to the air. A resistor designed to dissipate a large amount of heat must therefore have a large physical size. The heat dissipating capability of a resistor is measured in WATTS (this unit will be explained later in chapter 3). Some of the more common wattage ratings of carbon resistors are: one-eighth watt, one-fourth watt, one-half watt, one watt, and two watts. In some of the newer state-of-the-art circuits of today, much smaller wattage resistors are used. Generally, the type that you will be able to physically work with are of the values given. The higher the wattage rating of the resistor the larger is the physical size. Resistors that dissipate very large amounts of power (watts) are usually wirewound resistors. Wirewound resistors with wattage ratings up to 50 1-42 watts are not uncommon. Figure 1-30 shows some resistors which have different wattage ratings. Notice the relative sizes of the resistors. CARBON RESISTORS Figure 1-30. — Resistors of different wattage ratings. Standard Color Code System In the standard color code system, four bands are painted on the resistor, as shown in figure 1-31. ORANGE VIOLET RED SILVER :■ TOLERANCE MULTIPLIER SIGNIFICANT DIGIT •— 1 d SIGNIFICANT DIGIT Figure 1-31. — Resistor color codes. 1-43 Examples of resistor color codes. The color of the first band indicates the value of the first significant digit. The color of the second band indicates the value of the second significant digit. The third color band represents a decimal multiplier by which the first two digits must be multiplied to obtain the resistance value of the resistor. The colors for the bands and their corresponding values are shown in Table 1-1. Table 1-1. — Standard Color Code for Resistors COLOR SIGNIFICANT FIGURE DECIMAL MULTIPLIER TOLERANCE PERCENT RELIABILITY LEVEL PER 1 ,000 HRS. BLACK 0 1 PERCENT 1.0% BROWN 1 10 1 .1% RED 2 100 2 .01% ORANGE 3 1,000 .001% YELLOW 4 10,000 GREEN 5 100,000 BLUE 6 1 ,000,000 VIOLET 7 10,000,000 GRAY 8 100,000,000 WHITE 9 1,000,000,000 GOLD .1 5 SILVER .01 10 NO COLOR 20 Use the example colors shown in figure 1-31. Since red is the color of the first band, the first significant digit is 2. The second band is violet, therefore the second significant digit is 7. The third band is orange, which indicates that the number formed as a result of reading the first two bands is multiplied by 1000. In this case 27 x 1000 = 27,000 ohms. The last band on the resistor indicates the tolerance; that is, the manufacturer's allowable ohmic deviation above and below the numerical value indicated by the resistor's color code. In this example, the color silver indicates a tolerance of 10 percent. In other words, 1-44 the actual value of the resistor may fall somewhere within 10 percent above and 10 percent below the value indicated by the color code. This resistor has an indicated value of 27,000 ohms. Its tolerance is 10 percent x 27,000 ohms, or 2,700 ohms. Therefore, the resistor’s actual value is somewhere between 24,300 ohms and 29,700 ohms. When measuring resistors, you will find situations in which the quantities to be measured may be extremely large, and the resulting number using the basic unit, the ohm, may prove too cumbersome. Therefore, a metric system prefix is usually attached to the basic unit of measurement to provide a more manageable unit. Two of the most commonly used prefixes are kilo and mega. Kilo is the prefix used to represent thousand and is abbreviated k. Mega is the prefix used to represent million and is abbreviated M. In the example given above, the 27,000-ohm resistor could have been written as 27 kilohms or 27 k£2. Other examples are: 1,000 ohms = 1 kfi; 10,000 ohms = 10 kQ; 100,000 ohms = 100 k£2. Likewise, 1,000,000 ohms is written as 1 megohm or 1 MQ. and 10,000,000 ohms = 10 Mf2. Q62. A carbon resistor has a resistance of 50 ohms, and a tolerance of 5 percent. What are the colors of bands one, two, three, and four, respectively? SIMPLIFYING THE COLOR CODE. — Resistors are the most common components used in electronics. The technician must identify, select, check, remove, and replace resistors. Resistors and resistor circuits are usually the easiest branches of electronics to understand. The resistor color code sometimes presents problems to a technician. It really should not, because once the resistor color code is learned, you should remember it for the rest of your life. Black, brown, red, orange, yellow, green, blue, violet, gray, white — this is the order of colors you should know automatically. There is a memory aid that will help you remember the code in its proper order. Each word starts with the first letter of the colors. If you match it up with the color code, you will not forget the code. Bad Boys Run Over Yellow Gardenias Behind Victory Garden Walls, or: Black Bad Brown — Boys Red Run Orange — Over Yellow — Yellow Green — Gardenias Blue — Behind Violet — Victory Gray — Garden White Walls There are many other memory aid sentences that you might want to ask about from experienced technicians. You might find one of the other sentences easier to remember. There is still a good chance that you will make a mistake on a resistor's color band. Most technicians do at one time or another. If you make a mistake on the first two significant colors, it usually is not too 1-45 serious. If you make a miscue on the third band, you are in trouble, because the value is going to be at least 10 times too high or too low. Some important points to remember about the third band are: When the third band is ... . Black, the resistor’s value is less than 100 ohms. Brown, the resistor’s value is in hundreds of ohms. Red, the resistor’s value is in thousands of ohms. Orange, the resistor’s value is in tens of thousands of ohms. Yellow, the resistor’s value is in hundreds of thousands of ohms. Green, the resistor’s value is in megohms. Blue, the resistor’s value is in tens of megohms or more. Although you may find any of the above colors in the third band, red, orange, and yellow are the most common. In some cases, the third band will be silver or gold. You multiply the first two bands by 0.01 if it is silver, and 0.1 if it is gold. The fourth band, which is the tolerance band, usually does not present too much of a problem. If there is no fourth band, the resistor has a 20-percent tolerance; a silver fourth band indicates a 10-percent tolerance; and a gold fourth band indicates a 5-percent tolerance. Resistors that conform to military specifications have a fifth band. The fifth band indicates the reliability level per 1,000 hours of operation as follows: Fifth band color Level Brown 1.0% Red 0.1% Orange 0.01% Yellow 0.001% For a resistor whose the fifth band is color coded brown, the resistor’s chance of failure will not exceed 1 percent for every 1,000 hours of operation. In equipment such as the Navy’s complex computers, the reliability level is very significant. For example, in a piece of equipment containing 10,000 orange fifth-band resistors, no more than one resistor will fail during 1,000 hours of operation. This is very good reliability. More information on resistors is contained in NEETS Module 19. Q63. A carbon resistor has the following color bands: The first band is yellow, followed by violet, yellow, and silver. What is the ohmic value of the resistor? Q64. The same resistor mentioned in question 63 has a yellow fifth band. What does this signify? Q65. A resistor is handed to you for identification with the following color code: the first band is blue, followed by gray, green, gold, and brown. What is the resistor's value? 1-46 Some resistors, both wirewound and composition, will not use the resistor color code. These resistors will have the ohmic value and tolerance imprinted on the resistor itself. SUMMARY With the completion of this chapter, you now have gained the necessary information which is the foundation for the further study of electricity. The following is a summary of the important parts in the chapter. In describing the composition of matter, the following terms are important for you to remember: MATTER is defined as anything that occupies space and has weight. AN ELEMENT is a substance which cannot be reduced to a simpler substance by chemical means. A COMPOUND is a chemical combination of elements which can be separated by chemical means, but not by physical means. It is created by chemically combining two or more elements. A MIXTURE is a combination of elements or compounds that can be separated by physical means. A MOLECULE is the chemical combination of two or more atoms. In a compound, the molecule is the smallest particle that has all the characteristics of the compound. AN ATOM is the smallest particle of an element that retains the characteristics of that element. An atom is made up of electrons, protons, and neutrons. The number and arrangement of these subatomic particles determine the kind of element. PROTONS ELECTRONS NUCLEUS HYDROGEN NEUTRON HELIUM • PROTONS • NEUTRONS AN ELECTRON is considered to be a negative charge of electricity. A PROTON is considered to be a positive charge of electricity. A NEUTRON is a neutral particle in that it has no electrical charge. ENERGY in an electron is of two types — kinetic (energy of motion) and potential (energy of position). 1-47 ENERGY LEVELS of the electron exist because the electron has mass and motion. The motion gives it kinetic energy and its position gives it potential energy. Energy balance keeps the electron in orbit and should it gain energy it will assume an orbit further from the center of the atom. It will remain at that level for only a fraction of a second before it radiates the excess energy and goes back to a lower orbit. SHELLS AND SUBSHELLS of electrons are the orbits of the electrons in the atom. Each shell contains a maximum of 2 times its number squared electrons. Shells are lettered K through Q, starting with K, which is the closest to the nucleus. The shell can be split into 4 subshells labeled s, p, d, and f, which can contain 2, 6, 10, and 14 electrons, respectively. LETTER DESIGNATION SHELL NUMBER VALENCE OF AN ATOM is determined by the number of electrons in the outermost shell. The shell is referred to as the valence shell, and the electrons within it are valence electrons. An atom with few valence electrons requires little energy to free the valence electrons. IONIZATION refers to the electrons contained in an atom. An atom with a positive charge has lost some of its electrons, and is called a positive ion. A negatively charged atom is a negative ion. 1-48 CONDUCTORS, SEMICONDUCTORS, AND INSULATORS are categorized as such by the number of valence electrons in their atoms. The conductor normally has 3 or less valence electrons and offers little opposition to the flow of electrons (electric current). The insulator contains 5 or more valence electrons and offers high opposition to electron flow. The semiconductor usually has four valence electrons of conductivity and is in the midrange. The best conductors in order of conductance are silver, copper, gold, and aluminum. CHARGED BODIES affect each other as follows: When two bodies having unequal charges are brought close to each other, they will tend to attract each other in an attempt to equalize their respective charges. When two bodies, both having either positive or negative charges, are brought close together, they tend to repel each other as no equalization can occur. When the charge on one body is high enough with respect to the charge on an adjacent body, an equalizing current will flow between the bodies regardless of the conductivity of the material containing the bodies. A NEUTRAL BODY may be attracted to either a positively or negatively charged body due to the relative difference in their respective charges. CHARGED BODIES will attract or repel each other with a force that is directly proportional to the product of their individual charges and inversely proportional to the square of the distance between the bodies. 1-49 ELECTROSTATIC LINES of force are a graphic representation of the field around a charged body. These lines are imaginary. Lines from a positively charged body are indicated as flowing out from the body, while lines from a negatively charged body are indicated as flowing into the body. MAGNETISM is that property of a material which enables it to attract pieces of iron. A material with this property is called a magnet. Any material that is attracted to a magnet can be made into a magnet itself. FERROMAGNETIC MATERIALS are materials that are easy to magnetize; e.g., iron, steel, and cobalt. NATURAL MAGNETS , called magnetite, lodestones, or leading stones, were the first magnets to be studied. Most magnets in practical use are artificial or man-made magnets, and are made either by electrical means or by stroking a magnetic material with a magnet. RELUCTANCE is defined as the opposition of a material to being magnetized. PERMEABILITY is defined as the ease with which a material accepts magnetism. A material which is easy to magnetize does not hold its magnetism very long, and vice versa. RETENTIVITY is defined as the ability of a material to retain magnetism. A MAGNETIC POLE is located at each end of a magnet. The majority of the magnetic force is concentrated at these poles and is approximately equal at both poles. 1-50 MAGNETIC NORTH (SOUTHERN POLARITY) GEOGRAPHIC NORTH GEOGRAPHIC SOUTH MAGNETIC SOUTH (NORTHERN POLARRY) THE NORTH POLE , or north seeking pole, of a magnet freely suspended on a string always points toward the north geographical pole. THE LIKE POLES of magnets repel each other, while the unlike poles attract each other. 1-51 WEBER’S THEORY OF MAGNETISM assumes that all magnetic material is made up of magnetic molecules which, if lined up in north to south pole order, will be a magnet. If not lined up, the magnetic fields about the molecules will neutralize each other and no magnetic effect will be noted. BAR BEING MAGNETIZED ED ed EE kJLJ EE ED ED EE ee BAR MAGNETIZED THE DOMAIN THEORY OF MAGNETISM states that if the electrons of the atoms in a material spin more in one direction than in the other, the material will become magnetized. NUMBER OF NUMBER OF ELECTRONS ELECTRONS SPINNING SPINNING A MAGNETIC FIELD is said to exist in the space surrounding a magnet. 1-52 IRON FILINGS GLASS PLATE OVER MAGNET MAGNETIC LINES OF FORCE are imaginary lines used to describe the patterns of the magnetic field about a magnet. These lines are assumed to flow externally from the north pole and into the south pole. MAGNETIC FLUX is the total number of magnetic lines of force leaving or entering the pole of a magnet. 1-53 UNLIKE POLES ATTRACT LIKE POLES REPEL FLUX DENSITY is the number of flux lines per unit area. FIELD INTENSITY or the intensity of a magnetic field is directly related to the magnetic force exerted by the field. THE INTENSITY OF ATTRACTION/REPULSION between magnetic poles may be described by a law almost identical to Coulomb’s Law of Charged Bodies, that is, the force between two poles is directly proportional to the product of the pole strengths and inversely proportional to the square of the distance between the poles. MAGNETIC SHIELDING can be accomplished by placing a soft iron shield around the object to be protected, thus directing the lines of force around the object. 1-54 MAGNETS ARE CLASSIFIED BY SHAPE and include the bar magnet, the horseshoe magnet, and the ring magnet. The ring magnet is used in computer memory circuits; the horseshoe magnet in some meter circuits. ENERGY may be defined as the ability to do work. THE COULOMB (C) is the basic unit used to indicate an electrical charge. One coulomb is equal to 18 a charge of 6.28 x 10 electrons. When one coulomb of charge exists between two bodies, the electromotive force (or voltage) is one volt. VOLTAGE is measured as the difference of potential of two charges of interest. VOLTAGE MEASUREMENTS may be expressed in the following units: volts (V), kilovolts (kV), millivolts (mV), or microvolts (jxV). For example: 1 kV = 1,000 V 1 mV = 0.001 V 1 pV =0.000001 V METHODS OF PRODUCING A VOLTAGE include: 1. Friction 2. Pressure (piezoelectricity) 1-55 3 . Heat (thermoelectricity) 1-56 4. Light (photoelectricity) PHOTOSENSITIVE SILVER OXIDE SURFACE © O' Ivi MODE ELECTRONS EMTTED TOWARD ANODE LIGHT SOURCE E LE CTRON FLOW (A) LIGHT SOURCE SEMITRANSPARENT i LAYER PASSES LIGHT AND COLLECTS PHOTOELECTRONS PHOTOSENSITIVE COPPER OXIDE ELECTRON FLOW PURE COPPER BASE LAYER (B) 5. Chemical action (battery) 1-57 ZINC ELECTRODE + COPPER ELECTRODE DILUTE SULFURIC ACID ELECTROLYTE GLASS JAR 6. Magnetism (electromagnetic induction generator) ELECTROMAGNETIC INDUCTION GENERATOR To produce voltage by use of magnetism, three conditions must be met: There must be a CONDUCTOR in which the voltage will be produced; there must be a MAGNETIC FIELD in the conductor’s vicinity; and there must be relative motion between the field and conductor. When these conditions are met, electrons WITHIN THE CONDUCTOR are propelled in one direction or another, creating an electromotive force, or voltage. MAGNET MAGNETIC FIELD COPPER WIRE CONDUCTOR DIRECTION OF MOTION NO INDUCED E.M.F. NO MOTION INDUCED E.M.F. REVERSED MOTION REVERSED 1-58 ELECTRON CURRENT is based on the assumption that electron current flow is from negative to positive through a circuit. AN ELECTRIC CURRENT is a directed movement of electrons in a conductor or circuit. THE AMPERE is the basic unit used to indicate an electric current. A current of one ampere is said 18 to flow when one coulomb of charge (6.28 x 10 electrons) passes a given point in one second of time. Current measurements may be expressed in the following units: ampere (A), milliampere (mA), and microampere (jxA). Current in a circuit increases in direct proportion to the voltage (emf) applied across the circuit. RESISTANCE is the opposition to current. The ohm is the basic unit of resistance and is represented by the Greek letter omega (Q). A conductor is said to have one ohm of resistance when an emf of one volt causes one ampere of current to flow in the conductor. Resistance may be expressed in the following units: ohm (O), kilohm (k£2), and megohms (M£2). For example, 1,000,000:0, = 1,000 kO = 1 MO. THE RESISTANCE OF A MATERIAL is determined by the type, the physical dimensions, and the temperature of the material that is, 1. A good conductor contains an abundance of free electrons. 2. As the cross-sectional area of a given conductor is increased, the resistance will decrease. 3. As the length of a conductor is increased, the resistance will increase. 4. In a material having a positive temperature coefficient, the resistance will increase as the temperature is increased. THE CONDUCTANCE OF A MATERIAL is the reciprocal of resistance. THE UNIT OF CONDUCTANCE is the mho and the symbol is V. G or S. THE RESISTOR is manufactured to provide a specific value of resistance. THE CARBON RESISTOR is made of carbon, with fillers and binders blended in to control the ohmic value. 1-59 CARBON RESISTORS 1/4 -WATT 50 - WATT 2 - WATT W1 REWOUND RESISTORS THE RESISTANCE OF A WIREWOUND RESISTOR is determined by the metal content of the wire and the wire’s length. Wirewound resistors may be tapped so two or more different voltage values may be taken off the same resistor. TYPICAL RESISTOR TYPE SYMBOL FIXED CARBON FIXED WIREWOUND (TAPPED) — 'vA K— ADJUSTABLE WIREWOUND -y&r- POTENTIOMETER WScM RHEOSTAT vw-j 1-60 THE POTENTIOMETER AND THE RHEOST AT are variable resistors and can be adjusted to any resistance value within their ohmic range. The rheostat is usually used for relatively high current applications and has two connections; the potentiometer has 3 connections and is a relatively high- resistance, low-current device. Two examples of potentiometers. Example of a rheostat. 1-61 Table 1-1. — Standard Color Code for Resistors COLOR SIGNIFICANT FIGURE DECIMAL MULTIPLIER TOLERANCE PERCENT RELIABILITY LEVEL PER 1 ,000 HRS . BLACK 0 1 PERCENT 1.0% BROWN 1 10 1 .1% RED 2 100 2 .01% ORANGE 3 1,000 • 001% YELLOW 4 10,000 GREEN 5 100,000 BLUE 6 1 ,000,000 VIOLET 7 10,000,000 GRAY 8 100,000,000 WHITE 9 1,000,000,000 GOLD .1 5 SILVER .01 10 NO COLOR 20 THE WATTAGE RATING OF A RESISTOR is related to the resistor’s physical size, that is, the greater the surface area exposed to the air, the larger the rating. THE STANDARD COLOR CODE for resistors is used to determine the following: 1 . Ohmic value 2. Tolerance 3. Reliability level (on some resistors) ORANGE VIOLET RED SILVER ) TOLERANCE MULTIPLIER SIGNIFICANT DIGIT *— 1 d SIGNIFICANT DIGIT 1-62 ANSWERS TO QUESTIONS Ql. THROUGH Q65. Al. Anything that occupies space and has weight. Solids , liquids, gases. A2. A substance which cannot be reduced to a simpler substance by chemical means. A3. A substance consisting of two or more elements. A4. A compound is a chemical combination of elements that cannot be separated by physical means. A mixture is a physical combination of elements and compounds that are not chemically combined. A5. A chemical combination of two or more atoms. A6. Electrons -negative, protons-positive, and neutrons -neutral. A7. Kinetic energy. A8. Invisible light photons ( ultraviolet ) bombard the phosphor atom in the light tube. The phosphor atoms emit visible light photons. A9. The number of electrons in the outer shell. A10. An atom with more or less than its normal number of electrons. All. The number of valence electrons. A12. Through the accumulation of excess electrons. A13. By friction. A14. Negative. A15. Like charges repel, and unlike charges attract with a force directly proportional to the product of their charges and inversely proportional to the square of the distance between them. A16. The space between and around charged bodies. A17. Leaving positive, entering negative. A18. Motors, generators, speakers, computers, televisions, tape recorders, and many others. A19. Those materials that are attracted by magnets and have the ability to become magnetized. A20. The relative ease with which they are magnetized. A21. A material that exhibits low reluctance and high permeability, such as iron or soft steel. A22. The ability of a material to retain magnetism. A23. They are very similar; like charges repel, unlike charges attract, like poles repel — unlike poles attract. A24. To the magnetic north pole. A25. South pole at the right, north pole at the left. 1-63 A26. The domain theory is based upon the electron spin principle; Weber’s theory uses the concept of tiny molecular magnets. A27. To enable you to "see" the magnetic field. A28. No specific pattern , sawdust is a nonmagnetic material. A29. An imaginary line used to illustrate magnetic effects. A30. Electrostatic lines of force do not form closed loops. A31. By shielding or surrounding the instrument with a soft iron case , called a magnetic shield or screen. A3 2. In pairs , with opposite poles together to provide a complete path for magnetic flux. A33. The ability to do work. A34. Kinetic energy. A35. Potential energy. A3 6. Difference of potential. A3 7. 2100 volts. A38. (a) 250 kV, (b) 25 V, (c) 1 pV. A39. A voltage source. A40. Friction , pressure , heat, light, chemical action, and magnetism. A41. Pressure. A42. Heat. A43. Chemical. A44. Magnetic. A45. Electron theory assumes that electron flow is from negative to positive. A46. The speed of light ( 186,000 miles per second, 300,000,000 meters per second). A47. Current increases as voltage increases. A48. 0.35 amperes. A49. Q A50. When the need for conductivity is great enough to justify the additional expense. A51. WireB. A52. Positive. 1-64 A53. Zero temperature coefficient. A54. The mho (v), siemens. A55. They are reciprocals of each other. A56. -AAA/- A57. Its ability to dissipate heat. A58. 1. Change value with age. 2. Limited power capacity. A59. The wirewound resistor. A60. The rheostat may have two connections , one fixed and one moveable; the potentiometer always has three connections , one moveable and two fixed. A61. The rheostat. A62. The bands are green , black , black , and gold. A63. 4 70, 000 ohms (470 kilohms ). A64. The resistor’s chance of failure is 0.001 percent for 1000 hours of operation. A65. 6,800,000 ohms (6.8 megohms), with 5% tolerance, and a 1% reliability level. 1-65 CHAPTER 2 BATTERIES LEARNING OBJECTIVES Upon completing this chapter, you will be able to: 1 . State the purpose of a cell. 2. State the purpose of the three parts of a cell. 3. State the difference between the two types of cells. 4. Explain the chemical process that takes place in the primary and secondary cells. 5. Recognize and define the terms electrochemical action, anode, cathode, and electrolyte. 6. State the causes of polarization and local action and describe methods of preventing these effects. 7. Identify the parts of a dry cell. 8. Identify the various dry cells in use today and some of their capabilities and limitations. 9. Identify the four basic secondary cells, their construction, capabilities, and limitations. 10. Define a battery, and identify the three ways of combining cells to form a battery. 1 1 . Describe general maintenance procedures for batteries including the use of the hydrometer, battery capacity, and rating and battery charging. 12. Identify the five types of battery charges. 13. Observe the safety precautions for working with and around batteries. INTRODUCTION The purpose of this chapter is to introduce and explain the basic theory and characteristics of batteries. The batteries which are discussed and illustrated have been selected as representative of many models and types which are used in the Navy today. No attempt has been made to cover every type of battery in use, however, after completing this chapter you will have a good working knowledge of the batteries which are in general use. First, you will learn about the building block of all batteries, the CELL. The explanation will explore the physical makeup of the cell and the methods used to combine cells to provide useful voltage, current, and power. The chemistry of the cell and how chemical action is used to convert chemical energy to electrical energy are also discussed. 2-1 In addition, the care, maintenance, and operation of batteries, as well as some of the safety precautions that should be followed while working with and around batteries are discussed. Batteries are widely used as sources of direct-current electrical energy in automobiles, boats, aircraft, ships, portable electric/electronic equipment, and lighting equipment. In some instances, they are used as the only source of power; while in others, they are used as a secondary or standby power source. A battery consists of a number of cells assembled in a common container and connected together to function as a source of electrical power. THE CELL A cell is a device that transforms chemical energy into electrical energy. The simplest cell, known as either a galvanic or voltaic cell, is shown in figure 2-1. It consists of a piece of carbon (C) and a piece of zinc (Zn) suspended in ajar that contains a solution of water (H 2 0) and sulfuric acid (H 2 S0 4 ) called the electrolyte. ELECTRON CARBON (C) Figure 2-1. — Simple voltaic or galvanic cell. The cell is the fundamental unit of the battery. A simple cell consists of two electrodes placed in a container that holds the electrolyte. In some cells the container acts as one of the electrodes and, in this case, is acted upon by the electrolyte. This will be covered in more detail later. ELECTRODES The electrodes are the conductors by which the current leaves or returns to the electrolyte. In the simple cell, they are carbon and zinc strips that are placed in the electrolyte; while in the dry cell (fig. 2-2), they are the carbon rod in the center and zinc container in which the cell is assembled. 2-2 NEGATIVE TERMINAL (CATHODE) POSITIVE TERMINAL (ANODE) SEALING WAX ZINC CONTAINER AND NEGATIVE ELECTRODE WET PASTE ELECTROLYTE CARBON ROD, OR posftive ELECTRODE Figure 2-2. — Dry cell, cross-sectional view. ELECTROLYTE The electrolyte is the solution that acts upon the electrodes. The electrolyte, which provides a path for electron flow, may be a salt, an acid, or an alkaline solution. In the simple galvanic cell, the electrolyte is in a liquid form. In the dry cell, the electrolyte is a paste. CONTAINER The container which may be constructed of one of many different materials provides a means of holding (containing) the electrolyte. The container is also used to mount the electrodes. In the voltaic cell the container must be constructed of a material that will not be acted upon by the electrolyte. Ql. What is the purpose of a cell? Q2. What are the three parts of a cell? Q3. What is the purpose of each of the three parts of a cell? PRIMARY CELL A primary cell is one in which the chemical action eats away one of the electrodes, usually the negative electrode. When this happens, the electrode must be replaced or the cell must be discarded. In the galvanic -type cell, the zinc electrode and the liquid electrolyte are usually replaced when this happens. In the case of the dry cell, it is usually cheaper to buy a new cell. SECONDARY CELL A secondary cell is one in which the electrodes and the electrolyte are altered by the chemical action that takes place when the cell delivers current. These cells may be restored to their original condition by forcing an electric current through them in the direction opposite to that of discharge. The automobile storage battery is a common example of the secondary cell. 2-3 Q4. What are the two types of cells? Q5. What is the main difference between the two types of cells? ELECTROCHEMICAL ACTION If a load (a device that consumes electrical power) is connected externally to the electrodes of a cell, electrons will flow under the influence of a difference in potential across the electrodes from the CATHODE (negative electrode), through the external conductor to the ANODE (positive electrode). A cell is a device in which chemical energy is converted to electrical energy. This process is called ELECTROCHEMICAL action. The voltage across the electrodes depends upon the materials from which the electrodes are made and the composition of the electrolyte. The current that a cell delivers depends upon the resistance of the entire circuit, including that of the cell itself. The internal resistance of the cell depends upon the size of the electrodes, the distance between them in the electrolyte, and the resistance of the electrolyte. The larger the electrodes and the closer together they are in the electrolyte (without touching), the lower the internal resistance of the cell and the more current the cell is capable of supplying to the load. Q6. What is electrochemical action? Q7. What is another name for the (a) positive electrode, and the (b) negative electrode? PRIMARY CELL CHEMISTRY When a current flows through a primary cell having carbon and zinc electrodes and a diluted solution of sulfuric acid and water (combined to form the electrolyte), the following chemical reaction takes place. The current flow through the load is the movement of electrons from the negative electrode of the cell (zinc) and to the positive electrode (carbon). This causes fewer electrons in the zinc and an excess of electrons in the carbon. Figure 2-1 shows the hydrogen ions (H 2 ) from the sulfuric acid being attracted to the carbon electrode. Since the hydrogen ions are positively charged, they are attracted to the negative charge on the carbon electrode. This negative charge is caused by the excess of electrons. The zinc electrode has a positive charge because it has lost electrons to the carbon electrode. This positive charge attracts the negative ions (S0 4 ) from the sulfuric acid. The negative ions combine with the zinc to form zinc sulfate. This action causes the zinc electrode to be eaten away. Zinc sulfate is a grayish-white substance that is sometimes seen on the battery post of an automobile battery. The process of the zinc being eaten away and the sulfuric acid changing to hydrogen and zinc sulfate is the cause of the cell discharging. When the zinc is used up, the voltage of the cell is reduced to zero. In figure 2-1 you will notice that the zinc electrode is labeled negative and the carbon electrode is labeled positive. This represents the current flow outside the cell from negative to positive. The zinc combines with the sulfuric acid to form zinc sulfate and hydrogen. The zinc sulfate dissolves in the electrolyte (sulfuric acid and water) and the hydrogen appears as gas bubbles around the carbon electrode. As current continues to flow, the zinc gradually dissolves and the solution changes to zinc sulfate and water. The carbon electrode does not enter into the chemical changes taking place, but simply provides a return path for the current. 2-4 Q8. In the primary cell, why are negative ions attracted to the negative terminal of the cell? Q9. How do electrons get from the negative electrode to the positive electrode? Q10. What causes the negative electrode to be eaten away? SECONDARY CELL CHEMISTRY As stated before, the differences between primary and secondary cells are, the secondary cell can be recharged and the electrodes are made of different materials. The secondary cell shown in figure 2-3 uses sponge lead as the cathode and lead peroxide as the anode. This is the lead-acid type cell and will be used to explain the general chemistry of the secondary cell. Later in the chapter when other types of secondary cells are discussed, you will see that the materials which make up the parts of a cell are different, but that the chemical action is essentially the same. SOLUTION % I IS Si • I 1 & SULFURIC PC ID H 2 S0 4 PND WPTER H 2 0 SPONGE LEPD Pb (P) CHPRGED LEPD PEROXIDE Pb0 2 i SOLUTION SULFURIC PC ID INCREPSING WPTER DECREPSING ■ (D) CHPRGING INCREPSING SPONGE LEPD DECREPSING LEPD SULFPTE PbS0 4 + Pb INCREPSING LEPD PEROXIDE DECREPSING LEPD SULFPTE PbS0 4 + Pb0 wv LOPD SOLUTION A £ " m 'i £ SULFURIC PCID * DECREPSING * * a WPTER INCREPSING m i:b:i oischprging DECREPSING SPONGE LEPD INCREPSING LEPD SULFPTE Pb + PbS0, DECREPSING LEPD PEROXIDE INCREPSING LEPD SULFPTE PbO.-, + PbS0 4 * SOLUTION 1 MINIMUM SULFURIC PC ID MPXIMUM WPTER m (C) DISCHPRGED MINIMUM SPONGE LEPD MPXIMUM LEPD SULFPTE PbS0 1 + Pb MINIMUM LEPD PEROXIDE MPX I MU 1 LEPD SULFPTE PbSGj + Pb0 2 SPONGE LEPD LEPD PEROXIDE LEPD SULFPTE Figure 2-3. — Secondary cell. 2-5 Figure 2-3 view A shows a lead-acid secondary cell that is fully charged. The cathode is pure sponge lead, the anode is pure lead peroxide, and the electrolyte is a mixture of sulfuric acid and water. Figure 2-3 view B shows the secondary cell discharging. A load is connected between the cathode and anode; current flows negative to positive as shown. This current flow creates the same process as was explained for the primary cell with the following exceptions. In the primary cell the zinc cathode was eaten away by the sulfuric acid. In the secondary cell the sponge -like construction of the cathode retains the lead sulfate formed by the chemical action of the sulfuric acid and the lead. In the primary cell the carbon anode was not chemically acted upon by the sulfuric acid. In the secondary cell the lead peroxide anode is chemically changed to lead sulfate by the sulfuric acid. When the cell is fully discharged it will be as shown in figure 2-3 view C. The anode and cathode retain some lead peroxide and sponge lead but the amounts of lead sulfate in each is maximum. The electrolyte has a minimum amount of sulfuric acid. With this condition no further chemical action can take place within the cell. As you know, the secondary cell can be recharged. Recharging is the process of reversing the chemical action that occurs as the cell discharges. To recharge the cell, a voltage source, such as a generator, is connected as shown in figure 2-3 view D. The negative terminal of the voltage source is connected to the cathode of the cell and the positive terminal of the voltage source is connected to the anode of the cell. With this arrangement the lead sulfate is chemically changed back to sponge lead in the cathode, lead peroxide in the anode, and sulfuric acid in the electrolyte. After all the lead sulfate is chemically changed, the cell is fully charged as shown in figure 2-3 view A. Once the cell has been charged, the discharge-charge cycle may be repeated. Qll. Refer to figure 2-3(B). Why is the sulfuric acid decreasing? Q12. Refer to figure 2-3 (D). How is it possible for the sulfuric acid to be increasing? Q13. Refer to figure 2-3(D). When all the lead sulfate has been converted, what is the condition of the cell? POLARIZATION OF THE CELL The chemical action that occurs in the cell while the current is flowing causes hydrogen bubbles to form on the surface of the anode. This action is called POLARIZATION. Some hydrogen bubbles rise to the surface of the electrolyte and escape into the air, some remain on the surface of the anode. If enough bubbles remain around the anode, the bubbles form a barrier that increases internal resistance. When the internal resistance of the cell increases, the output current is decreased and the voltage of the cell also decreases. A cell that is heavily polarized has no useful output. There are several methods to prevent polarization or to depolarize the cell. One method uses a vent on the cell to permit the hydrogen to escape into the air. A disadvantage of this method is that hydrogen is not available to reform into the electrolyte during recharging. This problem is solved by adding water to the electrolyte, such as in an automobile battery. A second method is to use material that is rich in oxygen, such as manganese dioxide, which supplies free oxygen to combine with the hydrogen and form water. 2-6 A third method is to use a material that will absorb the hydrogen, such as calcium. The calcium releases hydrogen during the charging process. All three methods remove enough hydrogen so that the cell is practically free from polarization. LOCAL ACTION When the external circuit is removed, the current ceases to flow, and, theoretically, all chemical action within the cell stops. However, commercial zinc contains many impurities, such as iron, carbon, lead, and arsenic. These impurities form many small electrical cells within the zinc electrode in which current flows between the zinc and its impurities. Thus, the chemical action continues even though the cell itself is not connected to a load. Local action may be prevented by using pure zinc (which is not practical), by coating the zinc with mercury, or by adding a small percentage of mercury to the zinc during the manufacturing process. The treatment of the zinc with mercury is called amalgamating (mixing) the zinc. Since mercury is many times heavier than an equal volume of water, small particles of impurities weighing less than mercury will float to the surface of the mercury. The removal of these impurities from the zinc prevents local action. The mercury is not readily acted upon by the acid. When the cell is delivering current to a load, the mercury continues to act on the impurities in the zinc. This causes the impurities to leave the surface of the zinc electrode and float to the surface of the mercury. This process greatly increases the storage life of the cell. Q14. Describe three ways to prevent polarization. 015. Describe local action TYPES OF CELLS The development of new and different types of cells in the past decade has been so rapid that it is virtually impossible to have a complete knowledge of all the various types. A few recent developments are the silver-zinc, nickel-zinc, nickel-cadmium, silver-cadmium, organic and inorganic lithium, and mercury cells. PRIMARY DRY CELL The dry cell is the most popular type of primary cell. It is ideal for simple applications where an inexpensive and noncritical source of electricity is all that is needed. The dry cell is not actually dry. The electrolyte is not in a liquid state, but is a moist paste. If it should become totally dry, it would no longer be able to transform chemical energy to electrical energy. Construction of a Dry Cell The construction of a common type of dry cell is shown in figure 2-4. These dry cells are also referred to as Leclanche' cells. The internal parts of the cell are located in a cylindrical zinc container. This zinc container serves as the negative electrode (cathode) of the cell. The container is lined with a nonconducting material, such as blotting paper, to separate the zinc from the paste. A carbon electrode is located in the center, and it serves as the positive terminal (anode) of the cell. The paste is a mixture of several substances such as ammonium chloride, powdered coke, ground carbon, manganese dioxide, zinc chloride, graphite, and water. 2-7 POSITIUE TERMINAL Figure 2-4. — Cutaway view of the general-purpose dry cell. This paste, which is packed in the space between the anode and the blotting paper, also serves to hold the anode rigid in the center of the cell. When the paste is packed in the cell, a small space is left at the top for expansion of the electrolytic paste caused by the depolarization action. The cell is then sealed with a cardboard or plastic seal. Since the zinc container is the cathode, it must be protected with some insulating material to be electrically isolated. Therefore, it is common practice for the manufacturer to enclose the cells in cardboard and metal containers. The dry cell (fig. 2-4) is basically the same as the simple voltaic cell (wet cell) described earlier, as far as its internal chemical action is concerned. The action of the water and the ammonium chloride in the paste, together with the zinc and carbon electrodes, produces the voltage of the cell. Manganese dioxide is added to reduce polarization when current flows and zinc chloride reduces local action when the cell is not being used. A cell that is not being used (sitting on the shelf) will gradually deteriorate because of slow internal chemical changes (local action). This deterioration is usually very slow if cells are properly stored. If unused cells are stored in a cool place, their shelf life will be greatly preserved. Therefore, to minimize deterioration, they should be stored in refrigerated spaces. The blotting paper (paste-coated pulpboard separator) serves two purposes — (1) it keeps the paste from making actual contact with the zinc container and (2) it permits the electrolyte from the paste to filter through to the zinc slowly. The cell is sealed at the top to keep air from entering and drying the electrolyte. Care should be taken to prevent breaking this seal. Q16. What serves as the cathode of a dry cell? Q1 7. Why is a dry cell called a DRY cell? Q18. What does the term "shelf life" mean? 2-8 Mercuric-Oxide Zinc Cell The mercuric-oxide zinc cell (mercury cell) is a primary cell that was developed during World War II. Two important assets of the mercury cell are its ability to produce current for a long period of time and a long shelf life when compared to the dry cell shown in figure 2-4. The mercury cell also has a very stable output voltage and is a power source that can be made in a small physical size. With the birth of the space program and the development of small transceivers and miniaturized equipment, a power source of small size was needed. Such equipment requires a small cell which is capable of delivering maximum electrical energy at a constant discharge voltage. The mercury cell, which is one of the smallest cells, meets these requirements. Present mercury cells are manufactured in three basic types as shown in figure 2-5. The wound- anode type, shown in figure 2-5 view A, has an anode composed of a corrugated zinc strip with a paper absorbent. The zinc is mixed with mercury, and the paper is soaked in the electrolyte which causes it to swell and press against the zinc and make positive contact. This process ensures that the electrolyte makes contact with the anode. DOUBLE CELL TOP outer can WOUND ZINC HNODE HND ABSORBENT WITH ALKALINE ELECTROLYTE INNER CAM CELL SEAL (GROMMET) ADAPTER SLEEUE RETAINING RING BARRIER MERCURIC OXIDE DEPOLARIZER (CATHODE) WOUND ANODE FLAT (A) DOUBLE CELL INNER CAN ADAPTER SLEEUE OUTER CAN CELL SEAL (GROMMET) ZINC ANODE ABSORBENT AND ALKALINE ELECTROLYTE BARRIER MERCURIC OXIDE DEPOLARIZER (CATHODE) FLAT PELLET STRUCTURE (B) DOUBLE CELL TOP OUTER CAN INNER CAN BARRIER NEOPRENE DISK CELL SEAL (GROMMET) MERCURIC OXIDE DEPOLARIZER (CATHODE) ADAPTER SLEEUE ABSORBENT SLEEUE AND ALKALINE ELECTROLYTE ZINC OXIDE ANODE CYLINDRICAL STRUCTURE (C) Figure 2-5. — Mercury cells. In the pressed-powder cells, shown in figure 2-5 views B and C, the zinc powder for the anode is mixed prior to being pressed into shape. The absorbent shown in the figure is paper soaked in the electrolyte. The space between the inner and outer containers provides passage for any gas generated by an improper chemical balance or impurities present within the cell. If the anode and cathode of a cell are connected together without a load, a SHORT CIRCUIT condition exists. Short circuits (shorts) can be very dangerous. They cause excessive heat, pressure, and current flow which may cause serious damage to the cell or be a safety hazard to personnel. 2-9 WARNING Do not short the mercury cell. Shorted mercury cells have exploded with considerable force. Other Types of Cells There are many different types of primary cells. Because of such factors as cost, size, ease of replacement, and voltage or current needs, many types of primary cells have been developed. The following is a brief description of some of the primary cells in use today. The Manganese Dioxide -Alkaline-Zinc Cell is similar to the zinc-carbon cell except for the electrolyte used. This type of cell offers better voltage stability and longer life than the zinc-carbon type. It also has a longer shelf life and can operate over a wide temperature range. The manganese dioxide- alkaline-zinc cell has a voltage of 1.5 volts and is available in a wide range of sizes. This cell is commonly referred to as the alkaline cell. The Magnesium-Manganese Dioxide Cell uses magnesium as the anode material. This allows a higher output capacity over an extended period of time compared to the zinc-carbon cell. This cell produces a voltage of approximately 2 volts. The disadvantage of this type of cell is the production of hydrogen during its operation. The Lithium-Organic Cell and the Lithium-Inorganic Cell are recent developments of a new line of high-energy cells. The main advantages of these types of cells are very high power, operation over a wide temperature range, they are lighter than most cells, and have a remarkably long shelf life of up to 20 years. CAUTION Lithium cells contain toxic materials under pressure. Do not puncture, recharge, short-circuit, expose to excessively high temperatures, or incinerate. Use these batteries/cells only in approved equipment. Do not throw in trash. Q19. Why should a mercury cell NOT be shorted? Q20. What factors should be considered when selecting a primary cell for a power source? SECONDARY WET CELLS Secondary cells are sometimes known as wet cells. There are four basic type of wet cells, the lead- acid, nickel-cadmium, silver-zinc, and silver-cadmium. Lead- Acid Cell The lead-acid cell is the most widely used secondary cell. The previous explanation of the secondary cell describes exactly the manner in which the lead-acid cell provides electrical power. The discharging and charging action presented in electrochemical action describes the lead-acid cell. You should recall that the lead-acid cell has an anode of lead peroxide, a cathode of sponge lead, and the electrolyte is sulfuric acid and water. 2-10 Nickel-Cadmium Cell The nickel-cadmium cell (NIC AD) is far superior to the lead-acid cell. In comparison to lead-acid cells, these cells generally require less maintenance throughout their service life in regard to the adding of electrolyte or water. The major difference between the nickel-cadmium cell and the lead-acid cell is the material used in the cathode, anode, and electrolyte. In the nickel-cadmium cell the cathode is cadmium hydroxide, the anode is nickel hydroxide, and the electrolyte is potassium hydroxide and water. The nickel-cadmium and lead-acid cells have capacities that are comparable at normal discharge rates, but at high discharge rates the nickel-cadmium cell can deliver a larger amount of power. In addition the nickel-cadmium cell can: 1 . Be charged in a shorter time, 2. Stay idle longer in any state of charge and keep a full charge when stored for a longer period of time, and 3. Be charged and discharged any number of times without any appreciable damage. Due to their superior capabilities, nickel-cadmium cells are being used extensively in many military applications that require a cell with a high discharge rate. A good example is in the aircraft storage battery. Silver-Zinc Cells The silver-zinc cell is used extensively to power emergency equipment. This type of cell is relatively expensive and can be charged and discharged fewer times than other types of cells. When compared to the lead-acid or nickel-cadmium cells, these disadvantages are overweighed by the light weight, small size, and good electrical capacity of the silver-zinc cell. The silver-zinc cell uses the same electrolyte as the nickel-cadmium cell (potassium hydroxide and water), but the anode and cathode differ from the nickel-cadmium cell. The anode is composed of silver oxide and the cathode is made of zinc. Silver-Cadmium Cell The silver-cadmium cell is a fairly recent development for use in storage batteries. The silver- cadmium cell combines some of the better features of the nickel-cadmium and silver-zinc cells. It has more than twice the shelf life of the silver-zinc cell and can be recharged many more times. The disadvantages of the silver-cadmium cell are high cost and low voltage production. The electrolyte of the silver-cadmium cell is potassium hydroxide and water as in the nickel- cadmium and silver-zinc cells. The anode is silver oxide as in the silver-zinc cell and the cathode is cadmium hydroxide as in the nicad cell. You may notice that different combinations of materials are used to form the electrolyte, cathode, and anode of different cells. These combinations provide the cells with different qualities for many varied applications. Q21. What are the four basic types of secondary (wet) cells? Q22. What are the advantages of a nicad cell over a lead-acid cell? Q23. What type of cell is most commonly used for emergency systems? Q24. What three cells use the same electrolyte? 2-11 BATTERIES A battery is a voltage source that uses chemical action to produce a voltage. In many cases the term battery is applied to a single cell, such as the flashlight battery. In the case of a flashlight that uses a battery of 1.5 volts, the battery is a single cell. The flashlight that is operated by 6 volts uses four cells in a single case and this is a battery composed of more than one cell. There are three ways to combine cells to form a battery. COMBINING CELLS In many cases, a battery-powered device may require more electrical energy than one cell can provide. The device may require either a higher voltage or more current, and in some cases both. Under such conditions it is necessary to combine, or interconnect, a sufficient number of cells to meet the higher requirements. Cells connected in SERIES provide a higher voltage, while cells connected in PARALLEL provide a higher current capacity. To provide adequate power when both voltage and current requirements are greater than the capacity of one cell, a combination SERIES -PARALLEL network of cells must be used. Series-Connected Cells Assume that a load requires a power supply of 6 volts and a current capacity of 1/8 ampere. Since a single cell normally supplies a voltage of only 1.5 volts, more than one cell is needed. To obtain the higher voltage, the cells are connected in series as shown in figure 2-6. 6 UOLTS (B) Figure 2-6. — (A) Pictorial view of series-connected cells; (B) Schematic of series connection. Figure 2-6 view B is a schematic representation of the circuit shown in figure 2-6 view A. The load is shown by the resistance symbol and the battery is indicated by one long and one short line per cell. In a series hookup, the negative electrode (cathode) of the first cell is connected to the positive electrode (anode) of the second cell, the negative electrode of the second to the positive of the third, etc. 2-12 The positive electrode of the first cell and negative electrode of the last cell then serve as the terminals of the battery. In this way, the voltage is 1.5 volts for each cell in the series line. There are four cells, so the output terminal voltage is 1.5 x 4, or 6 volts. When connected to the load, 1/8 ampere flows through the load and each cell of the battery. This is within the capacity of each cell. Therefore, only four series-connected cells are needed to supply this particular load. CAUTION When connecting cells in series, connect alternate terminals together (- to +, - to +, etc.) Always have two remaining terminals that are used for connection to the load only. Do not connect the two remaining terminals together as this is a short across the battery and would not only quickly discharge the cells but could cause some types of cells to explode. Parallel-Connected Cells In this case, assume an electrical load requires only 1.5 volts, but will require 1/2 ampere of current. (Assume that a single cell will supply only 1/8 ampere.) To meet this requirement, the cells are connected in parallel, as shown in figure 2-7 view A and schematically represented in 2-7 view B. In a parallel connection, all positive cell electrodes are connected to one line, and all negative electrodes are connected to the other. No more than one cell is connected between the lines at any one point; so the voltage between the lines is the same as that of one cell, or 1.5 volts. However, each cell may contribute its maximum allowable current of 1/8 ampere to the line. There are four cells, so the total line current is 1/8 x 4, or 1/2 ampere. In this case four cells in parallel have enough capacity to supply a load requiring 1/2 ampere at 1.5 volts. LAMP (B) Figure 2-7. — (A) Pictorial view of parallel-connected cells; (B) Schematic of parallel connection. 2-13 Series-Parallel-Connected Cells Figure 2-8 depicts a battery network supplying power to a load requiring both a voltage and a current greater than one cell can provide. To provide the required 4.5 volts, groups of three 1.5-volt cells are connected in series. To provide the required 1/2 ampere of current, four series groups are connected in parallel, each supplying 1/8 ampere of current. ^ AMP VOLTS 4 ± VOLTS LOAD Figure 2-8. — Schematic of series-parallel connected cells. The connections shown have been used to illustrate the various methods of combining cells to form a battery. Series, parallel, and series-parallel circuits will be covered in detail in the next chapter, "Direct Current." Some batteries are made from primary cells. When a primary-cell battery is completely discharged, the entire battery must be replaced. Because there is nothing else that can be done to primary cell batteries, the rest of the discussion on batteries will be concerned with batteries made of secondary cells. Q25. What does the term battery normally refer to? Q26. What are the three ways of combining cells , and what is each used for? BATTERY CONSTRUCTION Secondary cell batteries are constructed using the various secondary cells already described. The lead-acid battery is one of the most common batteries in use today and will be used to explain battery construction. The nickel-cadmium battery is being used with increasing frequency and will also be discussed. Figure 2-9 shows the makeup of a lead-acid battery. The container houses the separate cells. Most containers are hard rubber, plastic, or some other material that is resistant to the electrolyte and mechanical shock and will withstand extreme temperatures. The container (battery case) is vented through vent plugs to allow the gases that form within the cells to escape. The plates in the battery are the cathodes and anodes that were discussed earlier. In figure 2-10 the negative plate group is the cathode of the individual cells and the positive plate group is the anode. As shown in the figure, the plates are interlaced with a terminal attached to each plate group. The terminals of the individual cells are connected together by link connectors as shown in figure 2-9. The cells are connected in series in the battery and the 2-14 positive terminal of one end cell becomes the positive terminal of the battery. The negative terminal of the opposite end cell becomes the negative terminal of the battery. FILLER OPENING IN CELL COUER PLATE STRAP CONTAINER NEGATIUE PLATE SEPARATOR RIB UENT PLUG TERMINAL POST TERMINAL CONNECTOR LINK CONNECTOR POSITIUE PLATE SEDIMENT SPACE Figure 2-9. — Lead-acid battery construction, POSITIVE PLATE GROU SEPARATOR NEGATIVE PLATE GROUP CELL ELEMENT PARTLY ASSEMBLED Figure 2-10. — Lead-acid battery plate arrangement. The terminals of a lead-acid battery are usually identified from one another by their size and markings. The positive terminal, marked (+) is sometimes colored red and is physically larger than the negative terminal, marked (-). 2-15 The individual cells of the lead-acid battery are not replaceable, so in the event one cell fails the battery must be replaced. The nickel-cadmium battery is similar in construction to the lead-acid battery with the exception that it has individual cells which can be replaced. The cell of the nicad battery is shown in figure 2-11. VENT AND FILLER CAP Figure 2-11. — Nickel-cadmium cell. The construction of secondary cell batteries is so similar, that it is difficult to distinguish the type of battery by simply looking at it. The type of battery must be known to properly check or recharge the battery. Each battery should have a nameplate that gives a description of its type and electrical characteristics. Q27. Other than the type of cell used, what is the major difference between the construction of the lead- acid and nicad battery? Q28. How is the type of battery most easily determined? BATTERY MAINTENANCE The following information concerns the maintenance of secondary-cell batteries and is of a general nature. You must check the appropriate technical manuals for the specific type of battery prior to performing maintenance on any battery . Specific Gravity For a battery to work properly, its electrolyte (water plus active ingredient) must contain a certain amount of active ingredient. Since the active ingredient is dissolved in the water, the amount of active ingredient cannot be measured directly. An indirect way to determine whether or not the electrolyte contains the proper amount of active ingredient is to measure the electrolyte's specific gravity. Specific gravity is the ratio of the weight of a certain amount of a given substance compared to the weight of the same amount of pure water. The specific gravity of pure water is 1.0. Any substance that floats has a specific gravity less than 1.0. Any substance that sinks has a specific gravity greater than 1.0. 2-16 The active ingredient in electrolyte (sulfuric acid, potassium hydroxide, etc.) is heavier than water. Therefore, the electrolyte has a specific gravity greater than 1.0. The acceptable range of specific gravity for a given battery is provided by the battery's manufacturer. To measure a battery's specific gravity, use an instrument called a HYDROMETER. The Hydrometer A hydrometer, shown in figure 2-12, is a glass syringe with a float inside it. The float is a hollow glass tube sealed at both ends and weighted at the bottom end, with a scale calibrated in specific gravity marked on its side. To test an electrolyte, draw it into the hydrometer using the suction bulb. Draw enough electrolyte into the hydrometer to make the float rise. Do not draw in so much electrolyte that the float rises into the suction bulb. The float will rise to a point determined by the specific gravity of the electrolyte. If the electrolyte contains a large amount of active ingredient, its specific gravity will be relatively high. The float will rise higher than it would if the electrolyte contained only a small amount of active ingredient. Figure 2-12. — Hydrometer. To read the hydrometer, hold it in a vertical position and read the scale at the point that surface of the electrolyte touches the float. Refer to the manufacturer's technical manual to determine whether or not the battery's specific gravity is within specifications. Note: Hydrometers should be flushed with fresh water after each use to prevent inaccurate readings. Storage battery hydrometers must not be used for any other purpose. Q29. What is the purpose of the hydrometer? 2-17 Q30. Which electrolyte has more active ingredient? Electrolyte A, specific gravity 1.015? Electrolyte B, specific gravity 1. 125? Other Maintenance The routine maintenance of a battery is very simple. Terminals should be checked periodically for cleanliness and good electrical connection. The battery case should be inspected for cleanliness and evidence of damage. The level of electrolyte should be checked and if the electrolyte is low, distilled water should be added to bring the electrolyte to the proper level. Maintenance procedures for batteries are normally determined by higher authority and each command will have detailed procedures for battery care and maintenance. Safety Precautions With Batteries All types of batteries should be handled with care: 1 . NEVER SHORT THE TERMINALS OF A BATTERY. 2. CARRYING STRAPS SHOULD BE USED WHEN TRANSPORTING BATTERIES. 3. PROTECTIVE CLOTHING, SUCH AS RUBBER APRON, RUBBER GLOVES, AND A FACE SHIELD SHOULD BE WORN WHEN WORKING WITH BATTERIES. 4. NO SMOKING, ELECTRIC SPARKS, OR OPEN FLAMES SHOULD BE PERMITTED NEAR CHARGING BATTERIES. 5. CARE SHOULD BE TAKEN TO PREVENT SPILLING OF THE ELECTROLYTE. In the event electrolyte is splashed or spilled on a surface, such as the floor or table, it should be diluted with large quantities of water and cleaned up immediately. If the electrolyte is spilled or splashed on the skin or eyes, IMMEDIATELY flush the skin or eyes with large quantities of fresh water for a minimum of 15 minutes. If the electrolyte is in the eyes, be sure the upper and lower eyelids are pulled out sufficiently to allow the fresh water to flush under the eyelids. The medical department should be notified as soon as possible and informed of the type of electrolyte and the location of the accident. CAPACITY AND RATING OF BATTERIES The CAPACITY of a battery is measured in ampere-hours. The ampere-hour capacity is equal to the product of the current in amperes and the time in hours during which the battery will supply this current. The ampere-hour capacity varies inversely with the discharge current. For example, a 400 ampere-hour battery will deliver 400 amperes for 1 hour or 100 amperes for 4 hours. Storage batteries are RATED according to their rate of discharge and ampere-hour capacity. Most batteries are rated according to a 20-hour rate of discharge. That is, if a fully charged battery is completely discharged during a 20-hour period, it is discharged at the 20-hour rate. Thus, if a battery can deliver 20 amperes continuously for 20 hours, the battery has a rating of 20 amperes x 20 hours, or 400 ampere-hours. Therefore, the 20-hour rating is equal to the average current that a battery is capable of supplying without interruption for an interval of 20 hours. (Note: Aircraft batteries are rated according to a 1-hour rate of discharge.) 2-18 All standard batteries deliver 100 percent of their available capacity if discharged in 20 hours or more, but they will deliver less than their available capacity if discharged at a faster rate. The faster they discharge, the less ampere-hour capacity they have. The low-voltage limit, as specified by the manufacturer, is the limit beyond which very little useful energy can be obtained from a battery. This low-voltage limit is normally a test used in battery shops to determine the condition of a battery. Q31. When should safety precautions pertaining to batteries be observed? Q32. How long should a 200 ampere-hour battery be able to deliver 5 amperes? BATTERY CHARGING It should be remembered that adding the active ingredient to the electrolyte of a discharged battery does not recharge the battery. Adding the active ingredient only increases the specific gravity of the electrolyte and does not convert the plates back to active material, and so does not bring the battery back to a charged condition. A charging current must be passed through the battery to recharge it. Batteries are usually charged in battery shops. Each shop will have specific charging procedures for the types of batteries to be charged. The following discussion will introduce you to the types of battery charges. The following types of charges may be given to a storage battery, depending upon the condition of the battery: 1 . Initial charge 2. Normal charge 3 . Equalizing charge 4. Floating charge 5. Fast charge Initial Charge When a new battery is shipped dry, the plates are in an uncharged condition. After the electrolyte has been added, it is necessary to charge the battery. This is accomplished by giving the battery a long low- rate initial charge. The charge is given in accordance with the manufacturer's instructions, which are shipped with each battery. If the manufacturer's instructions are not available, reference should be made to the detailed instructions for charging batteries found in current Navy directives. Normal Charge A normal charge is a routine charge that is given in accordance with the nameplate data during the ordinary cycle of operation to restore the battery to its charged condition. Equalizing Charge An equalizing charge is a special extended normal charge that is given periodically to batteries as part of a maintenance routine. It ensures that all the sulfate is driven from the plates and that all the cells 2-19 are restored to a maximum specific gravity. The equalizing charge is continued until the specific gravity of all cells, corrected for temperature, shows no change for a 4-hour period. Floating Charge In a floating charge, the charging rate is determined by the battery voltage rather than by a definite current value. The floating charge is used to keep a battery at full charge while the battery is idle or in light duty. It is sometimes referred to as a trickle charge and is accomplished with low current. Fast Charge A fast charge is used when a battery must be recharged in the shortest possible time. The charge starts at a much higher rate than is normally used for charging. It should be used only in an emergency, as this type charge may be harmful to the battery. Charging Rate Normally, the charging rate of Navy storage batteries is given on the battery nameplate. If the available charging equipment does not have the desired charging rates, the nearest available rates should be used. However, the rate should never be so high that violent gassing (explained later in this text) occurs. Charging Time The charge must be continued until the battery is fully charged. Frequent readings of specific gravity should be taken during the charge and compared with the reading taken before the battery was placed on charge. Gassing When a battery is being charged, a portion of the energy breaks down the water in the electrolyte. Hydrogen is released at the negative plates and oxygen at the positive plates. These gases bubble up through the electrolyte and collect in the air space at the top of the cell. If violent gassing occurs when the battery is first placed on charge, the charging rate is too high. If the rate is not too high, steady gassing develops as the charging proceeds, indicating that the battery is nearing a fully charged condition. WARNING A mixture of hydrogen and air can be dangerously explosive. No smoking, electric sparks, or open flames should be permitted near charging batteries. Q33. Can a battery be recharged by adding more electrolyte? Q34. If violent gassing occurs during a battery charge, what action should be taken? SUMMARY In this chapter you learned that batteries are widely used as sources of direct-current. You were introduced to electrochemical action and the way it works in a cell, the cell itself, the type and parts of a cell, and how cells are connected together to form batteries. You learned the construction and maintenance of batteries and some of the safety precautions in handling and working with batteries. 2-20 Several new terms were introduced in this chapter. The following is a summary of the chapter on batteries. A CELL is a device that transforms chemical energy into electrical energy. The cell has three parts; the electrodes, the electrolyte, and the container. There are two basic cells: primary and secondary. ZINC (Zn) ELECTRON zzzzzz zzzzzz CARBON (C) THE ELECTRODES are the current conductors of the cell. THE ELECTROLYTE is the solution that acts upon the electrodes. THE CONTAINER holds the electrolyte and provides a means of mounting the electrodes. THE PRIMARY CELL is a cell in which the chemical action finally destroys one of the electrodes, usually the negative. The primary cell cannot be recharged. THE SECONDARY CELL is a cell in which the chemical action alters the electrodes and electrolyte. The electrodes and electrolyte can be restored to their original condition by recharging the cell. 2-21 SOLUTION « 4 I*- V‘, SULFURIC fiCID H 2 S0 4 fiND UlfiTER H 2 0 SPONGE LEfiD Pb (fi) CHARGED LEfiD PEROXIDE Pb0 o SOLUTION * * I SULFURIC fiCID DECREfiSING UlfiTER INCREASING m (B) DISCHARGING DECREflS I NG DECREfiS I NG SPONGE LEAD INCREASING LEfiD SULFATE LEAD PEROXIDE INCREASING LEfiD SULFfiTE Pb + PbS0 Pb0 2 + PbS@ 4 - SOLUTION SULFURIC ACID INCREASING UlfiTER DECREASING (D) CHfiRGING INCREASING SPONGE LEfiD DECREfiSING LEfiD SULFATE PbS0 4 + Pb INCREASING LEfiD PEROXIDE DECREfiSING LEfiD SULFfiTE PbS0 4 + Pb0 2 1 * SOLUTION MINI MUM SULFURIC fiCID MAXIMUM WATER HW (C) DISCHARGED MINIMUM SPONGE LEfiD MAXIMUM LEfiD SULFfiTE PbS0 1 + Pb MINIMUM LEfiD PEROXIDE MAXIMUM LEfiD SULFfiTE PbS0j + Pb0 2 Ey; 1 ;':! SPONGE LEfiD LEfiD PEROXIDE LEfiD SULFfiTE ELECTROCHEMICAL ACTION is the process of converting chemical energy into electrical energy. THE ANODE is the positive electrode of a cell. THE CATHODE is the negative electrode of a cell. PRIMARY CELL CHEMISTRY is the process in which electrons leaving the cathode to the load cause a positive charge which attracts negative ions from the electrolyte. The negative ions combine with the material of the cathode and form a substance such as lead-sulfate. Electrons from the load to the anode create a negative charge which attracts positive ions (hydrogen) from the electrolyte. 2-22 NEGATIVE TERMINAL (CATHODE) POSITIVE TERMINAL (ANODE) SEALING WAX ZINC CONTAINER AND NEGATIVE ELECTRODE WET PASTE ELECTROLYTE CARBON ROD, OR posftive ELECTRODE SECONDARY CELL CHEMISTRY is the process in which the electrolyte acts upon and chemically changes both electrodes. This process also depletes the amount of active material in the electrolyte. A charging current applied to the cell reverses the process and restores the cell to its original condition. POLARIZATION is the effect of hydrogen surrounding the anode of a cell which increases the internal resistance of the cell. Polarization can be prevented by venting the cell, adding a material rich in oxygen, or adding a material that will absorb hydrogen. LOCAL ACTION is the continuation of current flow within the cell when there is no external load. It is caused by impurities in the electrode and can be prevented by the use of mercury amalgamated with the material of the electrode. DRY CELL is the type commonly referred to as the "flashlight battery." Since the electrolyte is not in liquid form, but is a paste, the term dry cell is used. In most dry cells the case is the cathode. POSITIUE TERMIMGL 2-23 SHELF LIFE is the period the cell may be stored and still be usable. MERCURY CELLS should never be shorted because of the danger of explosion. DOUBLE CELL TOP outer can WOUND ZINC ANODE AND ABSORBENT WITH ALKALINE ELECTROLYTE INNER CAN CELL SEAL (GROMMET) ADAPTER SLEEUE RETAINING RING BARRIER MERCURIC OK IDE DEPOLARIZER (CATHODE) WOUND ANODE FLAT DRY CELLS are of many types, each having advantages and disadvantages. The type selected for use depends on such factors as cost, size, ease of replacement, and voltage or current needs. THE LEAD-ACID CELL is the most widely used secondary cell. The lead-acid cell produces electricity by electrochemical action. The anode is lead peroxide, the cathode is sponge lead, and the electrolyte is sulfuric acid and water. POSITIVE PLATE GROU SEPARATOR NEGATIVE PLATE GROUP CELL ELEMENT PARTLY ASSEMBLED THE NICKEL-CADMIUM CELL , commonly called the NIC AD, has the following advantages over the lead-acid cell; charges in a shorter period of time, delivers a larger amount of power, stays idle longer, and can be charged and discharged many times. The anode is nickel hydroxide, the cathode is cadmium hydroxide, and the electrolyte is potassium hydroxide and water. 2-24 VENT AND FILLER CAP TERMINAL TERMINAL SEAL CONTACT TAB CASE SEPARATOR PLATE THE SILVER-ZINC CELL is used mostly for emergency equipment. It is light, small, and has a large power capacity for its size. The anode is silver oxide, the cathode is zinc, and the electrolyte is potassium hydroxide and water. THE SILVER-CADMIUM CELL combines the better features of the nickel-cadmium and silver- zinc cells. The anode is silver-oxide, the cathode is cadmium hydroxide, and the electrolyte is potassium hydroxide. A BATTERY is a voltage source in a single container made from one or more cells. The cells can be combined in series, parallel, or series-parallel. FILLER OPENING IN CELL COUER PLATE STRAP CONTAINER NEGATIUE PLATE SEPARATOR RIB CENT PLUG TERMINAL POST TERMINAL CONNECTOR LINK CONNECTOR POSITIUE PLATE SEDIMENT SPACE 2-25 SERIES CONNECTED CELLS provide a higher voltage than a single cell, with no increase in current. + 8 vVWNA LOAD AMP 6 VOLTS PARALLEL CONNECTED CELLS provide a higher current than a single cell, with no increase in voltage. 1/2 AMP-r-= 1 1/2 VOLTS LOAD SERIES-PARALLEL CONNECTED CELLS provide a higher voltage and a higher current than a single cell. VOLTS 4 ± VOLTS LOAD TYPES OF BATTERIES can be determined from nameplate data. HYDROMETER provides the means to check the specific gravity of the electrolyte. 2-26 SAFETY PRECAUTIONS should always be followed when working with or around batteries. CAPACITY is an indication of the current-supplying capability of the battery for a specific period of time; e.g., 400 ampere-hour. RATING is the capacity of the battery for a specific rate of discharge. In most batteries the rating is given for a 20 hour discharge cycle; e.g., 20 amperes for 20 hours. BATTERY CHARGE is the process of reversing the current flow through the battery to restore the battery to its original condition. The addition of active ingredient to the electrolyte will not recharge the battery. There are five types of charges: 1 . Initial charge 2. Normal charge 3 . Equalizing charge 4. Floating charge 5. Fast charge GASSING is the production of hydrogen gas caused by a portion of the charge current breaking down the water in the electrolyte. Steady gassing is normal during the charging process. Violent gassing indicates that the charge rate is too high. 2-27 ANSWERS TO QUESTIONS Ql. THROUGH Q34. ALA cell is a device that converts chemical energy to electrical energy. A2.The electrodes, the electrolyte, and the container. A3. The electrodes are the current conductors of the cell. The electrolyte is the solution that acts upon the electrodes. The container holds the electrolyte and provides a means of mounting the electrodes. A4. Primary and secondary. A5. The secondary cell can be restored to its original condition by an electric current. The primary cell cannot. A6.The process of converting chemical energy into electrical energy. A7.(a) The anode, (b) the cathode. A8.The positive charge caused by electrons leaving the negative electrode attracts the negative ions. A9.By current flow through the load. A 10. The chemical action between the negative electrode and the electrolyte. All. The sulfuric acid is chemically acting upon the anode and cathode which creates a current flow through the load. A12.The charging currents causes the lead sulfate in the anode and cathode to be changed back to lead peroxide, sponge lead, and sulfuric acid. A1 3. Fully charged. A14. Vent the cell, add a material rich in oxygen, and use a material that will absorb hydrogen. A1 5. Current flow in a cell with no external load. A 16. The zinc container. A 17. The electrolyte is not a liquid but is in the form of a paste. A 18. The period that a cell can be stored and still be useable. A 19. The danger of explosion. A20.Cost, size, ease of replacement, and voltage or current needs. A21. Lead-acid, nickel-cadmium (NICAD), silver-zinc, and silver-cadmium. A22.Can be charged in a shorter time, can deliver a larger amount of power, and stays idle longer. A2 3. Silver-zinc cell. A2 4. Silver-cadmium, silver-zinc, and nickel-cadmium. A25.A voltage source in a single container made from one or more cells. 2-28 A26. Series, to increase voltage but not current. Parallel, to increase current but not voltage. Series- Parallel, to increase both current and voltage. A27.The cells in the nicad battery can be replaced. A28.By looking at the nameplate data. A29. To measure the amount of active ingredient in the electrolyte. A3 0. Electrolyte B. It is heavier per unit volume. A31.At all times. A 3 2. Forty hours. A33.No, a current must be passed through the battery. A34. Reduce the charging rate. 2-29 CHAPTER 3 DIRECT CURRENT LEARNING OBJECTIVES Upon completing this chapter, you will be able to: 1 . Identify the term schematic diagram and identify the components in a circuit from a simple schematic diagram. 2. State the equation for Ohm’s law and describe the effects on current caused by changes in a circuit. 3. Given simple graphs of current versus power and voltage versus power, determine the value of circuit power for a given current and voltage. 4. Identify the term power, and state three formulas for computing power. 5. Compute circuit and component power in series, parallel, and combination circuits. 6. Compute the efficiency of an electrical device. 7. Solve for unknown quantities of resistance, current, and voltage in a series circuit. 8. Describe how voltage polarities are assigned to the voltage drops across resistors when Kirchhoff’s voltage law is used. 9. State the voltage at the reference point in a circuit. 10. Define open and short circuits and describe their effects on a circuit. 11. State the meaning of the term source resistance and describe its effect on a circuit. 12. Describe in terms of circuit values the circuit condition needed for maximum power transfer. 13. Compute efficiency of power transfer in a circuit. 14. Solve for unknown quantities of resistance, current, and voltage in a parallel circuit. 15. State the significance of the polarity assigned to a current when using Kirchhoff’s current law. 16. State the meaning of the term equivalent resistance. 17. Compute resistance, current, voltage, and power in voltage dividers. 18. Describe the method by which a single voltage divider can provide both positive and negative voltages. 19. Recognize the safety precautions associated with the hazard of electrical shock. 20. Identify the first aid procedures for a victim of electrical shock. 3-1 INTRODUCTION The material covered in this chapter contains many new terms that are explained as you progress through the material. The basic dc circuit is the easiest to understand, so the chapter begins with the basic circuit and from there works into the basic schematic diagram of that circuit. The schematic diagram is used in all your future work in electricity and electronics. It is very important that you become familiar with the symbols that are used. This chapter also explains how to determine the total resistance, current, voltage, and power in a series, parallel, or combination circuit through the use of Ohm’s and Kirchhoff’s laws. The voltage divider network, series, parallel, and series-parallel practice problem circuits will be used for practical examples of what you have learned. THE BASIC ELECTRIC CIRCUIT The flashlight is an example of a basic electric circuit. It contains a source of electrical energy (the dry cells in the flashlight), a load (the bulb) which changes the electrical energy into a more useful form of energy (light), and a switch to control the energy delivered to the load. Before you study a schematic representation of the flashlight, it is necessary to define certain terms. The LOAD is any device through which an electrical current flows and which changes this electrical energy into a more useful form. Some common examples of loads are a lightbulb, which changes electrical energy to light energy; an electric motor, which changes electrical energy into mechanical energy; and the speaker in a radio, which changes electrical energy into sound. The SOURCE is the device which furnishes the electrical energy used by the load. It may consist of a simple dry cell (as in a flashlight), a storage battery (as in an automobile), or a power supply (such as a battery charger). The SWITCH, which permits control of the electrical device, interrupts the current delivered to the load. SCHEMATIC REPRESENTATION The technician’s main aid in troubleshooting a circuit in a piece of equipment is the SCHEMATIC DIAGRAM. The schematic diagram is a "picture" of the circuit that uses symbols to represent the various circuit components; physically large or complex circuits can be shown on a relatively small diagram. Before studying the basic schematic, look at figure 3-1. This figure shows the symbols that are used in this chapter. These, and others like them, are referred to and used throughout the study of electricity and electronics. 3-2 The schematic in figure 3-2 represents a flashlight. View A of the figure shows the flashlight in the off or deenergized state. The switch (SI) is open. There is no complete path for current (I) through the circuit, and the bulb (DS1) does not light. In figure 3-2 view B, switch SI is closed. Current flows in the direction of the arrows from the negative terminal of the battery (BAT), through the switch (SI), through the lamp (DS1), and back to the positive terminal of the battery. With the switch closed the path for current is complete. Current will continue to flow until the switch (SI) is moved to the open position or the battery is completely discharged. 3-3 SI (A) DEENERGIZED (B) ENERGIZED Figure 3-2. — Basic flashlight schematic. Ql. In figure 3-2, what part of the circuit is the (a) load and (b) source? Q2. What happens to the path for current when SI is open as shown in figure 3 -2(A)? Q3. What is the name given to the "picture" of a circuit such as the one shown in figure 3-2? OHM’S LAW In the early part of the 19th century, George Simon Ohm proved by experiment that a precise relationship exists between current, voltage, and resistance. This relationship is called Ohm’s law and is stated as follows: The current in a circuit is DIRECTLY proportional to the applied voltage and INVERSELY proportional to the circuit resistance. Ohm’s law may be expressed as an equation: R Uhere: I = current in amperes E = voltage in volts R = resistance in ohms As stated in Ohm’s law, current is inversely proportional to resistance. This means, as the resistance in a circuit increases, the current decreases proportionately . 3-4 In the equation R if any two quantities are known, the third one can be determined. Refer to figure 3-2(B), the schematic of the flashlight. If the battery (BAT) supplies a voltage of 1.5 volts and the lamp (DS1) has a resistance of 5 ohms, then the current in the circuit can be determined. Using this equation and substituting values: , E 15 volts ^ I = — = — =.3 ampere R 5 ohms If the flashlight were a two-cell flashlight, we would have twice the voltage, or 3.0 volts, applied to the circuit. Using this voltage in the equation: , E 3.0 volts „ 1 = — = — =.6 ampere R 5 ohms You can see that the current has doubled as the voltage has doubled. This demonstrates that the current is directly proportional to the applied voltage. If the value of resistance of the lamp is doubled, the equation will be: 3.0 volts 1 0 ohms =.3 ampere The current has been reduced to one half of the value of the previous equation, or .3 ampere. This demonstrates that the current is inversely proportional to the resistance . Doubling the value of the resistance of the load reduces circuit current value to one half of its former value. APPLICATION OF OHM’S LAW By using Ohm’s law, you are able to find the resistance of a circuit, knowing only the voltage and the current in the circuit. In any equation, if all the variables (parameters) are known except one, that unknown can be found. For example, using Ohm’s law, if current (I) and voltage (E) are known, resistance (R) the only parameter not known, can be determined: 1. Basic formula: 2. Remove the divisor by multiplying both sides by R: 3-5 3. Result of step 2: R x I = E 4. To get R alone (on one side of the equation) divide both sides by I: R/ _ E X I 5. The basic formula, transposed for R, is: Refer to figure 3-3 where E equals 10 volts and I equals 1 ampere. Solve for R, using the equation just explained. Given: E = 10 volts 1=1 ampere Solution: Figure 3-3. — Determining resistance in a basic circuit. 3-6 Insert the values of the known quantities: 10 volts R — 1 ampere R = 10 ohms The basic formula can also be used to solve for E: E Take the basic formula: I = — R multiply both sides by R: I 1 Results: E = IxR This equation can be used to find the voltage for the circuit shown in figure 3-4. Given: I = .5 ampere R = AS ohms Solution: E = I x R E = .5 ampere x AS ohms E = 22.5 volts Figure 3-4. — Determining voltage in a basic circuit. 3-7 The Ohm’s law equation and its various forms may be obtained readily with the aid of figure 3-5. The circle containing E, I, and R is divided into two parts, with E above the line and with I and R below the line. To determine the unknown quantity, first cover that quantity with a finger. The position of the uncovered letters in the circle will indicate the mathematical operation to be performed. For example, to find I, cover I with a finger. The uncovered letters indicate that E is to be divided by R, or E R To find the formula for E, cover E with your finger. The result indicates that I is to be multiplied by R, or E = IR. To find the formula for R, cover R. The result indicates that E is to be divided by I, or You are cautioned not to rely wholly on the use of this diagram when you transpose the Ohm’s law formulas. The diagram should be used to supplement your knowledge of the algebraic method. Algebra is a basic tool in the solution of electrical problems. Q4. According to Ohm's law, what happens to circuit current if the applied voltage (a) increases, (b) decreases? Q5. According to Ohm's law, what happens to circuit current if circuit resistance (a) increases, (b) decreases? Q6. What is the equation used to find circuit resistance if voltage and current values are known? GRAPHICAL ANALYSIS OF THE BASIC CIRCUIT One of the most valuable methods of analyzing a circuit is by constructing a graph. No other method provides a more convenient or more rapid way to observe the characteristics of an electrical device. 3-8 The first step in constructing a graph is to obtain a table of data. The information in the table can be obtained by taking measurements on the circuit under examination, or can be obtained theoretically through a series of Ohm’s law computations. The latter method is used here. Since there are three variables (E, I, and R) to be analyzed, there are three distinct graphs that may be constructed. To construct any graph of electrical quantities, it is standard practice to vary one quantity in a specified way and note the changes which occur in a second quantity. The quantity which is intentionally varied is called the independent variable and is plotted on the horizontal axis. The horizontal axis is known as the X-AXIS. The second quantity, which varies as a result of changes in the first quantity, is called the dependent variable and is plotted on the vertical, or Y-AXIS. Any other quantities involved are held constant. For example, in the circuit shown in figure 3-6, if the resistance was held at 10 ohms and the voltage was varied, the resulting changes in current could then be graphed. The resistance is the constant, the voltage is the independent variable, and the current is the dependent variable. Figure 3-6. — Three variables in a basic circuit. Figure 3-7 shows the graph and a table of values. This table shows R held constant at 10 ohms as E is varied from 0 to 20 volts in 5-volt steps. Through the use of Ohm’s law, you can calculate the value of current for each value of voltage shown in the table. When the table is complete, the information it contains can be used to construct the graph shown in figure 3-7. For example, when the voltage applied to the 10-ohm resistor is 10 volts, the current is 1 ampere. These values of current and voltage determine a point on the graph. When all five points have been plotted, a smooth curve is drawn through the points. 3-9 Figure 3-7. — Volt-ampere characteristic. Through the use of this curve, the value of current through the resistor can be quickly determined for any value of voltage between 0 and 20 volts. Since the curve is a straight line, it shows that equal changes of voltage across the resistor produce equal changes in current through the resistor. This fact illustrates an important characteristic of the basic law — the current varies directly with the applied voltage when the resistance is held constant. When the voltage across a load is held constant, the current depends solely upon the resistance of the load. For example, figure 3-8 shows a graph with the voltage held constant at 12 volts. The independent variable is the resistance which is varied from 2 ohms to 12 ohms. The current is the dependent variable. Values for current can be calculated as: AMPERES OHMS (R) Figure 3-8. — Relationship between current and resistance. 3-10 Given: E = 12 volts R = 2 ohms to 12 ohms E Soultion: I = — R 12 volts I = = 1 ampere 12 ohms 12 volts I = = 12 ampere 10 ohms 12 volts „ _ I = — = 15 ampere 8 ohms 12 volts ^ I = = 2 ampere 6 ohms This process can be continued for any value of resistance. You can see that as the resistance is halved, the current is doubled; when the resistance is doubled, the current is halved. This illustrates another important characteristic of Ohm's law — current varies inversely with resistance when the applied voltage is held constant. Q7. Using the graph in figure 3-7, what is the approximate value of current when the voltage is 12.5 volts? Q8. Using the graph in figure 3-8, what is the approximate value of current when the resistance is 3 ohms ? POWER Power, whether electrical or mechanical, pertains to the rate at which work is being done. Work is done whenever a force causes motion. When a mechanical force is used to lift or move a weight, work is done. However, force exerted WITHOUT causing motion, such as the force of a compressed spring acting between two fixed objects, does not constitute work. Previously, it was shown that voltage is an electrical force, and that voltage forces current to flow in a closed circuit. However, when voltage exists but current does not flow because the circuit is open, no work is done. This is similar to the spring under tension that produced no motion. When voltage causes electrons to move, work is done. The instantaneous RATE at which this work is done is called the electric power rate, and is measured in WATTS. A total amount of work may be done in different lengths of time. For example, a given number of electrons may be moved from one point to another in 1 second or in 1 hour, depending on the RATE at which they are moved. In both cases, total work done is the same. However, when the work is done in a 3-11 short time, the wattage, or INSTANTANEOUS POWER RATE, is greater than when the same amount of work is done over a longer period of time. As stated, the basic unit of power is the watt. Power in watts is equal to the voltage across a circuit multiplied by current through the circuit. This represents the rate at any given instant at which work is being done. The symbol P indicates electrical power. Thus, the basic power formula is P = E x I, where E is voltage and I is current in the circuit. The amount of power changes when either voltage or current, or both voltage and current, are caused to change. In practice, the ONLY factors that can be changed are voltage and resistance. In explaining the different forms that formulas may take, current is sometimes presented as a quantity that is changed. Remember, if current is changed, it is because either voltage or resistance has been changed. Figure 3-9 shows a basic circuit using a source of power that can be varied from 0 to 8 volts and a graph that indicates the relationship between voltage and power. The resistance of this circuit is 2 ohms; this value does not change. Voltage (E) is increased (by increasing the voltage source), in steps of 1 volt, from 0 volts to 8 volts. By applying Ohm’s law, the current (I) is determined for each step of voltage. For instance, when E is 1 volt, the current is: R j _ * vo ^ 2 ohms I = 0.5 ampere Figure 3-9. — Graph of power related to changing voltage. 3-12 Power (P), in watts, is determined by applying the basic power formula: P = E x I P = 1 volt x 0.5 ampere P = 0.5 watt When E is increased to 2 volts: R I _ 2 volts 2 ohms I = 1 amp ere and P = E x I P = 2 volts x 1 ampere P = 2 watts When E is increased to 3 volts: = _E R 3 volts 2 ohms = 1.5 amperes = Ex I = 3 volts x 1.5 ampere = 4.5 watts You should notice that when the voltage was increased to 2 volts, the power increased from .5 watts to 2 watts or 4 times. When the voltage increased to 3 volts, the power increased to 4.5 watts or 9 times. This shows that if the resistance in a circuit is held constant, the power varies directly with the SQUARE OF THE VOLTAGE. Another way of proving that power varies as the square of the voltage when resistance is held constant is: 3-13 Since: I E R By substitution in: P You get: P Or: P Therefore: P = E x I ExE R Another important relationship may be seen by studying figure 3-10. Thus far, power has been calculated with voltage and current (P = Ex I), and with voltage and resistance Referring to figure 3-10, note that power also varies as the square of current just as it does with 2 voltage. Thus, another formula for power, with current and resistance as its factors, is P = I R. This can be proved by: Since: E = 1 x R By substi tuition in: P = E x I You get: P = I x Rx I Or: P = I x I x R Therefore: P = I 2 x R 3-14 Figure 3-10. — Graph of power related to changing current. Up to this point, four of the most important electrical quantities have been discussed. These are voltage (E), current (I), resistance (R), and power (P). You must understand the relationships which exist among these quantities because they are used throughout your study of electricity. In the preceding paragraphs, P was expressed in terms of alternate pairs of the other three basic quantities E, I, and R. In practice, you should be able to express any one of these quantities in terms of any two of the others. Figure 3-1 1 is a summary of 12 basic formulas you should know. The four quantities E, I, R, and P are at the center of the figure. Adjacent to each quantity are three segments. Note that in each segment, the basic quantity is expressed in terms of two other basic quantities, and no two segments are alike. 3-15 For example, the formula wheel in figure 3-11 could be used to find the formula to solve the following problem: A circuit has a voltage source that delivers 6 volts and the circuit uses 3 watts of power. What is the resistance of the load? Since R is the quantity you have been asked to find, look in the section of the wheel that has R in the center. The segment contains the quantities you have been given. The formula you would use is The problem can now be solved. Given: E = 6 volts P = 3 watts E 2 Soultion: R = — P (6 volts) 2 3 watts = 12 ohms 3-16 Q9. What is the term applied to the rate at which a mechanical or electrical force causes motion? Q10. How can the amount of current be changed in a circuit? Qll. What are the three formulas for electrical power? POWER RATING Electrical components are often given a power rating. The power rating, in watts, indicates the rate at which the device converts electrical energy into another form of energy, such as light, heat, or motion. An example of such a rating is noted when comparing a 150- watt lamp to a 100- watt lamp. The higher wattage rating of the 150- watt lamp indicates it is capable of converting more electrical energy into light energy than the lamp of the lower rating. Other common examples of devices with power ratings are soldering irons and small electric motors. In some electrical devices the wattage rating indicates the maximum power the device is designed to use rather than the normal operating power. A 150-watt lamp, for example, uses 150 watts when operated at the specified voltage printed on the bulb . In contrast, a device such as a resistor is not normally given a voltage or a current rating. A resistor is given a power rating in watts and can be operated at any combination of voltage and current as long as the power rating is not exceeded. In most circuits, the actual power used by a resistor is considerably less than the power rating of the resistor because a 50% safety factor is used. For example, if a resistor normally used 2 watts of power, a resistor with a power rating of 3 watts would be used. Resistors of the same resistance value are available in different wattage values. Carbon resistors, for example, are commonly made in wattage ratings of 1/8, 1/4, 1/2, 1, and 2 watts. The larger the physical size of a carbon resistor the higher the wattage rating. This is true because a larger surface area of material radiates a greater amount of heat more easily. When resistors with wattage ratings greater than 5 watts are needed, wirewound resistors are used. Wirewound resistors are made in values between 5 and 200 watts. Special types of wirewound resistors are used for power in excess of 200 watts. As with other electrical quantities, prefixes may be attached to the word watt when expressing very large or very small amounts of power. Some of the more common of these are the kilowatt (1,000 watts), the megawatt (1,000,000 watts), and the milliwatt (1/1,000 of a watt). Q12. What is the current in a circuit with 5 ohms of resistance that uses 180 watts of power? (refer to figure 3-12) Q13. What type of resistor should be used in the circuit described in question 12? Q14. What is the power used in a circuit that has 10 amperes of current through a 10-ohm resistor? 3-17 Figure 3-12. — Circuit for computing electrical quantities. POWER CONVERSION AND EFFICIENCY The term power consumption is common in the electrical field. It is applied to the use of power in the same sense that gasoline consumption is applied to the use of fuel in an automobile. Another common term is power conversion . Power is used by electrical devices and is converted from one form of energy to another. An electrical motor converts electrical energy to mechanical energy. An electric light bulb converts electrical energy into light energy and an electric range converts electrical energy into heat energy. Power used by electrical devices is measured in energy. This practical unit of electrical energy is equal to 1 watt of power used continuously for 1 hour. The term kilowatt hour (kWh) is used more extensively on a daily basis and is equal to 1,000 watt-hours. The EFFICIENCY of an electrical device is the ratio of power converted to useful energy divided by the power consumed by the device. This number will always be less than one (1.00) because of the losses in any electrical device. If a device has an efficiency rating of .95, it effectively transforms 95 watts into useful energy for every 100 watts of input power. The other 5 watts are lost to heat, or other losses which cannot be used. Calculating the amount of power converted by an electrical device is a simple matter. You need to know the length of time the device is operated and the input power or horsepower rating. Horsepower, a unit of work, is often found as a rating on electrical motors. One horsepower is equal to 746 watts . Example: A 3/4-hp motor operates 8 hours a day. How much power is converted by the motor per month? How many kWh does this represent? Given: t = 8 hrs x 30 days P = 3/4 hp Solution: Convert horsepower to watts P = hp x 746 watts P = 3/4 x 746 watts 3-18 p = 559 watts Convert watts to watt-hours P = work x time P = 559 watts x 8 x 30 P = 134,000 watt-hours per month (NOTE: These figures are rounded to the nearest 1000.) To convert to kWh _ Power in watt- hours ~ 1000 p _ 134000 in watt- hours ~ 1000 P = 13d kWh If the motor actually uses 137 kWh per month, what is the efficiency of the motor? Given: Power converted =134 kWh per month Power used =137 kWh per month Solution: _ Power converted Power used _ 13d kWh per month 137 kWh per month =.978 (Rounded to three figures) Q15. How much power is converted by a 1 -horsepower motor in 12 hours? Q16. What is the efficiency of the motor if it actually uses 9.5 kWh in 12 hours? EFF EFF EFF SERIES DC CIRCUITS When two unequal charges are connected by a conductor, a complete pathway for current exists. An electric circuit is a complete conducting pathway. It consists not only of the conductor, but also includes the path through the voltage source. Inside the voltage source current flows from the positive terminal, through the source, emerging at the negative terminal. 3-19 SERIES CIRCUIT CHARACTERISTICS A SERIES CIRCUIT is defined as a circuit that contains only ONE PATH for current flow. To compare the basic circuit that has been discussed and a more complex series circuit, figure 3-13 shows two circuits. The basic circuit has only one lamp and the series circuit has three lamps connected in series. Figure 3-13. — Comparison of basic and series circuits. Resistance in a Series Circuit Referring to figure 3-13, the current in a series circuit must flow through each lamp to complete the electrical path in the circuit. Each additional lamp offers added resistance. In a series circuit, THE TOTAL CIRCUIT RESISTANCE (R T ) IS EQUAL TO THE SUM OF THE INDIVIDUAL RESISTANCES. As an equation: R T = Ri + R 2 + R 3 + . . . R n NOTE: The subscript n denotes any number of additional resistances that might be in the equation. Example: In figure 3-14 a series circuit consisting of three resistors: one of 10 ohms, one of 15 ohms, and one of 30 ohms, is shown. A voltage source provides 1 10 volts. What is the total resistance? 3-20 vAA/V' R i ion =- 110 V R 3 30 Q vVvV Figure 3-14. — Solving for total resistance in a series circuit. Given: Ri = 10 ohms R 2 = 15 ohms R 3 = 30 ohms Soulution: Rt = Ri + R 2 + R 3 Rt = 10 ohms + 15 ohms + 30 ohms Rt = 55 ohms In some circuit applications, the total resistance is known and the value of one of the circuit resistors has to be determined. The equation R T = Rj + R 2 + R 3 can be transposed to solve for the value of the unknown resistance. Example: In figure 3-15 the total resistance of a circuit containing three resistors is 40 ohms. Two of the circuit resistors are 10 ohms each. Calculate the value of the third resistor (R 3 ). 3-21 Figure 3-15. — Calculating the value of one resistance in a series circuit. Gi ve n : Ri = 40 ohms R.2 = 10 ohms R3 = 10 ohms Solution: R T = Rl + R2 + R3 (Subtract Ri + R2 from both sides of the equation.) Rt ■ Ri ■ R2 = R3 R3 = Rt - Ri - R2 R3 = 40 ohms - 10 ohms - 10 ohms R3 = 40 ohms - 20 ohms R3 = 20 ohms Current in a Series Circuit Since there is only one path for current in a series circuit, the same current must flow through each component of the circuit. To determine the current in a series circuit, only the current through one of the components need be known. The fact that the same current flows through each component of a series circuit can be verified by inserting meters into the circuit at various points, as shown in figure 3-16. If this were done, each meter would be found to indicate the same value of current. 3-22 Figure 3-16. — Current in a series circuit. Voltage in a Series Circuit The voltage dropped across the resistor in a circuit consisting of a single resistor and a voltage source is the total voltage across the circuit and is equal to the applied voltage. The total voltage across a series circuit that consists of more than one resistor is also equal to the applied voltage, but consists of the sum of the individual resistor voltage drops. In any series circuit, the SUM of the resistor voltage drops must equal the source voltage. This statement can be proven by an examination of the circuit shown in figure 3-17. In this circuit a source potential (E T ) of 20 volts is dropped across a series circuit consisting of two 5-ohm resistors. The total resistance of the circuit (R T ) is equal to the sum of the two individual resistances, or 10 ohms. Using Ohm’s law the circuit current may be calculated as follows: Given: E? = 20 volts Rj =10 ohms Solution: 20 volts 10 ohms I T = 2 amps 3-23 Figure 3-17. — Calculating individual voltage drops in a series circuit. Since the value of the resistors is known to be 5 ohms each, and the current through the resistors is known to be 2 amperes, the voltage drops across the resistors can be calculated. The voltage (Ei) across Ri is therefore: Given: Ij = 2 amperes Ri = 5 ohms Solution: Ei = Ii x Ri Ei = 2 amperes x 5 ohms Ei = 10 volts By inspecting the circuit, you can see that R 2 is the same ohmic value as R x and carries the same current. The voltage drop across R 2 is therefore also equal to 10 volts. Adding these two 10-volts drops together gives a total drop of 20 volts, exactly equal to the applied voltage. For a series circuit then: Example: A series circuit consists of three resistors having values of 20 ohms, 30 ohms, and 50 ohms, respectively. Find the applied voltage if the current through the 30 ohm resistor is 2 amps. (The abbreviation amp is commonly used for ampere.) To solve the problem, a circuit diagram is first drawn and labeled (fig 3-18). 3-24 Figure 3-18. — Solving for applied voltage in a series circuit. Given: Ri = 20 ohms R .2 = 30 ohms R 3 = 50 ohms I = 2 amps Solution: Ej = Ej + E 2 + E 3 Ei = Ri x Ii (1 1 = The current through resistor Ri) E 2 = R 2 x I 2 E 3 = R 3 x I 3 Substituting: Et = (Ri x 1 1 ) + (R 2 x I 2 ) + (R 3 x I 3 ) Ej = (20 ohms x 2 amps ) + (30 ohms x 2 amps )+ (50 ohms x 2 amps ) Et = 40 volts + 60 volts + 100 volts Et = 200 volts NOTE: When you use Ohm’s law, the quantities for the equation MUST be taken from the SAME part of the circuit. In the above example the voltage across R 2 was computed using the current through R 2 and the resistance of R 2 . The value of the voltage dropped by a resistor is determined by the applied voltage and is in proportion to the circuit resistances. The voltage drops that occur in a series circuit are in direct proportion to the resistances. This is the result of having the same current flow through each resistor — the larger the ohmic value of the resistor, the larger the voltage drop across it. Q17. A series circuit consisting of three resistors has a current of 3 amps. IfRj = 20 ohms , R 2 = 60 ohms, and R 3 = 80 ohms, what is the (a) total resistance and (b) source voltage of the circuit? Q18. What is the voltage dropped by each resistor of the circuit described in question 17? Q19. If the current was increased to 4 amps, what would be the voltage drop across each resistor in the circuit described in question 17? Q20. What would have to be done to the circuit described in question 17 to increase the current to 4 amps ? Power in a Series Circuit Each of the resistors in a series circuit consumes power which is dissipated in the form of heat. Since this power must come from the source, the total power must be equal to the power consumed by the circuit resistances. In a series circuit the total power is equal to the SUM of the power dissipated by the individual resistors. Total power (P T ) is equal to: Pt — Pi + ?2 + P3 • • -Pn Example: A series circuit consists of three resistors having values of 5 ohms, 10 ohms, and 15 ohms. Find the total power when 120 volts is applied to the circuit. (See fig. 3-19.) Figure 3-19. — Solving for total power in a series circuit. 3-26 Given: Rl = 5 ohms R2 = 10 ohms R3 = 15 ohms E = 120 volts Solution: The total resistance is found first. Rl = K\ + R? + R3 Rt = 5 ohms + 10 ohms + 15 ohms Rt = 30 ohms By using the total resistance and the applied voltage, the circuit current is calculated. 1 = I _ 120 volts 30 ohms I = 4 amps By means of the power formulas, the power can be calculated for each resistor: For Ri : Pj = I 2 x R t Pi = ( 4 amps) 2 x 5 ohms Pi = 80 watts For R2 : P2 = I 2 x R2 P2 = ( 4 amps) 2 x 10 ohms P2 = 160 watts For R 3 : P 3 = I 2 x R 3 P3 = ( 4 amps) 2 x 15 ohms P3 = 240 watts For total power: Pt = Pi + P2 + P3 Pt = 80 watts + 160 watts + 240 watts Pt = 480 watts To check the answer, the total power delivered by the source can be calculated: 3-27 P source “ I source x E source ^source = 4 Simps X 120 Volts P* source = ^J80w&ttS The total power is equal to the sum of the power used by the individual resistors. SUMMARY OF CHARACTERISTICS The important factors governing the operation of a series circuit are listed below. These factors have been set up as a group of rules so that they may be easily studied. These rules must be completely understood before the study of more advanced circuit theory is undertaken. Rules for Series DC Circuits 1 . The same current flows through each part of a series circuit. 2. The total resistance of a series circuit is equal to the sum of the individual resistances. 3. The total voltage across a series circuit is equal to the sum of the individual voltage drops. 4. The voltage drop across a resistor in a series circuit is proportional to the ohmic value of the resistor. 5. The total power in a series circuit is equal to the sum of the individual powers used by each circuit component. SERIES CIRCUIT ANALYSIS To establish a procedure for solving series circuits, the following sample problems will be solved. Example: Three resistors of 5 ohms, 10 ohms, and 15 ohms are connected in series with a power source of 90 volts as shown in figure 3-20. Find the total resistance, circuit current, voltage drop of each resistor, power of each resistor, and total power of the circuit. AW R i sa R 3 isa AW 32NVE063 Figure 3-20. — Solving for various values in a series circuit. 3-28 In solving the circuit the total resistance will be found first. Next, the circuit current will be calculated. Once the current is known, the voltage drops and power dissipations can be calculated. Gi ve n : Ri = 5 ohms R 2 = 10 ohms R 3 = 15 ohms E = 90 volts Solution: Rj = Rl + R2 + R3 Rx = 5 ohms + 10 ohms + 15 ohms Rj = 30 ohms 90 volts 30 ohms I = 3 amps Ei-IRi Ei = 3 amperes x 5 ohms Ei = 15 volts E2 = IR2 E 2 = 3 amperes x 10 ohms E 2 = 30 volts E 3 = IR 3 E 3 = 3 amperes x 15 ohms E 3 = 45 volts Pi = I x Ei Pi = 3 amperes x 15 volts Pi = 45 watts P 2 = I x E 2 P 2 = 3 amperes x 30 volts P 2 = 90 watts 3-29 P 3 = I x E 3 P 3 = 3 amperes x AS volts P 3 = 135 watts Pj = Ei x I Pj = 90 volts x 3 amps Pj = 270 watts or ?j = Pi + P2 + P3 Px = 45 watts + 90 watts + 135 watts Pj = 270 watts Example: Four resistors, R\ = 10 ohms, R 2 = 10 ohms, R 3 = 50 ohms, and R 4 = 30 ohms, are connected in series with a power source as shown in figure 3-21. The current through the circuit is ampere. a. What is the battery voltage? b. What is the voltage across each resistor? c. What is the power expended in each resistor? d. What is the total power? Figure 3-21. — Computing series circuit values. 3-30 Given: Ri = 10 ohms R.2 = 10 ohms R3 = 50 ohms R4 = 30 ohms I = 0.5 amps Solution (a): Ej = IRt Rt = Ri + R2 + R3 + R4 Rt = 10 ohms + 10 ohms + 50 ohms + 30 ohms Rt = 100 ohms Ej = 0.5 ampsx 100 ohms Ej = 50 volts Solution (b): E1-IR1 Ej = 0.5 amperes x 10 ohms Ei = 5 volts E2 = IR2 E2 = 0.5 amperes x 10 ohms E2 = 5 volts E3 = IR3 E3 = 0.5 amperes x 50 ohms E3 = 25 volts E4 = IR4 E4 = 0.5 amperes x 30 ohms E4 = 15 volts Solution (c): 3-31 Pl-IEi Pi = 0.5 amperes x 5 volts Pi = 2.5 watts p 2 = ie 2 P2 = 0.5 amperes x 5 volts P2 = 2.5 watts p 3 = ie 3 P3 = 0.5 amperes x 25 volts P3 = 12.5 watts P 4 = IE4 P4 = 0.5 amperes x 15 volts P4 = 7.5 watts Solution (d): Pi = Pi + P 2 + P3 + P4 Pt = 2.5 watts +2.5 watts + 12.5 watts + 7.5 watts Pi = 25 watts or Pj = IEj Pj = 0. 5 amp ere s x 50 volts Pt = 25 watts or E t 2 P T =^— T r t _ (50 volts) 2 T 100 ohms ^ 2500 volts P t = ; 1 100 ohms Pt = 25 watts An important fact to keep in mind when applying Ohm’s law to a series circuit is to consider whether the values used are component values or total values. When the information available enables the use of Ohm’s law to find total resistance, total voltage, and total current, total values must be inserted into the formula. To find total resistance: 3-32 To find total voltage: Et = Ij = Rj To find total current: NOTE: I T is equal to I in a series circuit. However, the distinction between I T and I in the formula should be noted. The reason for this is that future circuits may have several currents, and it will be necessary to differentiate between I T and other currents. To compute any quantity (E, I, R, or P) associated with a single given resistor, the values used in the formula must be obtained from that particular resistor. For example, to find the value of an unknown resistance, the voltage across and the current through that particular resistor must be used. To find the value of a resistor: To find the voltage drop across a resistor: Er - Ir x R To find current through a resistor: I Q21. A series circuit consists of two resistors in series. = 25 ohms and R 2 = 30 ohms. The circuit current is 6 amps. What is the (a) source voltage, (b) voltage dropped by each resistor, (c) total power, and (d) power used by each resistor? KIRCHHOFF ’S VOLTAGE LAW In 1847, G. R. Kirchhoff extended the use of Ohm’s law by developing a simple concept concerning the voltages contained in a series circuit loop. Kirchhoff s voltage law states: "The algebraic sum of the voltage drops in any closed path in a circuit and the electromotive forces in that path is equal to zero." 3-33 To state Kirchhoff’s law another way, the voltage drops and voltage sources in a circuit are equal at any given moment in time. If the voltage sources are assumed to have one sign (positive or negative) at that instant and the voltage drops are assumed to have the opposite sign, the result of adding the voltage sources and voltage drops will be zero. NOTE: The terms electromotive force and emf are used when explaining Kirchhoff’s law because Kirchhoff’s law is used in alternating current circuits (covered in Module 2). In applying Kirchhoff’s law to direct current circuits, the terms electromotive force and emf apply to voltage sources such as batteries or power supplies. Through the use of Kirchhoff’s law, circuit problems can be solved which would be difficult, and often impossible, with knowledge of Ohm’s law alone. When Kirchhoff’s law is properly applied, an equation can be set up for a closed loop and the unknown circuit values can be calculated. POLARITY OF VOLTAGE To apply Kirchhoff’s voltage law, the meaning of voltage polarity must be understood. In the circuit shown in figure 3-22, the current is shown flowing in a counterclockwise direction. Notice that the end of resistor R b into which the current flows, is marked NEGATIVE (-). The end of R\ at which the current leaves is marked POSITIVE (+). These polarity markings are used to show that the end of Ri into which the current flows is at a higher negative potential than the end of the resistor at which the current leaves. Point A is more negative than point B. Figure 3-22. — Voltage polarities. Point C, which is at the same potential as point B, is labeled negative. This is to indicate that point C is more negative than point D. To say a point is positive (or negative) without stating what the polarity is based upon has no meaning. In working with Kirchhoff’s law, positive and negative polarities are assigned in the direction of current flow. 3-34 APPLICATION OF KIRCHHOFF’S VOLTAGE LAW Kirchhoff’s voltage law can be written as an equation, as shown below: E a + Eb + E c + . . . E n = 0 where E a , E b , etc., are the voltage drops or emf’s around any closed circuit loop. To set up the equation for an actual circuit, the following procedure is used. 1. Assume a direction of current through the circuit. (The correct direction is desirable but not necessary.) 2. Using the assumed direction of current, assign polarities to all resistors through which the current flows. 3. Place the correct polarities on any sources included in the circuit. 4. Starting at any point in the circuit, trace around the circuit, writing down the amount and polarity of the voltage across each component in succession. The polarity used is the sign AFTER the assumed current has passed through the component. Stop when the point at which the trace was started is reached. 5. Place these voltages, with their polarities, into the equation and solve for the desired quantity. Example: Three resistors are connected across a 50-volt source. What is the voltage across the third resistor if the voltage drops across the first two resistors are 25 volts and 15 volts? Solution: First, a diagram, such as the one shown in figure 3-23, is drawn. Next, a direction of current is assumed (as shown). Using this current, the polarity markings are placed at each end of each resistor and also on the terminals of the source. Starting at point A, trace around the circuit in the direction of current flow, recording the voltage and polarity of each component. Starting at point A and using the components from the circuit: ( + E,) + (+E 2 ) + (+E|)+(-E*)-0 Substituting values from the circuit: E x + 15 volts + 25 volts - 50 volts = 0 E x - 10 volts = 0 E x = 10 volts The unknown voltage (E x ) is found to be 10 volts 3-35 Figure 3-23. — Determining unknown voltage in a series circuit. Using the same idea as above, you can solve a problem in which the current is the unknown quantity. Example: A circuit having a source voltage of 60 volts contains three resistors of 5 ohms, 10 ohms, and 15 ohms. Find the circuit current. Solution: Draw and label the circuit (fig. 3-24). Establish a direction of current flow and assign polarities. Next, starting at any point — point A will be used in this example — write out the loop equation. Rl R3 Figure 3-24. — Correct direction of assumed current. 3-36 Basic equation: E2 + Ej + E A + E3 = 0 Since E=IR, by substitution: (IxR 2 )+(IxR 1 ) + E a +(IxR 3 ) = 0 Substituting Values: (I x 10 ohms) + (I x 5 ohms) + (-60 volts) + (I x 15 ohms)= 0 Combining like terms: (I x 30 ohms) + (-60 volts) = 0 (I x 30 ohms) = 60 volts I _ 60 volts 30 ohms I = 2 amp s Since the current obtained in the above calculations is a positive 2 amps, the assumed direction of current was correct. To show what happens if the incorrect direction of current is assumed, the problem will be solved as before, but with the opposite direction of current. The circuit is redrawn showing the new direction of current and new polarities in figure 3-25. Starting at point A the loop equation is: E3+ Ea + Ei + E2 = 0 (I x R 3 ) + E A + (I x Rj ) + (I x R 2 ) = 0 Substituting Values: (I x 15 ohms) + 60 volts + (I x 5 ohms) + (I x 10 ohms)= 0 Combining like terms: (I x 30 ohms) + 60 volts = 0 I x 30 ohms = -60 volts I = -60 volts 30 ohms 1 = -2 amp s 3-37 R 1 WVva 5 Q El 150 e 3 : W'/W' R 3 Figure 3-25. — Incorrect direction of assumed current. Notice that the AMOUNT of current is the same as before. The polarity, however, is NEGATIVE. The negative polarity simply indicates the wrong direction of current was assumed. Should it be necessary to use this current in further calculations on the circuit using Kirchhoff’s law, the negative polarity should be retained in the calculations. Series Aiding and Opposing Sources In many practical applications a circuit may contain more than one source of emf. Sources of emf that cause current to flow in the same direction are considered to be SERIES AIDING and the voltages are added. Sources of emf that would tend to force current in opposite directions are said to be SERIES OPPOSING, and the effective source voltage is the difference between the opposing voltages. When two opposing sources are inserted into a circuit current flow would be in a direction determined by the larger source. Examples of series aiding and opposing sources are shown in figure 3-26. 3-38 SERIES OPPOSING Figure 3-26. — Aiding and opposing sources. A simple solution may be obtained for a multiple-source circuit through the use of Kirchhoff’s voltage law. In applying this method, the same procedure is used for the multiple-source circuit as was used above for the single-source circuit. This is demonstrated by the following example. Example: Using Kirchhoff’s voltage equation, find the amount of current in the circuit shown in fig 3-27. Figure 3-27.-Solving for circuit current using Kirchhoff s voltage equation. 3-39 Solution: As before, a direction of current flow is assumed and polarity signs are placed on the drawing. The loop equation will be started at point A. E2 + Eri + E 1 + E3 + Er2 = 0 20 volts + (I x 60 ohms) + (- 180 volts) + AO volts + (I x 20 ohms) = 0 20 volts - 180 volts + A0 volts + (I x 60 ohms) + (I x 20 ohms) = 0 -120 volts + (I x 80 ohms) = 0 I x 80 ohms = 120 volts I _ 1 20 volts 80 ohms I =1.5 amps Q22. When using Kirchhoff’s voltage law, how are voltage polarities assigned to the voltage drops across resistors? Q23. Refer to figure 3-27, ifRj was changed to a 40-ohm resistor, what would be the value of circuit current ( I T ) ? Q24. Refer to figure 3-27. What is the effective source voltage of the circuit using the 40-ohm resistor? CIRCUIT TERMS AND CHARACTERISTICS Before you learn about the types of circuits other than the series circuit, you should become familiar with some of the terms and characteristics used in electrical circuits. These terms and characteristics will be used throughout your study of electricity and electronics. REFERENCE POINT A reference point is an arbitrarily chosen point to which all other points in the circuit are compared. In series circuits, any point can be chosen as a reference and the electrical potential at all other points can be determined in reference to that point. In figure 3-28 point A shall be considered the reference point. Each series resistor in the illustrated circuit is of equal value. The applied voltage is equally distributed across each resistor . The potential at point B is 25 volts more positive than at point A. Points C and D are 50 volts and 75 volts more positive than point A respectively. 3-40 D+75V 75V Eo = 25V C +50V = 25V B +25V = 25V A OV Figure 3-28. — Reference points in a series circuit. When point B is used as the reference, as in figure 3-29, point D would be positive 50 volts in respect to the new reference point. The former reference point, A, is 25 volts negative in respect to point B. Figure 3-29. — Determining potentials with respect to a reference point. 3-41 As in the previous circuit illustration, the reference point of a circuit is always considered to be at zero potential. Since the earth (ground) is said to be at a zero potential, the term GROUND is used to denote a common electrical point of zero potential. In figure 3-30, point A is the zero reference, or ground, and the symbol for ground is shown connected to point A. Point C is 75 volts positive in respect to ground. Figure 3-30. — Use of ground symbols. In most electrical equipment, the metal chassis is the common ground for the many electrical circuits. When each electrical circuit is completed, common points of a circuit at zero potential are connected directly to the metal chassis, thereby eliminating a large amount of connecting wire. The electrons pass through the metal chassis (a conductor) to reach other points of the circuit. An example of a chassis grounded circuit is illustrated in figure 3-31. 3-42 Figure 3-31. — Ground used as a conductor. Most voltage measurements used to check proper circuit operation in electrical equipment are taken in respect to ground. One meter lead is attached to a grounded point and the other meter lead is moved to various test points. Circuit measurement is explained in more detail in NEETS Module 3. OPEN CIRCUIT A circuit is said to be OPEN when a break exists in a complete conducting pathway. Although an open occurs when a switch is used to deenergize a circuit, an open may also develop accidentally. To restore a circuit to proper operation, the open must be located, its cause determined, and repairs made. Sometimes an open can be located visually by a close inspection of the circuit components. Defective components, such as burned out resistors, can usually be discovered by this method. Others, such as a break in wire covered by insulation or the melted element of an enclosed fuse, are not visible to the eye. Under such conditions, the understanding of the effect an open has on circuit conditions enables a technician to make use of test equipment to locate the open component. In figure 3-32, the series circuit consists of two resistors and a fuse. Notice the effects on circuit conditions when the fuse opens. 3-43 (A) NORMAL CIRCUIT (NORMAL CURRENT) (B) OPEN CIRCUIT (DUE TO EXCESSIVE CURRENT) Figure 3-32. — Normal and open circuit conditions. (A) Normal current; (B) Excessive current. Current ceases to flow; therefore, there is no longer a voltage drop across the resistors. Each end of the open conducting path becomes an extension of the battery terminals and the voltage felt across the open is equal to the applied voltage (E A ). An open circuit has INFINITE resistance. INFINITY represents a quantity so large it cannot be measured. The symbol for infinity is go. In an open circuit, R T = go. SHORT CIRCUIT A short circuit is an accidental path of low resistance which passes an abnormally high amount of current. A short circuit exists whenever the resistance of a circuit or the resistance of a part of a circuit drops in value to almost zero ohms. A short often occurs as a result of improper wiring or broken insulation. In figure 3-33, a short is caused by improper wiring. Note the effect on current flow. Since the resistor has in effect been replaced with a piece of wire, practically all the current flows through the short and very little current flows through the resistor. Electrons flow through the short (a path of almost zero resistance) and the remainder of the circuit by passing through the 10-ohm resistor and the battery. The amount of current flow increases greatly because its resistive path has decreased from 10,010 ohms to 10 ohms. Due to the excessive current flow the 10-ohm resistor becomes heated. As it attempts to dissipate this heat, the resistor will probably be destroyed. Figure 3-34 shows a pictorial wiring diagram, rather than a schematic diagram, to indicate how broken insulation might cause a short circuit. 3-44 Rl = 10,0000 /* SHORT 100 EXCESSIVE CURRENT B Figure 3-33. — Normal and short circuit conditions. INSULATED WIRE BARE WIRE Figure 3-34. — Short due to broken insulation. 3-45 SOURCE RESISTANCE A meter connected across the terminals of a good 1.5-volt battery reads about 1.5 volts. When the same battery is inserted into a complete circuit, the meter reading decreases to something less than 1.5 volts. This difference in terminal voltage is caused by the INTERNAL RESISTANCE of the battery (the opposition to current offered by the electrolyte in the battery). All sources of electromotive force have some form of internal resistance which causes a drop in terminal voltage as current flows through the source . This principle is illustrated in figure 3-35, where the internal resistance of a battery is shown as R[. In the schematic, the internal resistance is indicated by an additional resistor in series with the battery. The battery, with its internal resistance, is enclosed within the dotted lines of the schematic diagram. With the switch open, the voltage across the battery terminals reads 15 volts. When the switch is closed, current flow causes voltage drops around the circuit. The circuit current of 2 amperes causes a voltage drop of 2 volts across R[. The 1-ohm internal battery resistance thereby drops the battery terminal voltage to 13 volts. Internal resistance cannot be measured directly with a meter. An attempt to do this would damage the meter. Figure 3-35. — Effect of internal resistance. The effect of the source resistance on the power output of a dc source may be shown by an analysis of the circuit in figure 3-36. When the variable load resistor (R L ) is set at the zero-ohm position (equivalent to a short circuit), current (I) is calculated using the following formula: 100 volts 5 ohms = 20 amperes 3-46 This is the maximum current that may be drawn from the source. The terminal voltage across the short circuit is zero volts and all the voltage is across the resistance within the source. 4 E t t E s = OPEN - CIRCUIT VOLTAGE OF SOURCE Rj = INTERNAL RESISTANCE OF SOURCE E t = TERMINAL VOLTAGE R l = RESISTANCE OF LOAD P L = POWER USED IN LOAD I = CURRENT FROM SOURCE % EFF. = PERCENTAGE OF EFFICIENCY (A) CIRCUIT AND SYMBOL DESIGNATION r l E t 1 P L % EFF . 0 0 20 0 0 1 16.7 16.7 278.9 16.7 2 28.6 14.3 409 28.6 3 37.5 12.5 468.8 37.5 4 44.4 11.1 492.8 44.4 5 50 10 500 50 6 54.5 9.1 496.0 54.5 7 58.3 8.3 483.9 58.3 8 61.6 7.7 474.3 61.6 9 64.3 7.1 456.5 64.3 10 66.7 6.7 446.9 66.7 20 80 4 320 80 30 85.7 2.9 248.5 85.7 40 88.9 2.2 195.6 88.9 50 90.9 1.9 172.7 90.9 (B) CHART Cfl I— < _J a. 500 400 300 200 100 0 0 2 4 6 8 10 20 30 40 50 R L (OHMS) (C) GRAPH Figure 3-36. — Effect of source resistance on power output. 3-47 If the load resistance (R L ) were increased (the internal resistance remaining the same), the current drawn from the source would decrease. Consequently, the voltage drop across the internal resistance would decrease. At the same time, the terminal voltage applied across the load would increase and approach a maximum as the current approaches zero amps. POWER TRANSFER AND EFFICIENCY Maximum power is transferred from the source to the load when the resistance of the load is equal to the internal resistance of the source . This theory is illustrated in the table and the graph of figure 3-36. When the load resistance is 5 ohms, matching the source resistance, the maximum power of 500 watts is developed in the load. The efficiency of power transfer (ratio of output power to input power) from the source to the load increases as the load resistance is increased . The efficiency approaches 100 percent as the load resistance approaches a relatively large value compared with that of the source, since less power is lost in the source. The efficiency of power transfer is only 50 percent at the maximum power transfer point (when the load resistance equals the internal resistance of the source). The efficiency of power transfer approaches zero efficiency when the load resistance is relatively small compared with the internal resistance of the source. This is also shown on the chart of figure 3-36. The problem of a desire for both high efficiency and maximum power transfer is resolved by a compromise between maximum power transfer and high efficiency. Where the amounts of power involved are large and the efficiency is important, the load resistance is made large relative to the source resistance so that the losses are kept small. In this case, the efficiency is high. Where the problem of matching a source to a load is important, as in communications circuits, a strong signal may be more important than a high percentage of efficiency. In such cases, the efficiency of power transfer should be only about 50 percent; however, the power transfer would be the maximum which the source is capable of supplying. You should now understand the basic concepts of series circuits. The principles which have been presented are of lasting importance. Once equipped with a firm understanding of series circuits, you hold the key to an understanding of the parallel circuits to be presented next. Q25. A circuit has a source voltage of 100 volts and two 50-ohm resistors connected in series. If the reference point for this circuit is placed between the two resistors, what would be the voltage at the reference point? Q26. If the reference point in question 25 were connected to ground, what would be the voltage level of the reference point? Q27. What is an open circuit? Q28. What is a short circuit? Q29. Why will a meter indicate more voltage at the battery terminal when the battery is out of a circuit than when the battery is in a circuit? Q30. What condition gives maximum power transfer from the source to the load? Q31. What is the efficiency of power transfer in question 30? Q32. A circuit has a source voltage of 25 volts. The source resistance is 1 ohm and the load resistance is 49 ohms. What is the efficiency of power transfer? 3-48 PARALLEL DC CIRCUITS The discussion of electrical circuits presented up to this point has been concerned with series circuits in which there is only one path for current. There is another basic type of circuit known as the PARALLEL CIRCUIT with which you must become familiar. Where the series circuit has only one path for current, the parallel circuit has more than one path for current. Ohm’s law and Kirchhoff’s law apply to all electrical circuits, but the characteristics of a parallel dc circuit are different than those of a series dc circuit. PARALLEL CIRCUIT CHARACTERISTICS A PARALLEL CIRCUIT is defined as one having more than one current path connected to a common voltage source. Parallel circuits, therefore, must contain two or more resistances which are not connected in series. An example of a basic parallel circuit is shown in figure 3-37. Figure 3-37. — Example of a basic parallel circuit. Start at the voltage source (E s ) and trace counterclockwise around the circuit. Two complete and separate paths can be identified in which current can flow. One path is traced from the source, through resistance Ri, and back to the source. The other path is from the source, through resistance R 2 , and back to the source. Voltage in a Parallel Circuit You have seen that the source voltage in a series circuit divides proportionately across each resistor in the circuit. IN A PARALLEL CIRCUIT, THE SAME VOLTAGE IS PRESENT IN EACH BRANCH. (A branch is a section of a circuit that has a complete path for current.) In figure 3-37 this voltage is equal to the applied voltage (E s ). This can be expressed in equation form as: Voltage measurements taken across the resistors of a parallel circuit, as illustrated by figure 3-38 verify this equation. Each meter indicates the same amount of voltage. Notice that the voltage across each resistor is the same as the applied voltage. 3-49 Figure 3-38. — Voltage comparison in a parallel circuit. Example: Assume that the current through a resistor of a parallel circuit is known to be 4.5 milliamperes (4.5 mA) and the value of the resistor is 30,000 ohms (30 kQ). Determine the source voltage. The circuit is shown in figure 3-39. Given: R 2 = 30,000 ohms (30kQ) Ir 2 = 4.5 milliamps (4.5mA or .0045 amps) Solution: E = IR Er 2 = .0045 amp x 30,000 ohms Er 2 = 135 volts Figure 3-39. — Example problem parallel circuit. 3-50 Since the source voltage is equal to the voltage of a branch: Es = ER2 E$ = 135 volts To simplify the math operation, the values can be expressed in powers of ten as follows: 30,000 ohms =30 xl0 3 ohms 45mA = 45 xl0~ 3 amps Er2 =(4.5x 10 -3 ) amps x (30 x 10 3 ) ohms E r2 =(45x30xl0" 3 xlO 3 ) volts (10' 3 x 10 3 =10" 3+3 =10° =1) Er 2 =(45x 30 x l) volts E r2 =135 volts E S =e r2 E s =135 volts If you are not familiar with the use of the powers of 10 or would like to brush up on it, Mathematics, Vol. 1, NAVEDTRA 10069-C, will be of great help to you. Q33. What would the source voltage (E s ) in figure 3-39 be if the current through R 2 were 2 milliamps? Current in a Parallel Circuit Ohm’s law states that the current in a circuit is inversely proportional to the circuit resistance. This fact is true in both series and parallel circuits. There is a single path for current in a series circuit. The amount of current is determined by the total resistance of the circuit and the applied voltage. In a parallel circuit the source current divides among the available paths. The behavior of current in parallel circuits will be shown by a series of illustrations using example circuits with different values of resistance for a given value of applied voltage. Part (A) of figure 3-40 shows a basic series circuit. Here, the total current must pass through the single resistor. The amount of current can be determined. 3-51 Es 50V Figure 3-40. — Analysis of current in parallel circuit. Given: Solution: E s = 50 volts Rj = 10 ohms I = It = E R _ Es It = R 1 50 volts 1 0 ohms I T = 5 amps Part (B) of figure 3-40 shows the same resistor (Rj) with a second resistor (R 2 ) of equal value connected in parallel across the voltage source. When Ohm’s law is applied, the current flow through each resistor is found to be the same as the current through the single resistor in part (A). 3-52 Given: Solution: E$ = 50 volts Rl = 10 ohms R 2 = 10 ohms E s _e ri _e R2 i _ e R1 1,1 ' r7 _ 50 volts I P 1 — 10 ohms I R i = 5 amps 50 volts 10 ohms I R2 = 5 amps It is apparent that if there is 5 amperes of current through each of the two resistors, there must be a TOTAL CURRENT of 10 amperes drawn from the source. The total current of 10 amperes, as illustrated in figure 3-40(B), leaves the negative terminal of the battery and flows to point a. Since point a is a connecting point for the two resistors, it is called a JUNCTION. At junction a, the total current divides into two currents of 5 amperes each. These two currents flow through their respective resistors and rejoin at junction b. The total current then flows from junction b back to the positive terminal of the source. The source supplies a total current of 10 amperes and each of the two equal resistors carries one-half the total current. Each individual current path in the circuit of figure 3-40(B) is referred to as a BRANCH. Each branch carries a current that is a portion of the total current. Two or more branches form a NETWORK. From the previous explanation, the characteristics of current in a parallel circuit can be expressed in terms of the following general equation: It = II + I 2 + • • • In 3-53 Compare part (A) of figure 3-41 with part (B) of the circuit in figure 3-40. Notice that doubling the value of the second branch resistor (R 2 ) has no effect on the current in the first branch (Iri), but does reduce the second branch current (I R2 ) to one-half its original value. The total circuit current drops to a value equal to the sum of the branch currents. These facts are verified by the following equations. Given: E s = 50 volts =10 ohms R 2 = 20 ohms Solution: R E S = e ri = E R2 I - E Ri r i I _ 50 volts 10 ohms I R1 = 5 amps I 50 volts 20 ohms I R2 = 2.5 amps h = 1 ri + 1 R2 I T = 5 amps + 2.5 amps I T =7.5 amps 3-54 Figure 3-41. — Current behavior in parallel circuits. The amount of current flow in the branch circuits and the total current in the circuit shown in figure 3-4 1(B) are determined by the following computations. Given: E s = 50 volts Rj = 10 ohms R 2 =10 ohms R 3 = 10 ohms 3-55 Solution: I 50 volts 10 ohms I R1 = 5 amps I 50 volts 10 ohms I R2 = 5 amps 1 50 volts 10 ohms Jt - 1 R1 + !r2 + ^R3 I T =5 amps + 5 amps + Samps I T = 15 amps Notice that the sum of the ohmic values in each circuit shown in figure 3-41 is equal (30 ohms), and that the applied voltage is the same (50 volts). However, the total current in 3-4 1(B) (15 amps) is twice the amount in 3-41(A) (7.5 amps). It is apparent, therefore, that the manner in which resistors are connected in a circuit, as well as their actual ohmic values, affect the total current. The division of current in a parallel network follows a definite pattern. This pattern is described by KIRCHHOFF’S CURRENT LAW which states: 3-56 "The algebraic sum of the currents entering and leaving any junction of conductors is equal to zero." This law can be stated mathematically as: I a + lb + • • • I n + 0 where: I a , I b , etc., are the currents entering and leaving the junction. Currents ENTERING the junction are considered to be POSITIVE and currents LEAVING the junction are considered to be NEGATIVE. When solving a problem using Kirchhoff’s current law, the currents must be placed into the equation WITH THE PROPER POLARITY SIGNS ATTACHED. Example: Solve for the value of I 3 in figure 3-42. Given: 11 = 10 amps 12 = 3 amps h\ = 5 amps I a + It +... I n = 0 Solution: fa + l b + • • • I a + 0 Figure 3-42. — Circuit for example problem. The currents are placed into the equation with the proper signs. 3-57 Il+ I 2 + 13+ 1^ = 0 10 amps + (-3 amps) + I 3 + (-5 amps) = 0 I 3 + 2 amps = 0 13 = -2 amps I 3 has a value of 2 amperes, and the negative sign shows it to be a current LEAVING the junction. Example. Using figure 3-43, solve for the magnitude and direction of I 3 . Given: Solution: Figure 3-43. — Circuit for example problem. 11 = 6 amps 1 2 = 3 amps 1 4 = 5 amps I a + 4 +... In = 0 Ii + h + l3 + I 4 = 0 6 amps + (-3 amps) + I 3 + (-5 amps) = 0 I 3 + (-2 amps) = 0 13 = -2 amps 3-58 I 3 is 2 amperes and its positive sign shows it to be a current entering the junction. Q34. There is a relationship between total current and current through the individual components in a circuit. What is this relationship in a series circuit and a parallel circuit? Q35. In applying Kirchhoff’s current law, what does the polarity of the current indicate? Resistance in a Parallel Circuit In the example diagram, figure 3-44, there are two resistors connected in parallel across a 5-volt battery. Each has a resistance value of 10 ohms. A complete circuit consisting of two parallel paths is formed and current flows as shown. Figure 3-44. — Two equal resistors connected in parallel. Computing the individual currents shows that there is one -half of an ampere of current through each resistance. The total current flowing from the battery to the junction of the resistors, and returning from the resistors to the battery, is equal to 1 ampere. The total resistance of the circuit can be calculated by using the values of total voltage (E T ) and total current (I T ). NOTE: From this point on the abbreviations and symbology for electrical quantities will be used in example problems. Given: E T = 5 V I T - 1 A Solution: 3-59 This computation shows the total resistance to be 5 ohms; one-half the value of either of the two resistors. Since the total resistance of a parallel circuit is smaller than any of the individual resistors, total resistance of a parallel circuit is not the sum of the individual resistor values as was the case in a series circuit. The total resistance of resistors in parallel is also referred to as EQUIVALENT RESISTANCE (R eq ). The terms total resistance and equivalent resistance are used interchangeably. There are several methods used to determine the equivalent resistance of parallel circuits. The best method for a given circuit depends on the number and value of the resistors. For the circuit described above, where all resistors have the same value , the following simple equation is used: R N R eq = equivalent parallel resistance R= ohmic value of one resistor N = number of resistors This equation is valid for any number of parallel resistors of EQUAL VALUE. Example: Four 40-ohm resistors are connected in parallel. What is their equivalent resistance? Given: Rl + R 2 + R 3 + R 4 Rl = 40 Q Solution: R eq R eq R eq = R ~ N = 40Q ~4~ = 10 Q Figure 3-45 shows two resistors of unequal value in parallel. Since the total current is shown, the equivalent resistance can be calculated. 3-60 Figure 3-45. — Example circuit with unequal parallel resistors. Given: E s =30 V I T = 15 A Solution: E-eq E-eq E-eq Es = 30 V 15A = 2 Q The equivalent resistance of the circuit shown in figure 3-45 is smaller than either of the two resistors (R h R 2 ). An important point to remember is that the equivalent resistance of a parallel circuit is always less than the resistance of any branch . Equivalent resistance can be found if you know the individual resistance values and the source voltage. By calculating each branch current, adding the branch currents to calculate total current, and dividing the source voltage by the total current, the total can be found. This method, while effective, is somewhat lengthy. A quicker method of finding equivalent resistance is to use the general formula for resistors in parallel: 111 1 = — + + Rl R 2 R 3 R„ If you apply the general formula to the circuit shown in figure 3-45 you will get the same value for equivalent resistance (2£T) as was obtained in the previous calculation that used source voltage and total current. 3-61 Given: Rj — 3Q R2 = 6Q Solution: R eq R 1 R 2 1 1 1 = + R eg 3Q 6Q Convert the fractions to a common denominator. 1 2 1 — + R eq 6Q 6Q 1 _ 3 R eq 6Q 1 _ 1 R eq 2Q Since both sides are reciprocals (divided into one), disregard the reciprocal function. R eq =2Q The formula you were given for equal resistors in parallel < r *-£> is a simplification of the general formula for resistors in parallel _^ = A + _L+J_+ _i_ R eq R 1 R 2 R 3 There are other simplifications of the general formula for resistors in parallel which can be used to calculate the total or equivalent resistance in a parallel circuit. RECIPROCAL METHOD. — This method is based upon taking the reciprocal of each side of the equation. This presents the general formula for resistors in parallel as: R 1 *q 1 1 1 — + — +... — R 1 R 2 R n 3-62 This formula is used to solve for the equivalent resistance of a number of unequal parallel resistors. You must find the lowest common denominator in solving these problems. If you are a little hazy on finding the lowest common denominator, brush up on it in Mathematics Volume 7, NAVEDTRA 10069 (Series). Example: Three resistors are connected in parallel as shown in figure 3-46. The resistor values are: Ri = 20 ohms, R 2 = 30 ohms, R 3 = 40 ohms. What is the equivalent resistance? (Use the reciprocal method.) Figure 3-46. — Example parallel circuit with unequal branch resistors. Given: R i = 20 Q R 2 =30Q R 3 =40Q Solution: 3-63 1 "i i r — + — + — 20Q 30Q AOQ 1 6 ^ 4 ^ 3 120Q + 120Q + 120Q R eq = 9.23Q PRODUCT OVER THE SUM METHOD. — A convenient method for finding the equivalent, or total, resistance of two parallel resistors is by using the following formula. Rj x ^2 Rl +R 2 This equation, called the product over the sum formula, is used so frequently it should be committed to memory. Example: What is the equivalent resistance of a 20-ohm and a 30-ohm resistor connected in parallel, as in figure 3-47? Figure 3-47. — Parallel circuit with two unequal resistors. 3-64 Given: Solution: R t = 20Q R2 ~ 30Q P~1 x P-2 Rl + R 2 2 0 s 2 x 3 0 ^ 2 20Q + 30Q 600 Q 50 12Q Q36. Four equal resistors are connected in parallel, each resistor has an ohmic value of 100 ohms, what is the equivalent resistance? Q37. Three resistors connected in parallel have values of 12 kQ, 20 kQ, and 30 kQ. What is the equivalent resistance? Q38. Two resistors connected in parallel have values of 10 kQ and 30 kQ. What is the equivalent resistance ? Power in a Parallel Circuit Power computations in a parallel circuit are essentially the same as those used for the series circuit. Since power dissipation in resistors consists of a heat loss, power dissipations are additive regardless of how the resistors are connected in the circuit. The total power is equal to the sum of the power dissipated by the individual resistors. Like the series circuit, the total power consumed by the parallel circuit is: p T = Pi + P2 Example: Find the total power consumed by the circuit in figure 3-48. 3-65 Given: Solution: Figure 3-48. — Example parallel circuit. Rl = 10 Q Iri=5A R 2 = 25 Q lR2= 2A R 3 = 50 Q lR3= 1A P = I 2 R Pri = (Iri) 2x Rl Pri = (5 A) 2 x 10O Pri = 250W 3-66 PR2 = (IR2 ) 2 x P-2 P R2 = (2 A ) 2 X 25Q P R 2 = 100W Pr3 = (Ir3) 2 x R 3 P R3 = (1 A) 2 x50Q Pr 3 = SOW P T = Pri + Pr2 + Pr3 P T = 250W + 100W+50W P T = 400W Since the total current and source voltage are known, the total power can also be computed by: Given: E s = 50V I T = 8 A Solution: Pj = E$ x If P T = 50 V x 8 A P T = 400W Equivalent Circuits In the study of electricity, it is often necessary to reduce a complex circuit into a simpler form. Any complex circuit consisting of resistances can be redrawn (reduced) to a basic equivalent circuit containing the voltage source and a single resistor representing total resistance. This process is called reduction to an EQUIVALENT CIRCUIT. Figure 3-49 shows a parallel circuit with three resistors of equal value and the redrawn equivalent circuit. The parallel circuit shown in part A shows the original circuit. To create the equivalent circuit, you must first calculate the equivalent resistance. 3-67 Figure 3-49. — Parallel circuit with equivalent circuit. Given: 45Q 45Q 45Q R *' eq N _ 45Q R eq 3 R eq = 15 Q Once the equivalent resistance is known, a new circuit is drawn consisting of a single resistor (to represent the equivalent resistance) and the voltage source, as shown in part B. Rules for Parallel DC Circuits 1. The same voltage exists across each branch of a parallel circuit and is equal to the source voltage. Ri = R3 = Solution: 3-68 2. The current through a branch of a parallel network is inversely proportional to the amount of resistance of the branch. 3. The total current of a parallel circuit is equal to the sum of the individual branch currents of the circuit. 4. The total resistance of a parallel circuit is found by the general formula: ^eq ^1 ^2 or one of the formulas derived from this general formula. 5. The total power consumed in a parallel circuit is equal to the sum of the power consumptions of the individual resistances. SOLVING PARALLEL CIRCUIT PROBLEMS Problems involving the determination of resistance, voltage, current, and power in a parallel circuit are solved as simply as in a series circuit. The procedure is the same — (1) draw the circuit diagram, (2) state the values given and the values to be found, (3) select the equations to be used in solving for the unknown quantities based upon the known quantities, and (4) substitute the known values in the equation you have selected and solve for the unknown value. Example: A parallel circuit consists of five resistors. The value of each resistor is known and the current through Ri is known. You are asked to calculate the value for total resistance, total power, total current, source voltage, the power used by each resistor, and the current through resistors R 2 , R 3 , R 4 , and r 5 . Given: Rl = 20Q R 2 = 300 R 3 = 180 R 4 = 180 R 5 = 18Q Iri = 9A Find: R t , Ej , lx, P t , Ir2, Ir3- Ir4- Ir5 ' PrI* Pr2 ' P R3- PR4' P R5 This may appear to be a large amount of mathematical manipulation. However, if you use the step- by-step approach, the circuit will fall apart quite easily. The first step in solving this problem is for you to draw the circuit and indicate the known values as shown in figure 3-50. 3-69 Figure 3-50. — Parallel circuit problem. There are several ways to approach this problem. With the values you have been given, you could first solve for R T , the power used by R l5 or the voltage across R b which you know is equal to the source voltage and the voltage across each of the other resistors. Solving for R T or the power used by R\ will not help in solving for the other unknown values. Once the voltage across R] is known, this value will help you calculate other unknowns. Therefore the logical unknown to solve for is the source voltage (the voltage across Rj). Given: R\ = 200 Iri = 9A Eri = E$ Solution: Es = Ri x Iri E$ = 9A x 20Q E s = 180V Now that source voltage is known, you can solve for current in each branch. Given: E s = 180V R? = 300 R 3 = 180 R 4 = 18Q R 5 = 180 3-70 Solution: 1 180 V 18 Q I = 10 A Since R 3 = R 4 = R 5 and the voltage across each branch is the same Ir< =10 A Ir 5 =10A Solving for total resistance. Given: Rl = 200 R 3 = 3 0 Q R 3 = 180 R^= 18Q R5 = 180 3-71 Solution: 1 _ i a i ^ i i ^ i R7 20Q 30Q 18Q 18Q \SQ 1 _ 9 + 6 + 10 + 10 +10Q R7 180 (LCD) 45Q "l8C>" 180 45Q 4Q An alternate method for solving for R T can be used. By observation, you can see that R 3 , R 4 , and R 5 are of equal ohmic value. Therefore an equivalent resistor can be substituted for these three resistors in solving for total resistance. Given: R3 =R^ = R5 = 180 Solution: ^eql R-eql “ The circuit can now be redrawn using a resistor labeled R eq i in place of R 3 , R 4 , and R 5 as shown in figure 3-51. 3-72 Figure 3-51. — First equivalent parallel circuit. An equivalent resistor can be calculated and substituted for Rj and R 2 by use of the product over the sum formula. Given: Rq = 20 Q R 2 = 30Q Solution: Rl x ^2 Rl + R 2 20Q x 30 Q 20Q + 30Q The circuit is now redrawn again using a resistor labeled R eq 2 in place of Ri and R 2 as shown in figure 3-52. 3-73 Es Req 2 12G 180 V Figure 3-52. — Second equivalent parallel circuit. You are now left with two resistors in parallel. The product over the sum method can now be used to solve for total resistance. Given: Solution: R eq i = 6Q R eq i = 12Q R t = D _ R 1 X R 2 eq _ R 1+ R 2 R T - R eq1 xR eq2 R eq 1 + R eq 2 Rt ■ 6Q x12Q 6Q +12Q = 4Q This agrees with the solution found by using the general formula for solving for resistors in parallel. The circuit can now be redrawn as shown in figure 3-53 and total current can be calculated. 3-74 Figure 3-53. — Parallel circuit redrawn to final equivalent circuit. Given: E s =180V Rj = 4 'i, 2 Solution: _ _ 180V 1 T - I T = 45A This solution can be checked by using the values already calculated for the branch currents. Given: Iri=9A I R2 =6A i R3 =ioa Ir4 = 10A I R5 =10A Solution: h =I R1 +I R2 + " • ^ Rn I T = 9 A + 6A + 10 A + 10A + 10 A I T =45A 3-75 Now that total current is known, the next logical step is to find total power. Given: E s =180V I T = ASA Solution: P p T p T p T Solving for the power in each branch. Given: E s = 180V Iri=9A 1 R2 = 6A i R3 =ioa I R 4 = 10A I R5 =10A Solution: P = El P R1 = E S X 1 R1 P R1 =180V x 9 A P R1 = 1620W P R2 = E S x !r2 P R2 =180V x 6 A P R2 =1080W P R3 = E S x 1 R3 P R3 = 180 V x 10A P R3 = 1800W Since I R3 = I R4 = I R5 then, P R3 = P R4 = P R5 = 1800 W. The previous calculation for total power can now be checked. = El = E$ x It = 180V x 45A = 8100 watts = 8.1 kW 3-76 Given: P R 1= 1620W P R2 = 1080W P R3 = 1800W P R 4 = 1800W P R5 = 1800W Solution: Pj= PrI + Pr2+ Pr3 + PR4 + PR5 P T = 1620W+ 1080 W+ 1800 W+ 1800W+1800W P T = 8100W P T = 8. lkW Q39. What term identifies a single resistor that represents total resistance of a complex circuit? Q40. The total power in both series and parallel circuits is computed with the formula: Pt- Pi + P2 + P 3 +...P n . Why can this formula be used for both series and parallel circuits? Q41. A circuit consists of three resistors connected in parallel across a voltage source. R / = 40Q, R 2 = 30Q, R 3 = 40Q, and P R3 = 360 watts. Solve for R T , E s and I R2 . (Hint: Draw and label the circuit first.) SERIES-PARALLEL DC CIRCUITS In the preceding discussions, series and parallel dc circuits have been considered separately. The technician will encounter circuits consisting of both series and parallel elements. A circuit of this type is referred to as a COMBINATION CIRCUIT. Solving for the quantities and elements in a combination circuit is simply a matter of applying the laws and rules discussed up to this point. SOLVING COMBINATION-CIRCUIT PROBLEMS The basic technique used for solving dc combination-circuit problems is the use of equivalent circuits. To simplify a complex circuit to a simple circuit containing only one load, equivalent circuits are substituted (on paper) for the complex circuit they represent. To demonstrate the method used to solve combination circuit problems, the network shown in figure 3-54(A) will be used to calculate various circuit quantities, such as resistance, current, voltage, and power. 3-77 Figure 3-54. — Example combination circuit. Examination of the circuit shows that the only quantity that can be computed with the given information is the equivalent resistance of R 2 and R 3 . Given: R 2 = 20Q R 3 = 30Q Solution: D _R 2 x R 3 (Product over K eq 1 p + p. the sum) 2 3 _20Qx30Q eql 20Q +30Q Now that the equivalent resistance for R 2 and R 3 has been calculated, the circuit can be redrawn as a series circuit as shown in figure 3-54(B). 3-78 The equivalent resistance of this circuit (total resistance) can now be calculated. Given: Ri = 8Q (Resistors in series) R eq l = 12Q Solution: R eq “ R 1 + R eq 1 R e q = 8Q + 12Q R eg = 20Q or R j = 20Q The original circuit can be redrawn with a single resistor that represents the equivalent resistance of the entire circuit as shown in figure 3-54(C). To find total current in the circuit: Given: E s = 60 V R j = 20Q Solution: (Ohm's Law) To find total power in the circuit: Given: E s = 60 V I T = 3A 3-79 Solution: Pj — E LOAD ~+ ~~ t + 1 32NVE142 Figure 2D. — Battery consisting of 12 cells. IN ANSWERING QUESTIONS 2-44 AND 2-45, REFER TO FIGURE 2D. EACH CELL EQUALS 1.5 VOLTS AND HAS A CAPACITY OF 1/8 AMPERE. 2-44. What type of connection is used to combine the cells? 1. Series 2. Parallel 3. Series-parallel 2-45. What is the (a) voltage output and (b) current capacity of the circuit? 1. (a) 1.5 volts (b) 1.5 amperes 2. (a) 4.5 volts (b) 1/2 ampere 3. (a) 9 volts (b) 1/4 ampere 4. (a) 18 volts (b) 1/8 ampere 2-46. What is the first step in performing maintenance on a secondary-cell battery? 1 . Check the level of the electrolyte 2. Check the technical manual for information on the specific type of battery 3. Check the terminals for cleanliness and good electrical connection 4. Check the battery case for cleanliness and evidence of damage 2-47. When a hydrometer is used to check the specific gravity of the electrolyte in a battery, to what level should the electrolyte be drawn? 1 . Enough to j ust wet the float 2. Enough so the float will rise without entering the suction bulb 3. Enough so the top one -third of the float will rise into the suction bulb 4. Enough so the float is completely covered by the electrolyte 2-48. To flush a hydrometer, which of the following liquids should be used? 1 . Sulfuric acid 2. Salt water 3. Freshwater 4. A solution of baking soda and water 2-49. If the electrolyte level in a battery is low, what should be added to the electrolyte to bring it to the proper level? 1 . Tap water 2. Sulfuric acid 3 . Potassium hydroxide 4. Distilled water 13 2-50. Which one of the following safety precautions for batteries is NOT correct? 1 . Terminals should be electrically connected together before transporting a battery 2. Care should be taken to prevent the spilling of electrolyte 3. Smoking, open flames, and electrical sparks are prohibited around charging batteries 4. Protective clothing, such as rubber apron, rubber gloves, and face shield, should be worn when working on batteries 2-5 1 . If electrolyte comes in contact with the skin, what first aid treatment should be given immediately to the affected area? 1 . Cover with petroleum j elly 2. Wrap with a sterile bandage 3. Apply an antiseptic lotion 4. Flush with fresh water 2-52. A battery with a capacity of 600 ampere-hours should provide 3 amperes for a maximum of how many hours? 1. 100 hr 2. 200 hr 3. 300 hr 4. 600 hr 2-53. A battery is rated according to a 20-hour rate of discharge at 300 ampere-hours. Which of the following currents is the maximum current that will allow the battery to deliver its rated capacity? 1. 15 amperes 2. 20 amperes 3. 25 amperes 4. 30 amperes 2-54. Which of the following types of routine charges follows the nameplate data in restoring a battery to its charged condition during the ordinary cycle of operation? 1 . Initial 2. Floating 3. Normal 4. Fast THIS SPACE LEFT BLANK INTENTIONALLY. 14 IN ANSWERING QUESTIONS 2-55 THROUGH 2-58, MATCH THE DESCRIPTION GIVEN IN THE FOLLOWING LIST WITH THE TYPE OF BATTERY CHARGE IN THE QUESTION. A. TYPE OF B. DESCRIPTION CHARGE 2-55. Initial charge 1. Used in emergency only 2-56. Equalizing charge 2. Used periodically as part of a maintenance routine 2-57. Floating charge 3. Used to keep a battery at full charge while the battery is idle 2-58. Fast charge 4. Used after electrolyte is added to a dry-shipped battery 2-59. If violent gassing occurs during the charging of a battery, which of the following actions should be taken? 1 . Increase the room ventilation 2. Decrease the room temperature 3. Increase the charging rate 4. Decrease the charging rate 2-60. If a battery is being charged at the proper rate, which, if any of the following types of gassing should occur? 1 . Steady gassing 2. Intermittent gassing 3. Violent gassing 4. None 15 ASSIGNMENT 3 Textbook assignment: Chapter 3, “Direct Current,” pages 3-1 through 3-126. Figure 3A. — Basic circuit. IN ANSWERING QUESTIONS 3-1 THROUGH 3-3, REFER TO FIGURE 3A. 3-1 . What parts of the circuit represent the (a) source and (b) load? 1. (a) E s (b) S, 2. (a) E s (b) Ri 3. (a) Si (b) R\ 4. (a) Si (b) E s 3-2. Which of the following terms describes the circuit condition? 1 . Partially shorted 2. Partially open 3. Shorted 4. Open 3-3. Which of the following terms describes the figure 3 A? 1 . Parts layout 2. Exploded view 3. Wiring diagram 4. Schematic diagram 3-4. If circuit voltage is held constant, circuit current will react in what manner as the resistance (a) increases, and (b) decreases? 1 . (a) Increase 2. (a) Increase 3. (a) Decrease 4. (a) Decrease (b) decrease (b) increase (b) decrease (b) increase 3-5. If circuit resistance is held constant, circuit current will react in what manner as the voltage (a) increases, and (b) decreases? 1 . (a) Increase 2. (a) Increase 3. (a) Decrease 4. (a) Decrease (b) decrease (b) increase (b) decrease (b) increase 3-6. According to Ohm's law, what formula should be used to calculate circuit voltage if resistance and current value are known? 1 . 2 . 3. E = E = E = R 7 I R IR 4. E 16 E Volts 32NVE144 Figure 3B. — Graph of current and voltage. IN ANSWERING QUESTIONS 3-7 AND 3-8, REFER TO FIGURE 3B. 3-7. If the current is 15 amperes, what is the value of the voltage? 1. 50 V 2. 75 V 3. 100 V 4. 150 V 3-8. If the voltage is 200 volts, what is the value of the current? 1. 10 A 2. 20 A 3. 30 A 4. 40 A 3-9. Which of the following terms applies to the rate at which an electrical force causes motion? 1 . Power 2. Energy 3 . Inertia 4. Each of the above 3-10. Which of the following circuit quantities can be varied ONLY by varying one of the other circuit quantities? 1. Voltage 2. Current 3. Resistance 4. Each of the above 3-11. Which of the following is a correct formula for determining power in an electrical circuit? 1 P = El 2. P = I 1 2 3 4 R R 4 . Each of the above 3-12. What is the current in a circuit with 15 ohms of resistance that uses 135 watts of power? 1. 10 A 2. 15 A 3. 3 A 4. 9 A 3-13. What is the total power used by a 15 -ohm resistor with 4 amps of current? 1. 60 W 2. 240 W 3. 360 W 4. 900 W 3-14. What type of resistor should be used in question 3-13? 1 . Carbon 2. Wire wound 3. Precision 4. Composition 17 3-15. How much total energy is converted by a 1-horsepower motor in 1 0 hours? 1. 7.46 kWh 2. 8.32 kWh 3. 8.59 kWh 4. 9.32 kWh 3-16. If the energy used by the motor in question 3-15 is 9.5 kWh, what is the efficiency of the motor? 1. .981 2. .904 3. .876 4. .785 Figure 3C. — Series circuit. IN ANSWERING QUESTIONS 3-17 THROUGH 3-23, REFER TO FIGURE 3C. 3-17. What is the total circuit resistance (R)? 1. 20Q 2. 60Q 3. 1800 4. 2400 3-18. If the circuit current is 3 amps, what is the source voltage (E s )? 1. 60 V 2. 180 V 3. 540 V 4. 720 V 3-19. What is the total voltage dropped by each resistor in question 3-18? 1. 20 V 2. 60 V 3. 180 V 4. 540 V 3-20. If the current decreases to 2 amps, what is the total voltage drop across each resistor? 1. 120 V 2. 230 V 3. 310V 4. 400 V 3-21. What would have to be done to the circuit to cause the current to decrease to 2 amps? 1 . The source voltage would have to be increased 2. The source voltage would have to be decreased 3. The resistance of Ri would have to be decreased 4. One of the resistors would have to be removed from the circuit 3-22. If the circuit current is 2 amps, what is the total power used by each resistor? 1. 240 W 2. 460 W 3. 620 W 4. 800 W 3-23. What is the total power used in the circuit if E s = 360 V? 1. 720 W 2. 1380 W 3. 1860 W 4. 2400 W 18 3-24. When Kirchoff s voltage law is used to assign polarities to the voltage drop across a resistor, which of the following references is used to indicate the end of the resistor that the current enters? 1 . Ground 2. Neutral 3. Negative 4. Positive 3-27. Which of the following terms applies to a circuit in which there is NO complete path for current? 1 . Open 2. Short 3. Closed 4. Grounded 3-28. A circuit in which the resistance is almost zero ohms is referred to by which of the following terms? 1 . Open 2. Short 3. Closed 4. Broken THIS SPACE LEFT BLANK INTENTIONALLY. 32NVE146 Figure 3D. — Multiple source circuit. IN ANSWERING QUESTIONS 3-25 AND 3-26, REFER TO FIGURE 3D. 3-25. What is the effective source voltage? 1. 15 V 2. 25 V 3. 50 V 4. 75 V 3-26. What is the total amount and direction of current through R 3 ? 1. 1.0 A from Y to X 2. 1.0 A from X to Y 3. .33 A from Y to X 4. .33 A from X to Y 19 Figure 3E. — Series circuit and source resistance. IN ANSWERING QUESTIONS 3-29 THROUGH 3-32, REFER TO FIGURE 3E. 3-32. To achieve maximum power transfer in the circuit, which of the following conditions must be met? 1. Ri = R L 2. I S = I L 3. E s = E l 4. K s = K l 3-33. Maximum power is transferred from a source to a load when the value of the load resistance is of what value when compared to the source resistance? 1 . Equal 2. Twice 3. One-half 4. Several times 3-29. If R 2 has a short circuit, what will most likely happen to the circuit? 1 . Ri will be destroyed 2. E s will increase 3 . V will indicate O volts 4. Si will automatically open 3-30. What is the total voltage drop across Rj when the switch is closed? 1. 2.5 V 2. 6.5 V 3. 97.5 V 4. 100.0 V 3-31. What will the meter indicate with (a) S] open, and (b) Si closed? 1 . (a) 100 V (b) 100 V 2 . (a) 97.5 V (b) 100 V 3 . (a) 100 V (b) 97.5 4 . (a) 97.5 V (b) 97.5 3-34. When maximum power is transferred from a source to a load, what is the efficiency of power transfer? 1. 5% 2. 25% 3. 50% 4. 95% 3-35. A circuit consists of three resistors connected in parallel. Rj = 30 ohms, R 2 = 15 ohms, and R 3 = 1 0 ohms. If the current through R 2 = 4 amperes, what is the total source voltage? 1. 20 V 2. 60 V 3. 120 V 4. 220 V 3-36. What is the relationship of total current to the current through a component in (a) a series circuit, and (b) a parallel circuit? 1 . 2 . 3. 4. (a) Divides (a) Divides (a) Equals (a) Equals (b) divides (b) equals (b) equals (b) divides 20 3-37. If a current has a negative polarity when THIS SPACE LEFT BLANK Kirchoff s current law is applied, which of INTENTIONALLY, the following, statements is true of the current? 1 . It is from a battery 2. It is from a generator 3. It is entering a junction 4. It is leaving a junction 3-38. Three equal resistors are connected in parallel and each resistor has an ohmic value of 300 ohms. What is the equivalent resistance of the circuit? 1 . 1000 2. 1500 3. 6000 4. 9000 3-39. Three resistors with ohmic values of 120 ohms, 60 ohms, and 40 ohms are connected in parallel. What is the equivalent resistance of the circuit? 1 . 100 2 . 200 3. 300 4. 400 3-40. Two resistors with ohmic values of 90 ohms and 45 ohms are connected in parallel. What is the equivalent resistance of the circuit? 1 . 100 2 . 200 3. 300 4. 400 3-41. Which of the following terms describes a single resistor that represents a complex circuit? 1 . Equal resistor 2. Phantom resistor 3. Schematic resistor 4. Equivalent resistor 21 3-46. What is the total power consumed by R 3 ? IN ANSWERING QUESTIONS 3-42 THROUGH 3-46, REFER TO FIGURE 3F. 3-42. What is the value of E s ? 1. 336 V 2. 300 V 3. 240 V 4. 120 V 3-43. What is the value of current through R 2 ? 1. 1 A 2. 2 A 3. 3 A 4. 4 A 3-44. What is the approximate value of total resistance? 1. 80 2. 370 3. 1120 4. 2570 3-45. What is the value of total power? 1. 1.2 kW 2. 1.5 kW 3. 1.8 kW 4. 2.0 kW 1. 108 W 2. 240 W 3. 360 W 4. 1200 W IN ANSWERING QUESTIONS 3-47 THROUGH 3-49, REFER TO FIGURE 3G. 3-47. What is the value of the total resistance? 1. 3.60 2. 150 3. 340 4. 400 3-48. What is the total power used in the circuit? 1. 22.5 W 2. 26.5 W 3. 60.0 W 4. 250.0 W 3-49. What is the total voltage drop across R 3 ? 1. 8 V 2. 12 V 3. 18 V 4. 30 V 22 IN ANSWERING QUESTIONS 3-50 AND 3-51, REFER TO FIGURE 3H. 3-53. If an open occurs in a parallel branch of a circuit, what is the effect on (a) total resistance, and (b) total current? 1 . 2 . 3. 4. (a) Increases (a) Increases (a) Decreases (a) Decreases (b) decreases (b) increases (b) decreases (b) increases 3-54. If a short circuit occurs in a series portion of a circuit, what is the effect on (a) total resistance, and (b) total current? 1 . 2 . 3. 4. (a) Increases (a) Increases (a) Decreases (a) Decreases (b) decreases (b) increases (b) decreases (b) increases 3-50. What is the value of total resistance? 1. 50 2. 80 3. 130 4. 150 3-51. If an equivalent resistor is used to represent the network of R b R 2 , R 3 , R 4 , R 5 , and R 6 , what is the total voltage drop across this resistor? 1. 8V 2. 26V 3. 52V 4. 60V 3-52. If an open occurs in a series portion of a circuit, what is the effect on (a) total resistance, and (b) total current? 3-55. If a short circuit occurs in a parallel branch of a circuit, what is the effect in (a) total resistance, and (b) total current? 1 . 2 . 3. 4. (a) Increases (a) Increases (a) Decreases (a) Decreases (b) decreases (b) increases (b) decreases (b) increases 3-56. If one branch of a parallel network shorts, what portion of the circuit current, if any, will flow through the remaining branches? 1 . An amount determined by the combined resistance of the remaining branches 2. All 3. One-half 4. None 1 . (a) Decreases to zero (b) Becomes infinite 2. (a) Decreases to zero (b) Decreases to zero 3. (a) Becomes infinite (b) Becomes infinite 4. (a) Becomes infinite (b) Decreases to zero 3-57. Which of the following circuit quantities need NOT be known before designing a voltage divider? 1 . The current of the source 2. The voltage of the source 3. The current requirement of the load 4. The voltage requirement of the load 23 THE FOLLOWING INFORMATION IS TO BE USED IN ANSWERING QUESTIONS 3-58 THROUGH 3-60: A VOLTAGE DIVIDER IS REQUIRED TO SUPPLY A SINGLE LOAD WITH +150 VOLTS AND 300 MILLIAMPS OF CURRENT. THE SOURCE VOLTAGE IS 250 VOLTS. (HINT: DRAW THE CIRCUIT.) 3-58. What should be the value of the bleeder current? 1. 3 A 2. 300 mA 3. 30 mA 4. 3 mA 3-59. What should be the ohmic value of the bleeder resistor? 1. 50 2. 500 3. 5 k 4. 50 k 3-60. What is the value of total current? 1. 303 mA 2. 330 mA 3. 600 mA 4. 3300 mA Figure 31. — Voltage divider. IN ANSWERING QUESTIONS 3-61 THROUGH 3-66, REFER TO FIGURE 31. 3-61. Why must the value of Rj be calculated first? 1 . For convenience 2. The current through R 2 depends on the value of Ri 3. The voltage drop across Ridepends on the value of load 1 4. In any circuit, values for resistors labeled R\ are calculated first 3-62. How is the current through R 2 calculated? 1 . By adding I R i and the current requirement of load 1 2. By adding the current requirements of load 1 and load 2 3. By subtracting the current requirement of load 1 from the current requirement of load 2 4. By subtracting the current requirement of load 2 from the current requirement of load 1 24 3-63. How is the voltage drop across R 2 calculated? 1 . By adding the voltage requirements of load 1 and load 2 2. By subtracting the voltage drops across R 5 and R 3 from the source voltage 3. By subtracting the voltage requirement of load 1 from the voltage requirement of load 2 4. By subtracting the voltage requirements of load 1 and load 2 from the source voltage 3-64. What is the minimum wattage rating required for R 5 ? 1. 1 W 2. 2W 3. 1/2 W 4. 1/4 W 3-65. What is the total power supplied by the source? 1. 3.765 W 2. 7.965 W 3. 8.209 W 4. 8.965 W 3-66. What is the purpose of using the series- parallel network consisting of R 3 , R 4 , and R 5 in place of a single resistor? 1 . It provides the desired resistance with resistor values that are easily obtainable 2. It provides the close tolerance required for the circuit 3. It is more reliable than the use of a single resistor 4. It costs less by using three resistors of lower wattage rating than a single, large power resistor 3-67. A single voltage divider provides both negative and positive voltages from a single source voltage through the use of a 1 . ground between two of the dividing resistors 2 . ground to the positive terminal of the source 3. ground to the negative terminal of the source 4. ground to the input of all loads requiring a negative voltage 3-68. Which of the following voltages are considered dangerous? 1. Voltages above 115 volts only 2. Voltages above 230 volts only 3. Voltages above 450 volts only 4. All voltages 3-69. If you discover a possible malfunction in an electric circuit, which of the following actions should be taken? 1 . Attempt repairs yourself 2. Report the malfunction to a qualified technician 3. Ignore the malfunction unless you were assigned to repair it 4. Secure the circuit immediately by removing power at the nearest switch 3-70. If a person has stopped breathing and there is NO detectable heartbeat, who should perform CPR? 1 . Medical personnel only 2. The first person on the scene 3. Emergency Medical Technicians only 4. Trained, qualified personnel only 25